SlideShare a Scribd company logo
ĐỀ THI THỬ TUYỂN SINH VÀO LỚP 10
THPT MÔN TOÁN CÁC TỈNH NĂM HỌC
2023-2024 CÓ LỜI GIẢI CHI TIẾT
WORD VERSION | 2023 EDITION
ORDER NOW / CHUYỂN GIAO QUA EMAIL
TAILIEUCHUANTHAMKHAO@GMAIL.COM
Đ Ề L U Y Ệ N T H I T U Y Ể N S I N H
L Ớ P 1 0 M Ô N T O Á N
Ths Nguyễn Thanh Tú
eBook Collection
Hỗ trợ trực tuyến
Fb www.facebook.com/DayKemQuyNhon
Mobi/Zalo 0905779594
Tài liệu chuẩn tham khảo
Phát triển kênh bởi
Ths Nguyễn Thanh Tú
Đơn vị tài trợ / phát hành / chia sẻ học thuật :
Nguyen Thanh Tu Group
vectorstock.com/28062405
PHÒNG GIÁO DỤC VÀ ĐÀO TẠO
ĐÔNG ANH
ĐỀ KIỂM TRA KHẢO SÁT LỚP 9
NĂM HỌC 2023 – 2024
Môn: TOÁN
Thời gian làm bài: 120 phút
Bài I ( 2,0 điểm )
Cho hai biểu thức
3
5
x
A
x
−
=
+
và
4 2 13
9
3 3
x x x
B
x
x x
− −
= + +
−
+ −
với 0; 9
x x
 
1) Tính giá trị của biểu thức A khi 25
x =
2) Rút gọn biểu thức B.
3) Cho .
P B A
= . Tìm các giá trị của x thỏa mãn ( )
1 2 3 . 3 5
x x P x
− = + + +
Bài II ( 2,5 điểm )
1) Giải bài toán sau bằng cách lập phương trình hoặc hệ phương trình:
Một người đi xe đạp từ A đến B cách nhau 24 km. Khi từ B về A người đó tăng vận tốc
thêm 4 km/h so với lúc đi. Vì vậy thời gian về ít hơn thời gian đi là 30 phút. Tính vận
tốc của người đi xe đạp đó khi đi từ A đến B?.
2) Một hộp sữa ông thọ hình trụ có chiều cao là 8cm và bán kính đáy là 3,5 cm. Nhà
sản xuất đã dán giấy xung quanh hộp sữa để ghi các thông tin về
sản phẩm. Hãy tính diện tích giấy cần dùng cho 1 hộp sữa ( Coi
mép giấy dán, các mép của hộp sữa và độ dày của giấy in không
đáng kể. Lấy 3,14
  ).
Bài III (2,0 điểm )
1) Giải hệ phương trình sau
5 3 16
2 5 4
x y
x y
 − + =


− − =


2) Cho phương trình 2
2 0
x mx
− − = ( x là ẩn số).
a) Tìm giá trị của mđể phương trình đã cho có hai nghiệm phân biệt.
b) Tìm giá trị của mđể phương trình đã cho có hai nghiệm phân biệt 1 2
,
x x thỏa mãn
2 2
1 2 8
x x
+ =
Bài IV ( 3,0 điểm ): Cho đường tròn tâm ( )
O đường kính . Dây CD vuông góc với
đường kính AB tại H,( H khác O, A và B). E là một điểm thuộc cung nhở BD ( E khác
B và D); AE cắt CD tại F.
1) Chứng minh: Tứ giác BEFH nội tiếp đường tròn.
2) Chứng minh:H là trung điểm của CD và 2
.
4.
CD AH HB
=
3) Đường thẳng qua H song song với CE cắt đường thẳng AE và BE lần lượt tại I và
K. Lấy G là trung điểm của đoạn thẳng IK. Hỏi ∆DGK có là tam giác cân được hay
không?
Bài V ( 0,5 điểm ) Cho x, y, z > 0 và x.y.z = 1 Tìm giá trị nhỏ nhất của biểu thức
2 2 2
1 1 1
x y z
A
y z x
= + +
+ + +
…………. Hết …………
Cán bộ coi thi không giải thích gì thêm
HƯỚNG DẪN CHẤM
KIỂM TRA KHẢO SÁT TOÁN 9
NĂM HỌC 2023 – 2024
Môn: toán
A. Hướng dẫn chung
- Học sinh giải theo cách khác mà đúng, đủ các bước thì giám khảo chấm điểm tối đa. -
Trong mỗi bài, nếu ở một bước nào đó bị sai thì các bước sau có liên quan không được điểm.
- Bài hình học bắt buộc phải vẽ đúng hình thì mới chấm điểm, nếu không có hình vẽ đúng ở
phần nào thì giám khảo không ghi điểm phần lời giải liên quan đến hình của phần đó. - Điểm
toàn bài là tổng điểm của các ý, các câu, tính đến 0,25 điểm và không làm tròn.
B. Nội dung, biểu điểm
BÀI Ý SƠ LƯỢC LỜI GIẢI ĐIỂM
Bài I
(2,0
điểm)
1
(0,5đ) 1) Thay x = 25 (TMĐK) vào biểu thức A ta đc :
25 3
25 5
A
−
=
+
0,25
1
5
A =
0,25
2
(1đ)
4 2 13
9
3 3
x x x
B
x
x x
− −
= + +
−
+ −
0,25
( )( )
4 2 13
3 3
3 3
x x x
B
x x
x x
− −
= + −
+ −
+ −
0,25
4 12 2 13 3
3
x x x x x
B
x
− + − − − −
=
+
0,25
( )( )
25
3 3
x
B
x x
−
=
+ −
0,25
3
(0,5đ)
( )
( ) ( )
( ) ( )
2 2
5
.
3
1 2( 3). 3 5 0; 9
1 2 10 3 5
9 2 3 5 0
2 18 4 6 5 0
5 6 5 9 4 4 0
5 3 2 0
x
P A B
x
x x P x x x
x x x
x x x
x x x
x x x x
x x
−
= =
+
− = + + +  
 − = − + +
 + − − + =
 + − − + =
 + − + + + − + =
 + − + − =
Giải ra tìm được x = 4 ( thỏa mãn ĐKXĐ). Vậy x = 4
0,25
0,25
Bài II
(2,5
điểm)
1
(2đ)
Gọi vận tốc của người đi xe đạp khi đi từ A đến B là x km/h
ĐK x >0 0,25
Thời gian của người đi xe đạp khi đi từ A đến B là:
24
( )
h
x 0,25
Vận tốc của người đi xe đạp khi đi từ B về A là: x + 4 (km/h) 0,25
Thời gian của người đi xe đạp khi đi từ B đến A là
24
( )
4
h
x +
0,25
Vì thời gian về ít hơn thòi gian đi là 30 phút bằng
1
2
giờ nên ta
có phương trình :
24 24 1
4 2
x x
− =
+
0,25
2
4 192 0
x x
 + − = 0,25
12( ) 16( )
x TM x KTM
= = −
Vậy vận tốc của người đi xe đạp từ A đến B là 12 km/h 0,25
1
(0,5đ)
2) 2
2 2.3,14.3,5.8 175,84
xq
S Rh cm

=  = 0,25
Vậy diện tích của phần giấy cần dùng khoảng 175,84cm2
0,25
Bài III
(2,0
điểm)
1
(1đ)
5 3 16
2 5 4
x y
x y
 − + =


− − =


2 5 6 32
2 5 4
x y
x y
 − + =

 
− − =


Đk; : 5
x  0,25
7 28 4
5 3 16 5 16 12 4
y y
x y x
= =
 
 

 
− + = − = − =
 
 
0,25
4
5 16
y
x
=

 
− =

0,25
4
21( )
y
x TM
=

 
=

Vậy hệ pt có nghiệm duy nhất là (21;4)
0,25
2
(1đ)
a) Tính 2
8
m
 = +
0
  với mọi m
0,25
Suy ra phương trình có hai nghiệm phân biệt x1, x2 với mọi m 0,25
b) Áp dụng định lý Vi-ét ta có 1 2
1 2
. 2
x x m
x x
+ =


= −

Để ( )
2
2 2
1 2 1 2 1 2
8 2 . 8
x x x x x x
+ =  + − =
Suy ra 2
4 8
m + =
0,25
2
4 2
m m
 =  = 
Vậy 2
m =  là giá trị cần tìm
0,25
Bài
IV
(3,0
điểm)
Vẽ hình
đúng
đến ý a
0,25
2
(0,75đ)
Xét đường tròn đường kính AB có 0
90
AEB = ( Góc nội tiếp chắn
nửa đường tròn đường kính AB)
0
90
BEF
 =
CD AB
⊥ tại H 0
90
BHF
 =
0,75
Xét tứ giác BEFH có 0
180
BEF
BHF + = 0,25
Mà chúng ở vị trí đối diện nhau nên tứ giác BHFE là tứ giác nội
tiếp đường tròn
0,25
2
(1đ)
Xét đường tròn (O) có AB là đường kính, AB CD
⊥ tại H (gt)
=> CH = HD ( quan hệ đường kính, dây cung)
=> H là trung điểm của đoạn thẳng CD
0,25
0,25
Xét (O) có 0
90
ACB = ( góc nội tiếp chắn nửa đường tròn)
Xét ABC
 vuông tại C, có CH là đường cao
2
.
CH AH HB
 =
0,25
Mà
2
CD
CH = ( H là trung điểm của CD)
Nên 2
4 .
CD AH HB
 =
0,25
3
(1đ)
/ /
HI CE DHI DCE
 = ( hai góc so le trong)
Xét (O) có DAE DCE
= ( hai góc nội tiếp cùng chắn cung DE)
DHI DAE DHI DAI
 =  = DAHI có
Xét tứ giác DAHI có DHI DAI
=
Mà H, A là hai đỉnh kê nhau cùng nhìn cạnh DI
=> Tứ giác AHID nội tiếp đường tròn 0,25
Xét đường tròn ngoại tiếp tứ giác AHID, có:
AHD AID
= ( hai góc nội tiếp cùng chắn cung AD)
Mà
0 0
90 90
AHD AID
= = =
0,25
Xét (O) có DBE DAE
= ( hai góc nội tiếp cùng chắn cung DE)
( )
DAE DAI DHI cmt
DHI DBE
= =
= =
Hay DHK DBK
=
=> Tứ giác DHBK nội tiếp đường tròn
=> 0
180
DHB DKB
+ =
Mà 0 0 0
90 90 90
DHB DKB DKE
= = = = =
0,25
Xét tứ giác DIEK có 0
90
DIE IEK DKE
= = =
=> Tứ giác DIEK là hình chữ nhật
=> IK = DE và IK, DE cắt nhau tại trung điểm G của mỗi đường
Mà GD = GK => ∆DGK cân tại G
0,25
Bài V
(0,5
điểm)
Áp dụng BĐT Cauchy cho hai số dương
2 2
2 2
2 2
1 1
2 .
1 4 1 4
1 1
2 .
1 4 1 4
1 1
2 .
1 4 1 4
x y x y
x
y y
y z y z
y
z z
z x z x
z
x x
 + +
+  =

+ +


+ +

+  =

+ +

 + +
 +  =
+ +


Cộng vế với vế ba BĐT trên ta được
( )
2 2 2
2 2 2
2 2 2
3
1 1 1
1 4 1 4 1 4
3
1 1 1 4 4
3 3 3 3 3
.3.
1 1 1 4 4 4 4 2
x y y z z x
x y z
y z x
x y z x y z
x y z
y z x
x y z
x y z
xyz
y z x
     
+ + +
+ + + + +  + +
     
+ + +
     
+ +
 + +  − − + + +
+ + +
+ +
 + +  −  − =
+ + +
3
2
A
 
0,25
Dấu “=” xảy ra 1( )
x y z TM
 = = =
Vậy giá trị nhỏ nhất của A là
3
2
khi 1
x y z
= = =
0,25
Lưu ý: Các cách làm khác nếu đúng vẫn cho điểm tối đa
PHÒNG GIÁO DỤC VÀ ĐÀO TẠO
QUẬN ĐỐNG ĐA
ĐỀ KHẢO SÁT CHẤT LƯỢNG LỚP 9
NĂM HỌC 2023 – 2024
Môn: TOÁN
Ngày khảo sát; 17/4/2024
Thời gian làm bài: 120 phút
Bài I ( 2,0 điểm )
Cho hai biểu thức
2 6
1
x
A
x
+
=
−
và
2
2
2
x x
B
x
x x
+
= −
+
+ −
với 0, 1.
x x
 
1) Tính giá trị của biểu thức A khi 9.
x =
2) Rút gọn B.
3) Chứng minh 6
A
B

Bài II ( 2,5 điểm )
1) Giải bài toán sau bằng cách lập phương trình hoặc hệ phương trình:
Một ô tô đi từ A đến B dài 90km. Khi về ô tô đi theo đường khác dài hơn 10km
và mỗi giờ ô tô đi được nhiều hơn lúc đi 10km nên thời gian về ít hơn thời gian đi là 15
phút. Tính vận tốc lúc đi và lúc về?.
2) Một lon nước ngọt hình trụ có đường kính đáy là 6cm, độ dài trục là 11cm.
Tính thể tích lon nước ngọt ( cho 3,14
  ).
Bài III (2,0 điểm )
1) Giải hệ phương trình sau
1 4
3
2 1 5
3 2
5
2 1 5
x y
x y

+ =
 − +


 − = −
 − +

2) Trong mặt phẳng toạ độOxy , cho parabol ( ) 2
:
P y x
= và đường thẳng
( ): 1
d y mx m
= − + (mlà tham số ).
a) Tìm mđể ( )
d cắt ( )
P tại 2 điểm phân biệt.
b) Gọi 1 2
,
x x là hoành độ giao điểm của ( )
d và ( )
P tìm tất cả các giá trị của m
để 2
1 2
x x
=
Bài IV ( 3,0 điểm ): Cho đường tròn tâm ( )
O đường kính ABvà d là một tiếp tuyến
của đường tròn ( )
O tại điểm A. Trên đường thẳng d lấy điểm M (khác A) và trên
đoạn OBlấy điểm N (khác O và B). Đường thẳng MN cắt đường tròn ( )
O tại hai điểm
và D (C nằm giữa M và D ). Gọi H là trung điểm của đoạn thẳngCD .
1) Chứng minh 4 điểm , , ,
A O H M cùng nằm trên một đường tròn.
2) Chứng minh 2
.
MA MD MC
=
3) Đường thẳng qua D song song với MOcắt ABvà BC lần lượt tại K và F .
Chứng minh tứ giác AHKDnội tiếp và K là trung điểm của đoạn thẳng DF .
Bài V ( 0,5 điểm ) Cho hai số thực a và b thoả mãn điều kiện 2
( 1)
a b ab
+ − = . Tìm giá
trị nhỏ nhất của biểu thức
9
P
a b
a b
= +
+
+
…………. Hết …………
Cán bộ coi thi không giải thích gì thêm
HƯỚNG DẪN CHẤM KHẢO SÁT TOÁN 9
NĂM HỌC 2023 – 2024
(Ngày khảo sát 17.4.2024)
BÀI SƠ LƯỢC LỜI GIẢI ĐIỂM
Bài I
(2,0
điểm)
Cho hai biểu thức
2 6
1
x
A
x
+
=
−
và
2
2
2
x x
B
x
x x
+
= −
+
+ −
với 0, 1.
x x
 
2,0
1) Tính giá trị biểu thức A khi x = 9. 0,5
+ Thay x = 9 (TMĐK) vào biểu thức A ta đc :
2 9 6 12 3
9 1 8 2
A
+
= = =
−
Vậy giá trị của biểu thức
3
2
A = khi 9
x =
0,25
0,25
2) Rút gọn B(đk 0, 1.
x x
  ) 0,5
Với điều kiện 0, 1
x x
  ta có:
2 2
2 ( 2)( 1) 2
2
x x x x
B
x x x x
x x
+ +
= − = −
+ + − +
+ −
0,25
2 ( 1) 2
( 2)( 1) ( 2)( 1)
x x x x x x
x x x x
+ − − + − +
= =
+ − + −
0,25
2
( 2)( 1)
x
x x
+
=
+ −
0,25
1
1
x
=
−
với 0, 1
x x
 
0,25
3) Chứng minh 6
A
B

0,5
Ta có
2 6 4
2
1 1
A x
B x x
+
= = +
+ +
0,25
4
2 2 4 6
1
x
+  + =
+
với 0, 1
x x
 
0,25
Bài II
(2,5
điểm)
1) Một ô tô đi từ A đến B dài 90km. Khi về ô tô đi theo đường khác dài
hơn 10km và mỗi giờ ô tô đi được nhiều hơn lúc đi 10km nên thời gian
về ít hơn thời gian đi là 15 phút. Tính vận tốc lúc đi và lúc về?.
2đ
+ Gọi vận tốc ô tô lúc đi từ A đến B là x (km/h; x >0); đổi 15 phút =
1
4
(h)
0,25
+ Thời gian ô tô đi từ A đến B là:
90
( )
h
x
0,25
+ Vận tốc ô tô đi từ B về A là : x + 10 (km/h) 0,25
+ Thời gian ô tô đi từ B về A là
90 10 100
( )
10 10
h
x x
+
=
+ +
0,25
=> Ta có phương trình :
90 100 1
10 4
x x
− =
+
0,25
+ Giải phương trình thu được x = 40 (tmđk) và x = - 90 (loại) 0,5
+ Kết luận 0,25
2) Một lon nước ngọt hình trụ có đường kính đáy là 6cm, độ dài trục
là 11cm. Tính thể tích lon nước ngọt ( cho 3,14
  ).
0,5
Ta có R= 3cm, h=11cm nên ( )
2 3
V = R h = 3,14.9.11= 310,86 cm
 0,25
Vậy thể tích của lon nước ngọt là 310,86 cm3 0,25
Bài III
(2,0
điểm) 1. Giải hệ :
1 4
3
2 1 5
3 2
5
2 1 5
x y
x y

+ =
 − +


 − = −
 − =

1,0
+ Đk; :
1
; 5
2
x y
  −
Đặt
1
2 1
1
5
a
x
b
y

=
 −


 =
+


0,25
+ Giải được 1; 1
a b
= − = 0,25
Thay
1
1 2 1 1 0
2 1
x x
x
+ =  − = −  =
−
( TMĐK)
1
1 5 1 4
5
y y
y
+ =  + =  = −
+
( TMĐK)
0,25
+ Kết luận 0,25
2) Trong mặt phẳng toạ độOxy , cho parabol ( ) 2
:
P y x
= và đường thẳng
( ): 1
d y mx m
= − + ( mlà tham số ).
1,0
a) Tìm mđể ( )
d cắt ( )
P tại 2 điểm phân biệt. 0,5
Xét phương trình hoành độ giao điểm của đường thẳng ( )
d và parabol ( )
P
2 2
1 1 0
1 1 0
(1)
x mx m x mx m
a b c m m
= − +  − + − =
+ + = − + − =
Nên phương trình (1) có nghiệm là 1
x = và 1
x m
= −
0,25
+ Vì ( )
d cắt ( )
P tại 2 điểm phân biệt nên 1 1 2
m m
 −   0,25
b) 1 2
,
x x thỏa mãn 2
1 2
x x
= 0,5
TH1: ( )
2 2( )
1 1
0( )
m TM
m
m TM
=

= −   =

Vậy m = 0 thì….
0,25
TH2: 2
1 1 2( )
m m TM
= −  = 0,25
Bài
IV
(3,0
điểm)
Vẽ hình đúng đến câu 1
0,25
1) Chứng minh 4 điểm , , ,
A O H M cùng nằm trên một đường tròn. 0,75
Vì MA là tiếp tuyến của 0
( ) 90
O MAO
 =
H là trung điểm của CD OH CD
 ⊥ tại H 0
90
OHC OHM
 = = 0,25
Xét tứ giác AOHM có: 0 0 0
90 90 180
MAO OHM
+ = + = mà hai góc này là
hai góc đối nhau nên tứ giác AOHM là tứ giác nội tiếp
0,25
Suy ra 4 điểm , , ,
A O H M cùng nằm trên một đường tròn. (đpcm) 0,25
2) Chứng minh 2
.
MA MD MC
= 1,0
Xét ∆ AMC và ∆ DMA có AMDchung 0,25
+) MDA MAC
= ( góc nội tiếp và góc tạo bởi tiếp tuyến với dây cung cùng
chắn cung AC)
0,25
∆ AMC∽∆ DMA( g – g) 0,25
2
.
AM MC
MA MC MD
DM MA
 =  = (đpcm)
0,25
3) Chứng minh tứ giác AHKDnội tiếp và K là trung điểm của đoạn
thẳng DF
1,0
Ta có: / / ( )
DK MO gt MDK DMO
 = ( hai góc so le trong)
Vì tứ giác AOHM là tứ giác nội tiếp (cm câu 1) HMO HAO
 =
( góc nội tiếp cùng chắn cung HO )
( )
BAH MDK DMO
 = = 0,25
Tứ giác AHKDlà tứ giác nội tiếp (hai đỉnh kề cùng nhìn cạnh đối diện
dưới các góc bằng nhau)
0,25
Tứ giác AHKD nội tiếp DAK DHK
= (góc nội tiếp cùng chắn DK)
DAB DCB
= (góc nội tiếp cùng chắn BD) DCB DHK
 =
0,25
Mà ;
DCB DHK ở vị trí đồng vị nên / / / /
HK CB HK CF
 .
Mặt khác H là trung điểm của CD K
 là trung điểm của DF
0,25
Bài V
(0,5
điểm)
Cho hai số thực a và b thoả mãn điều kiện 2
( 1)
a b ab
+ − = . Tìm giá trị
nhỏ nhất của biểu thức
9
P
a b
a b
= +
+
+
0,5
( )
( ) ( )
2
2
9 4 5 5 5 13
4
2 . 4
2 2
2
1 2
2 3
P a b a b
a b a b a b
a b
a b
a b
a b ab a b
a b
= + = + + +  + +  + =
+ + +
+
+
+
 
+ −     + 
 
 
+
0,25
Dấu “=” có
13
2
P = khi 1
a b
= =
KL:
13
min 2
P = khi 1
a b
= =
0,25
Lưu ý: Các cách làm khác nếu đúng vẫn cho điểm tối đa
UBND QUẬN HỒNG BÀNG
TRƯỜNG THCS HỒNG BÀNG
ĐỀ THI THỬ VÀO LỚP 10 THPT
Năm học 2023 – 2024
Môn: Toán 9
Thời gian làm bài: 120 phút
Bài 1. (1,5 điểm)
Cho ( )
2 2 1 1 1
3 2 3 1; :
1 2 1
x x
A B
x x x x
   
− +
= − + − = − −
   
− + −
 
 
với 0, 1.
x x
 
a) Rút gọn biểu thức A, B .
2) Tìm các giá trị của x để giá trị của biểu thức B bằng giá trị của biểu thức A
Bài 2. ( 1,5 điểm )
1. Cho hàm số 2 1
y mx m
= + − có đồ thị là (d) . Tìm giá trị của m biết rằng (d) cắt
đường thẳng 3 2
y x
= − tại điểm có hoành độ bằng 2.
2. Một mảnh vườn hình chữ nhật có các kích thước là 20m và 30m. Người ta tăng
các kích thước của nó lên mỗi chiều x(m) được hình chữ nhật mới có chu vi là y (m).
a) Hãy lập hàm số y theo x.
b) Nếu chu vi của mảnh vườn sau khi mở rộng là 200m. Hãy tính các kích thước
của mảnh vườn sau khi mở rộng
Bài 3. (2,5 điểm )
1. Cho phương trình ( )
2
2 3 0 (1)
x m x m
+ − + − = ( m là tham số)
a) Chứng minh với mọi m phương trình (1) luôn có nghiệm.
b) Tìm m để phương trình (1) có hai nghiệm âm phân biệt.
2. Một phòng họp có 360 ghế và được chia thành các dãy có số ghế bằng nhau. Nếu
thêm cho mỗi dãy 4 ghế và bớt đi 3 dãy thì số ghế trong phòng không thay đổi. Hỏi ban đầu
có bao nhiêu dãy ghế và trong mỗi dãy có bao nhiêu ghế?
Bài IV ( 0,75 điểm ):
Một bồn đựng nước dạng hình hộp chữ nhật có các kích thước trên hình bên dưới.
a) Tính diện tích bề mặt của bồn (không tính nắp).
b) Một vòi bơm công suất 120lít/phút để bơm một lượng
nước vào bồn lên độ cao cách nắp bồn là 1,5m thì phải mất
bao nhiêu phút? (cho biết ban đầu bồn không chứa nước).
Bài V ( 3 điểm ) Từ điểm A nằm ngoài (O) , vẽ hai tiếp tuyến AB, AC ( B, C là hai tiếp
điểm), gọi H là giao điểm của OA và BC .
a) Chứng minh tứ giác OBAC nội tiếp và OA BC
⊥
b) Kẻ đường kính BK của (O), cho AK cắt (O) tại điểm E ( E K
 ) Chứng minh
2
.
AB AE AK
= và tứ giác OHEK nội tiếp.
c) Tia BK và tia AC cắt nhau tại F, kẻ ( )
CI BK I BK
⊥  , AK và CI cắt nhau tại
M. Gọi N là trung điểm của AB. Chứng minh ba điểm F, M, N thẳng hàng.
Bài VI (0,75 đ)
Cho x, y là các số dương và x + y = 2. Chứng minh 2 2
1 1
3 4
P x y
x y
 
= + + 
 
 
…………. Hết …………
Họ và tên thí sinh: ………………………….. Số báo danh: ……………………………....
Giám thị 1: …………………………………... Giám thị 2: ………………..………………
ĐÁP ÁN VÀ BIỂU ĐIỂM KỲ ĐỀ THI THỬ LẦN 1
MÔN TOÁN LỚP 9
BÀI SƠ LƯỢC LỜI GIẢI ĐIỂM
Bài 1
(1,5
điểm)
a) 1,0 điểm
( )
2
3 2 3 1 3 2 3 1
A = − + − = − + −
0,25
2 3 3 1 1
= − + − = 0,25
( )( ) ( )( ) ( )
4 1
:
1 . 2 1 . 2
1
1
x x
B
x x x x x x
 
− −
 
= −
 
− + − +
 
−
− 0,25
( ) ( )
( )
1
3 3
.
1 2
1 . 2
x x x
B
x
x x
−
−
=  =
− +
− + 0,25
b) 0,5 điểm
Cho
3
1
2
x
A B
x
=  =
+
0,25
3 2 2 2 1 1
x x x x x
 = +  =  =  = ( không thỏa mãn) 0,25
Bài 2
(1,5
điểm)
a) 0,75 điểm
Để (d) 2 1
y mx m
= + − cắt đường thẳng 3 2
y x
= − thì
3
2 3 (*)
2
m m
  
Gọi điểm có hoành độ bằng 2 là 0
(2; )
A y
Vì 0
(2; )
A y thuộc 3 2
y x
= − nên 0 3.2 2 4
y = − =
Do đó (2;4)
A
0,25
Vì (2;4)
A thuộc (d) nên 4 2 .2 1
m m
= + −
5 5 1
m m
 =  = (thỏa mãn điều kiện (*)) 0,25
Vậy đường thẳng (d) cắt đường thẳng 3 2
y x
= − tại điểm có hoành độ
bằng 2 khi m = 1
0,25
b) 0,75 điểm
Chu vi mảnh vườn là ( )
2 20 30 4 100
y x x x
= + + + = + 0,25
Chu vi là 200m nên ta có 4 100 200 25
x x
+ =  = 0,25
Vậy kích thước của mảnh vườn sau khi mở rộng là 45m và 55m 0,25
Bài 3
(2,0
điểm)
1a ( 0,5 điểm)
( )
2
) 2 3 0 (1)
a x m x m
+ − + − = ( m là tham số)
( ) ( ) ( )
2 2
2
2 4.1. 3 8 16 4 0
m m m m m m R
 = − − − = − + = −   
0,25
Phương trình có nghiệm với mọi m 0,25
1b ( 1,0 điểm)
( ) ( )( )
2
) 2 3 0 1 3 0
b x m x m x x m
+ − + − =  − + − =
Do đó phương trình (1) có nghiệm 1 2
1; 3
x x m
= − = − 0,25
Vì 1 1 0
x = −  nên phương trình (1) có hai nghiệm phân biệt 0,5
2
2 1
0 3 0 3
3 1 4
x m m
x x m m
 −  
  
  
  
 −  − 
 
 0,25
Vậy với 3; 4
m m
  thì phương trình đã cho có hai nghiệm âm phân biệt. 0,25
2. (1,0 điểm)
Gọi x là số dãy ghế trong phòng họp ( x > 3 và x là số nguyên dương) 0,25
Số ghế trên một dãy là
360
x
(ghế) nếu bớt đi 3 dãy thì số dãy ghế còn lại là
3
x − (dãy)
Số ghế trên một dãy là
360
3
x −
(ghế) 0,25
Do số ghế trong mỗi dãy tăng thêm 4 ghế nên ta có phương trình
2
18( )
360 360
4 3 270 0
15( )
3
x tm
x x
x ktm
x x
=

− =  − − =  
= −
− 
0,25
Vậy số dãy ghế là 18 (dãy) và số ghế trong một dãy là 20 (ghế) 0,25
Bài 4
(0,75
điểm)
Diện tích bề mặt của bồn :
( ) 2
3,1.2,3 11,5.2,3 .2 11,5.3,1 102,81( )
S m
= + + =
Một vòi bơm công suất 120 lít/phút để bơm một lượng nước vào bồn lên
độ cao cách nắp bồn là 1,5m thì phải mất bao lâu? (bồn không chứa nước).
Thể tích của nước cần bơm là:
3
11,5.3,1.(2,3 1,5) 28,52( )
V m
= − =
Lại có 3
1 1000
m l
= vậy 3
28,52( ) 28520
m = ( lít)
Lại có, 1 phút bơm được 120 lít nên 28520( lít) sẽ bơm trong
28520
237,67
120
 ( phút)
Bài 5
(3,0
điểm)
Vẽ hình đúng đến câu a
0,25
a) 1,0 điểm Chứng minh tứ giác OBAC nội tiếp và OA BC
⊥
0
90
OBA OCA
= = ( AB, AC là tiếp tuyến của (O)) 0,25
0
180
OBA OCA
 + =
Vậy tứ giác OBAC nội tiếp (tứ giác có tổng hai góc đối bằng 1800
)
0,25
Ta có OB = OC (bán kính (O) )
AB = AC (tính chất hai tiếp tuyến cắt nhau)
0,25
=> OA là đường trung trực của BC => OA BC
⊥ 0,25
b) 1,0 điểm Chứng minh 2
.
AB AE AK
= và tứ giác OHEK nội tiếp.
0
90
BEK = ( (góc nội tiếp chắn nửa đường tròn (O) ) => BE AK
⊥ 0,25
Xét ABK
 vuông tại B, có đường cao BE
2
.
AE AK AB
= (hệ thức lượng trong tam giác vuông). 0,25
Xét ABO
 vuông tại B, có đường cao BH:
2
.
AH AO AB
= (hệ thức lượng trong tam giác vuông).
2
. . ( )
AH AO AE AK AB
= = = 0,25
Xét AHE
 và AKO
 có OAK chung,
AH AE
AK AO
= ( vì . .
AH AO AE AK
= )
( )
AHE AKO cgc AHE AKO
  =
 ∽
Vậy tứ giác OHEK nội tiếp (tứ giác có góc ngoài bằng góc đối trong)
0,25
c) 0,75 điểm Chứng minh: ba điểm F, M, N thẳng hàng. 1,0
Gọi S là giao điểm của KC và BA
0
90
BCK = (góc nội tiếp chắn nửa đường tròn (O) ) BC SK
 ⊥
BKS
 có O là trung điểm của BK; OA//KS ( cùng vuông góc với BC)
=> A là trung điểm của BS => AB = AS
/ / ( )
IM KM
IM AB BK
BA KA
⊥ = = (hệ quả Talet trong KBA
 )
/ / ( )
CM KM
CM AS BK
AS KA
⊥ = = (hệ quả Talet trong KSA
 )
IM CM KM
BA AS KA
 
= = =
 
 
Mà BA = AS nên IM = CM => M là trung điểm của IC 0,25
Gọi FM cắt BA tại N’.
/ / ' ( )
' '
IM FM
IM N B BK
BN FN
⊥ = = (hệ quả Talet trong '
FBN
 )
/ / '( )
' '
CM FM
CM AN BK
AN FN
⊥ = = (hệ quả Talet )
' ' '
IM CM FM
BN AN FN
 
= = =
 
 
0,25
Mà IM = CM nên BN’ = AN’=> N’ là trung điểm của BA.
Mà N là trung điểm của BA nên N trùng N’
Mà N’, F, M thẳng hàng nên N, F, M thẳng hàng
0,25
Bài 6
(0,75
điểm)
Áp dụng AM-GM, ta có 2 2 2 2
1 1 1
. 3 2 . 3
P x y x y
x y xy
 
= + +  +
 
 
0,5
Áp dụng AM-GM, ta có 2 2 1
x y xy xy
= +  =  0,25
Suy ra
3 1 2 1 2
2 2 2 2 . 4
P xy xy xy
xy xy xy xy xy
 + = + +  +  (đpcm)
Dấu “=” xảy ra khi 1
x y
= =
0,25
UBND QUẬN BA ĐÌNH
PHÒNG GIÁO DỤC VÀ ĐÀO TẠO
ĐỀ CHÍNH THỨC
ĐỀ KHẢO SÁT CHẤT LƯỢNG LỚP 9
NĂM HỌC 2022 – 2023
Môn: TOÁN 9
Ngày kiểm tra: 10/3/2023
Thời gian làm bài: 120 phút
Bài I ( 2,0 điểm )
Cho hai biểu thức
2
x
A
x
+
= và
3 4 1
2
2
x x
B
x
x x
− +
= −
−
−
với 0; 4
x x
 
1) Tính giá trị của biểu thức A khi 9.
x =
2) Chứng minh rằng
2
x
B
x
−
= .
3) Cho :
P A B
= tìm số tự nhiên x để biểu thức P đạt giá trị lớn nhất
Bài II ( 2,5 điểm )
1) Giải bài toán sau bằng cách lập phương trình hoặc hệ phương trình:
Hai bạn Minh và An xuất phát cùng một lúc từ địa điểm A để đi đến địa điểm B bằng
phương tiện xe đạp điện. Mỗi giờ bạn Minh đi nhanh hơn bạn An 2 km nên bạn Minh đến B sớm
hơn bạn An 2,5 phút. Biết quãng đường AB dài 13 km, tính vận tốc xe của mỗi người. Hỏi Minh
và An đi như vậy có đúng vận tốc quy định hay không nếu căn cứ theo quy định vận tốc tối đa của
xe đạp điện là 25km/h.
2) Một ly rượu bằng thủy tinh phần đựng rượu dạng hình nón
có đường kính miệng ly là 9 cm, chiều cao hình nón (như hình vẽ)
là 6 cm. Hỏi ly đó có thể chứa đầy được bao nhiêu mililiter (ml)
rượu? (lấy 3,14
  và coi độ dày thành ly là không đáng kể).
Bài III (2,0 điểm )
1) Giải hệ phương trình sau
2
3 5
1
1
2 1
1
y
x
y
x

+ =
 −


 − = −
 −

2) Trong mặt phẳng toạ độOxy , cho đường thẳng 2
( ) : 2( 1) 3
d y m x m m
= − − + parabol
( ) 2
:
P y x
= (mlà tham số ).
a) Với 3
m = tím tọa giao điểm của ( )
d cắt ( )
P .
b) Tìm mđể ( )
d cắt ( )
P tại 2 điểm phân biệt có hoành độ là số đo chiều dài và chiều rộng
của hình chữ nhật có diện tích bằng
7
4
(đvdt).
Bài IV ( 3,0 điểm ): Cho tam giác ABC vuông tại A. Đường tròn( )
;
O R đường kính AB cắt đoạn
thẳng BC tại điểm thứ hai là D. Kẻ đường thẳng AH vuông góc với đường thẳng OC tại điểm H;
đường thẳng AH cắt đoạn thẳng BC tại điểm M.
1) Chứng minh tứ giác ACDH là tứ giác nội tiếp.
2) Chứng minh 2
.)
OH OC R
= và ∆OHB đồng dạng với ∆OBC
3) Từ O kẻ đường thẳng vuông góc với BD tại K. Chứng minh HM là tia phân giác của góc
DHB và . . .
MB MD MK MC
=
Bài V ( 0,5 điểm ) Cho ab, là các số thực không âm thỏa mãn a b + =1. Tìm giá trị lớn nhất
và giá trị nhỏ nhất của biểu thức ( 1) ( 1)
P a b b a
= + + +
…………. Hết …………
HƯỚNG DẪN CHẤM BÀI KIỂM TRA KHẢO SÁT TOÁN LỚP 9
NĂM HỌC 2022 – 2023
BÀI NỘI DUNG ĐIỂM
Bài I
(2,0
điểm)
Cho hai biểu thức
2
x
A
x
+
= và
3 4 1
2
2
x x
B
x
x x
− +
= −
−
−
với
0; 4
x x
 
1) Tính giá trị của biểu thức A khi 9.
x =
0,5
Thay x = 9 (TMĐK) vào biểu thức A 0,25
Ta tính được :
5
3
A =
0,25
2) Chứng minh rằng
2
x
B
x
−
= .
1,0
3 4 1
2
2
3 4 1
2
( 2)
x x
B
x
x x
x x
x
x x
B
− +
= −
−
−
− +
−
−
−
= 0,25
( )
3 4
( 2)
2
2
( 2)
x x x
B
x x
x
B
x x
− + −
=
−
−
=
−
0,25
2
x
B
x
−
= (điều phải chứng minh)
0,25
3) Cho :
P A B
= tìm số tự nhiên x để biểu thức P đạt giá trị lớn
nhất
0,5
2 2
: :
2
.
2
x x
P A B
x x
x x
P
x x
+ −
= =
+
=
−
2
2
x
P
x
+
=
−
với 0; 4
x x
 
4
1
2
P
x
= +
−
Trường hợp 1 : 0 4 0
x P
   
Trường hợp 2 : 4, 5
x x N x
   
5
2 5 2
4 4
2 5 2
x
x
x
 
 −  −
 
− −
4 4
1 1
2 5 2
x
 +  +
− −
5 2
5 2
P
+
 
−
9 4 5
P
  +
0,25
Kết hợp 2 trường hợp  P đạt giá trị lớn nhất là 9 4 5
+ khi x = 5 (thỏa
mãn điều kiện
0,25
Bài II
(2,0
điểm)
1) Giải bài toán sau bằng cách lập phương trình hoặc hệ phương
trình:
Hai bạn Minh và An xuất phát cùng một lúc từ địa điểm A
để đi đến địa điểm B bằng phương tiện xe đạp điện. Mỗi giờ bạn
Minh đi nhanh hơn bạn An 2 km nên bạn Minh đến B sớm hơn bạn
An 2,5 phút. Biết quãng đường AB dài 13 km, tính vận tốc xe của
mỗi người. Hỏi Minh và An đi như vậy có đúng vận tốc quy định
hay không nếu căn cứ theo quy định vận tốc tối đa của xe đạp điện
là 25km/h.
1,5
Gọi vận tốc xe của bạn An là x (km/h, x > 0) 0,25
Khi đó vận tốc xe của bạn Minh là x + 2 (km/h 0,25
Thời gian bạn An đi hết quãng đường AB là:
13
x
(h)
Thời gian bạn Minh đi hết quãng đường AB là:
13
2
x +
(h)
0,25
Vì bạn Minh đến nơi sớm hơn An 2,5 phút nên ta có phương trình
2
13 13 1
2 624 0
2 24
x x
x x
− =  + − =
+
(1)
0,25
Giải phương trình (1) ta được x1 = 24 (TMĐK); x2 = −26 (Loại) 0,25
Vận tốc xe của An là 24 km/h, vận tốc xe của Minh là 26 km/h.
Vậy bạn An đi đúng vận tốc quy định, còn bạn Minh đi không đúng vận
tốc quy định.
0,25
2) Một ly rượu bằng thủy tinh phần đựng rượu dạng hình nón có
đường kính miệng ly là 9 cm, chiều cao hình nón (như hình vẽ) là 6
cm. Hỏi ly đó có thể chứa đầy được bao nhiêu mililiter (ml) rượu?
(lấy 3,14
  và coi độ dày thành ly là không đáng kể).
0,5
Thể tích hình nón là 2 2 3
1 1
(9: 2) .6 40,5 127,17( )
3 3
V r h cm
  
= = = 
127,17
V ml
 
0,25
Vậy ly có thể chứa đầy khoảng 127,17 cm3
0,25
Bài III
(2,0
điểm) 1) Giải hệ phương trình sau
2
3 5
1
1
2 1
1
y
x
y
x

+ =
 −


 − = −
 −

1,0
Đk; : 1
x 
2 2
3 5 3 5 2
3 5
1 1
1
1 2
2 1 4 2 7 7
1 1
y y
y
x x
x
y y y
x x
 
+ = + = 
  + =
− −
   −
 
  
  
− = − − = − =

 
− −
 
0,25
0,25
0( )
1
1 1
2( )
1
1
x tm
y
x
x tm
y
y
 =



=
 − =
 

 
  =
= 

 
=


0,25
Vậy hệ phương trình đã cho có 2 nghiệm ( ) ( ) ( )
 
; 0;1 ; 2;1
x y  0,25
2) Trong mặt phẳng toạ độOxy , cho đường thẳng
2
( ) : 2( 1) 3
d y m x m m
= − − + parabol ( ) 2
:
P y x
= (mlà tham số ).
a) Với 3
m = tím tọa giao điểm của ( )
d cắt ( )
P .
1,0
Hoành độ giao điểm của ( )
d và ( )
P là nghiệm của phương trình:
2 2 2 2
2( 1) 3 2( 1) 3 0 (*)
x m x m m x m x m m
= − − +  − − + − =
Thay 3
m = vào (*) ta có:
( )
2
4 0 – 4 0 0
x x x x x
− =  =  = hoặc 4
x =
0,5
Với 1 1
0 0
x y
=  =
Với 2
2 2
0 4 16
x y
=  = =
0,25
Vậy với m = 3 thì ( )
d cắt ( )
P tại hai điểm phân biệt ( )
0;0
A và
( )
4;16
B
0,25
b) Tìm mđể ( )
d cắt ( )
P tại 2 điểm phân biệt có hoành độ là số đo
chiều dài và chiều rộng của hình chữ nhật có diện tích bằng
7
4
(đvdt).
0,5
Xét phương trình (*) 2 2
2( 1) 3 0
x m x m m
− − + − =
( )
2
2 2 2 2
1; 2( 1); 3
' 1 1( 3 ) 2 1 3 1
a b m c m
m m m m m m m m
= = − = −
= − − − = − + − + = +
( )
d cắt ( )
P tại 2 điểm phân biệt khi ∆’>0
1 0
1
m
m
 + 
  −
0,25
Theo Vi- et có 2
1 2 1 2
: 2 – 2 ; . – 3
x x m x x m m
+ = =
Để 1 2
,
x x là chiều dài và chiểu rộng của hình chữ nhật có diện tích bằng
7
4
thì cần có:
1 2
0; 0
x x
  sao cho 1 2 2
1
7
.
4 4 12 7 0 (**)
m
x x
m m


=  
− − =

Giải PT(**) suy ra 1
1
2
m = − ( loại); 2
7
2
m = (Thỏa mãn)
Vậy
7
2
m = là giá trị cần tìm.
0,25
Bài
IV
Cho tam giác ABC vuông tại A. Đường tròn( )
;
O R đường kính AB
cắt đoạn thẳng BC tại điểm thứ hai là D. Kẻ đường thẳng AH
vuông góc với đường thẳng OC tại điểm H; đường thẳng AH cắt
đoạn thẳng BC tại điểm M.
1) Chứng minh tứ giác ACDH là tứ giác nội tiếp.
1,0
Vẽ hình đúng đến câu a 0,25
(3,0
điểm)
Chứng minh được 0
90
AHC ADC
= = 0,25
Mà 2 đỉnh H và D là 2 đỉnh kề nhau 0,25
Suy ra tứ giác ACDH là tứ giác nội tiếp (đpcm) 0,25
2) Chứng minh 2
.)
OH OC R
= và ∆OHB đồng dạng với ∆OBC 1,0
Chứng minh 2
.
OH OC OA
= mà OA OB R
= = nên 2
.
OH OC R
= 0,25
Suy ra 2
.
OH OC OB
= suy ra
OH OB
OB OC
=
0,25
Xét ∆OHBvà ∆OBC có
HOB chung ;
OH OB
OB OC
=
0,25
Suy ra ∆OHB∽ ∆OBC (c.g.c) 0,25
3) Từ O kẻ đường thẳng vuông góc với BD tại K. Chứng minh HM
là tia phân giác của góc DHB và . . .
MB MD MK MC
=
1,0
∆OHB∽ ∆OBC (cmt) => OHB OBC
=
Chứng minh ;
CHD CAD CAD OBC
= =
Suy ra OHB CHD
=
Từ đó suy ra DHM BHM
= hay HM là tia phân giác của BHD
0,25
∆ DHB có HM là đường phân giác trong
MD HD
MB HB
 =
Chứng minh HC là phân giác ngoài của ∆ DHB
CD HD
CB HB
= =
Vậy . .
MD CD
MD CB MB CD
MB CB
=  =
0,25
Vì OK vuông góc với BD tại K nên K là trung điểm của BD Từ đó
suy ra
.( ) .( )
2 . ( )
MD MB MC MB MC MD
MD MB MC MB MD
+ = −
 = − 0,25
2 . 2 .
. .
MD MB MK MC
MB MD MK MC
 =
 = 0,25
Bài V
(0,5
điểm)
Cho ab, là các số thực không âm thỏa mãn a b + =1. Tìm giá trị lớn
nhất và giá trị nhỏ nhất của biểu thức ( 1) ( 1)
P a b b a
= + + +
0,5
+) Tìm giá trị nhỏ nhất của biểu thức P
Vì , 0 0
a b ab
   . Ta có ( 1) ( 1)
a b ab a a b a
+ = +  + 
Tương tự có ( 1)
b a b
+  Suy ra 1
P a b a b P
 +  +  
Dấu “=” xảy ra khi
, 0
1
1
0
0
a b
a
a b
b
ab


=


+ = 
 
=

 =

hoặc
0
1
a
b
=


=

Vậy 1
MinP = 
1
0
a
b
=

 
=

hoặc
0
1
a
b
=


=

0,25
+ ) Tìm giá trị lớn nhất của biểu thức P
Ta có 2 3 2 3 ( 1) 2 3 ( 1)
P a b b a
= + + +
Áp dụng bất đẳng thức Cauchy với hai số không âm 3 , 1
a b+ ta có:
2 3 ( 1) 3 1
a b a b
+  + +
Tương tự 2 3 ( 1) 3 1
b a b a
+  + +
Suy ra 2 3 3 1 3 1 2 3 4 4 2
P a b b a P a b
 + + + + +   + +
2 3 6 3
P P
   
Dấu “=” xảy ra khi
, 0
1 1
3 1 2
3 1
a b
a b
a b
a b
b a


 + =

  = =

= +

 = +

KL:
1
3
2
MaxP a b
=  = =
0,25
Lưu ý: Các cách làm khác nếu đúng vẫn cho điểm tối đa
TRƯỜNG THPT SƠN TÂY ĐỀ THI THỬ KỲ THI TUYỂN SINH LỚP 10 THPT ( LẦN 2)
NĂM HỌC 2023 – 2024
Môn thi: TOÁN
Thời gian làm bài: 120 phút
Bài I ( 2,0 điểm
Cho hai biểu thức
1
1 1
x
A
x x
= +
− −
và
1
x
B
x x
=
+ +
với 0, 1.
x x
 
a) Tính giá trị của biểu thức B khi 4.
x =
b) Rút gọn biểu thức M = AB.
c) Khi x > 1 hãy so sánh giá trị biểu thức M với 1.
Bài II ( 2,0 điểm )
1) Giải bài toán sau bằng cách lập phương trình hoặc hệ phương trình:
Tuần trước, Khuê mua 1 kg táo và 1 kg cam ở siêu thị hết 130 nghìn đồng. Hôm nay, Khuê
quay lại siêu thị mua cùng lượng hoa quả như vậy nhưng phải trả 154 nghìn đồng. Người bán hàng
giải thích giá cam đã tăng 15% và giá táo đã tăng 20% so với tuần trước. Hỏi giá tiền mỗi kg táo và
cam ngày hôm nay là bao nhiều nghìn đồng.
2) Quả bóng rổ size 7 có đường kính 24,5 cm. Tính diện tích bề mặt quả
bóng rổ đó ( lấy 3,14
  ).
Bài III (2,5 điểm )
1) Giải hệ phương trình sau
1
3
1
2 3
x
y
x
y

+ =



 − =


2) Trong mặt phẳng toạ độOxy , cho parabol ( ) 2
:
P y x
= và đường thẳng : 2 1
d y x m
= + +
(mlà tham số ).
a) Chứng minh đường thẳng d luôn cắt ( )
P tại 2 điểm phân biệt.
b) Tìm x để đường thẳng d cắt ( )
P tại 2 điểm phân biệt có hoành độ 1 2
,
x x ( với 1 2
x x
 ))
thỏa mãn 1 2 2 1 8.
x x x x
+ − =
Bài IV ( 3,0 điểm )
Cho đường tròn tâm ( )
O và điểm M nằm ngoài đường tròn ( )
O . Qua điểm M kẻ các tiếp
tuyến ,
MA MB tới đường tròn ( )
O với ,
A Blà hai tiếp điểm. Từ điểm A kẻ đường thẳng song song
với MB cắt đường tròn ( )
O tại điểm C ( C khác A), đường thẳng MC cắt đường tròn ( )
O tại điểm
D ( D khác C ). Gọi H là giao điểm của AB và .
MO
a) Chứng minh tứ giác MAOB nội tiếp.
b) Chứng minh 2 .
MA MD MC
= và .
BDM ADB
=
c) Gọi F là điểm đối xứng với D qua MO. Chứng minh , ,
C H F thẳng hàng.
Bài V ( 0,5 điểm )
Cho hai số dương a,b thoả mãn điều kiện 2 2
a b
+  . Tìm giá trị nhỏ nhất của biểu thức
1 5
2 2
P ab
ab
a b
= + +
+
…………. Hết …………
Họ và tên thí sinh:…………………………………Số báo danh…………………………….
Họ và tên của cán bộ coi thi:…………………….Chữ ký của cán bộ coi thi…………….
TRƯỜNG THPT SƠN
TÂY
HƯỚNG DẪN CHẤM MÔN TOÁN
ĐỀ THI THỬ KỲ THI TUYỂN SINH LỚP 10 THPT (
LẦN 2)
NĂM HỌC 2023 – 2024
( Hướng dẫn chấm gồ 04 trang )
I. HƯỚNG DẪN CHUNG
• Hướng dẫn chấm chỉ trình bày sơ lược các bước giải, lời giải của học sinh cần lập luận chặt
chẽ, hợp logic. Nếu học sinh trình bày cách làm khác mà đúng thì vẫn được điểm theo thang
điểm tương ứng.
• Điểm toàn bài không làm tròn.
II. ĐÁP ÁN VÀ THANG ĐIỂM
BÀI SƠ LƯỢC LỜI GIẢI ĐIỂM
Bài I
(2,0
điểm)
a) Tính giá trị của biểu thức Bkhi 4
x = 0,5
Với 4
x = thoả mãn điều kiện, ta thay 4
x = vào biểu thức B. 0,25
Tính được
4 4
7
4 4 1
B = =
+ +
0,25
b) Rút gọn biểu thức M AB
= 0,5
Với điều kiện 0, 1
x x
  ta có:
( )( )
1 1
. . .
1 1 1 1 1
1 1
x x x x
M A B
x x x x x x x
x x
 
   
= = + = +
   
− − + + − + +
+ −
 
 
0,25
( )( ) ( )( )
1
.
1
1 1 1 1
x x x
x x
x x x x
 
+
 
= +
  + +
+ − + −
 
0,25
1
.
1 1
x x x
x x x
+ +
=
− + +
0,25
Vậy
1
x
M
x
=
−
với 0, 1
x x
 
0,25
c) Khi 1
x  hãy so sánh giá trị của biểu thức M với 1 0,5
Ta có
( )
1 1 1
1
1 1 1
x
x
M
x x x
− +
= = = +
− − −
0,25
Khi 1
x  ta có
1 1
1 0 0 1 1
1 1
x
x x
−     + 
− −
Vậy khi 1
x  ta có 1.
M 
0,25
Bài
II
(2,0
điểm)
1) Giải bài toán lập phương trình hoặc hệ phương trình 1,5
Gọi x và y theo thứ tự là giá của 1 kg táo, 1 kg cam tuần trước ( tính theo nghìn
đồng),
điều kiện 0 , 130.
x y
 
0,25
Tuần trước, Khuê mua 1 kg táo và 1 kg cam hết 130 nghìn đồng nên ta có x+y=130.
(1)
0,25
Hôm nay, giá 1 kg cam đã tăng 15% nên số tiền cam phải trả là
115
15%
100
y y y
+ =
giá 1 kg táo đã tăng 20% nên số tiền táo phải trả là
120
20%
100
x x x
+ =
0,25
Khuê phải trả 154 nghìn đồng nên ta có phương trình ( )
120 115
154. 2
100 100
x y
+ =
Từ (1) và (2) ta có hệ phương trình 120 115
154
1 1
130
00 00
x y
x y
+ =



 =

+
0,25
120 115 15400 120 115 15400 120 115 15400
120 115
130 120 120 15600 5 200
40 90
40
0
154 0
x
x y x y x y
x y
x y y y
y x
y
+ = + = =
  
  
  
  
= =
 
 
 
=
 
+ = + = + =
+ =
0,25
Vậy tuần này, giá 1 kg táo là
120
108
100
x = ( nghìn đồng), giá 1 kg cam là
115
46
100
y =
(nghìn đồng)
0,25
2) Tính diện tích bề mặt bóng rổ 0,5
Bán kính quả bóng rổ 2
24,5
12,25( )
2
R cm
= = . Diện tích bề mặt bóng rổ 2
4
S R

=
0,25
( )
2 2
4.3,14. 12,25 1884,785( )
S S cm
= =  . Vậy giá trị gần đúng của diện tích bề
mặt quả bóng rổ là 2
1884,785( )
cm
0,25
Bài
III
(2,5
điểm)
1. Giải hệ PT
1
3
1
2 3
x
y
x
y

+ =



 − =


1,0
Điều kiện xác định: 0
y  , Đặt
1
u
y
= ta có hệ phương trình
3
2 3
x u
x u
+ =


− =

0,25
3 6 2 3
3 3 1
x x x
x u u x u
= = =
  
  
  
+ = = − =
  
0,25
1
1 1
y
y
=  = ( TMĐK)
0,25
Vậy hệ phương trình có nghiệm là ( ) ( )
; 2;1
x y = 0,25
2. Giải bài toán về parabol ( ) 2
:
P y x
= và đường thẳng : 2 1
d y x m
= + + 1,5
a) Chứng minh d luôn cắt ( )
P 1,0
Phương trình hoành độ giao điểm của đường thẳng d và ( )
P là: 2 1
x x m
= + + 0,25
2
2 1 0
x x m
 − − − = (1) 0,25
Biệt thức ∆’ 2
m
= + 0,25
Vì 0 2 0
m m
m R m R
   = +    nên PT (1) luôn có hai nghiệm phân biệt
Vậy đường thẳng đường thẳng d luôn cắt ( )
P tại 2 điểm phân biệt.
0,25
b) Tìm x để 1 2 2 1 8.
x x x x
+ − = 0,5
Theo giả thiết ta suy ra 1 2
,
x x là hai nghiệm phân biệt của PT (1)
Theo định lý Viet ta có
1 2
1 2
2
. 1
x x
x x m
+ =



= − −


Do 1 2
1 0 . ,0
m x x
− −  = mà 1 2 1 2
0; 0
x x x x
 =  
0,25
1 2 2 1 1 2 2 1
8. 8
x x x x x x x x
+ − =  − + + =
5
5
5
m
m
m
=

 =   = −

Vậy tập hợp các giá trị cần tìm của m là {-5; 5}
0,25
Bài
IV
(3,0
điểm)
a) Chứng minh tứ giác MAOB nội tiếp. 1,0
Vẽ hình đúng đến ý a) 0,25
Do MAlà tiếp tuyến
với đường tròn ( )
O tại
A suy ra 0
90
OAM =
Do MBlà tiếp tuyến
với đường tròn ( )
O tại
B suy ra 0
90
OBM =
0,25
Suy ra
0
180
OAM OBM
+ =
Mà OAM và OBM là
hai góc đối nên tứ giác
MAOBnội tiếp
0,25
0,25
b) Chứng minh 2
.
MA MD MC
= và .
BDM ADB
= 1,5
Chứng minh 2 .
MA MD MC
= 0,75
Xét ∆ MAC và ∆ MDA có AMC chung 0,25
ACM DAM
= ( góc nội tiếp và góc tạo bởi tiếp tuyến với dây cung cùng chắn cung
AD )
0,25
∆ MAC∽∆ MDA( g-g) 2
.
MA MC
MA MD MC
MD MA
 =  = (đpcm)
0,25
Chứng minh .
BDM ADB
= 0,75
Xét ∆ BDM và ∆ ADB ta có:
+) DBM DAB
= ( góc nội tiếp và góc tạo bởi tiếp tuyến với dây cung cùng chắn
cung BD )
+) ACD DBA
= ( góc nội tiếp và góc tạo bởi tiếp tuyến với dây cung cùng chắn
cung AD )
Mà ACM DMB
= ( hai góc so le trong do AC//MB) nên DBA DMB
=
0,25
0,25
∆ BDM ∽∆ ADB( g – g) BDM ADB
 = 0,25
c) Gọi F là điểm đối xứng với D qua MO. Chứng minh , ,
C H F thẳng hàng 0,5
Chứng minh tứ giác DHOC nội tiếp
Ta có ∆ AMOvuông tại A và AH MO
⊥
2
.
MA MH MO
= =
Theo kết quả phần (b) 2
.
MA MD MC
=
Suy ra . .
MD MC MH MO
=
Xét ∆ MCO và ∆ MHD có
CMO chung và
MC MO
MH MD
= nên ∆ MCO và ∆ MHD đồng dạng (cg-c)
MCO MHD DHOC
 =  là tứ giác nội tiếp.
0,25
D
H
O
M
A
B
C
D
H
O
M
A
B
C
F
Ta có CHO CDO
= ( góc nội tiếp cùng chắn cung OC )
Lại có CDO MCO
= ( tam giác OCDcân tại O )
Mà MCO MHD
= ( do tứ giác DCOH nội tiếp)
MHD MHF
= ( do ,
D F đối xứng nhau qua MO)
Vậy CHO MHF
=
Ta lại có , ,
M H O thẳng hàng nên 0 0
180 180
CHO MHC MHF MHC
+ =  + =
Vậy ba điểm , ,
F H C thẳng hàng.
0,25
Bài V
(0,5
điểm)
Ta có 2 2 2 2
1 1 1 4 1
( ) ( )
2 2
2
10
P ab ab
ab ab ab
a b ab a b
= + + = + + + +
+ +
Ta có BĐT ( ) ( ) ( )
2 2 2
0 4 4 4
x y x y xy xy x y xy
−   − +   + 
Với hai số dương ,
x yta có ( )
2 4 1 1 4
4
x y
x y xy
xy x y x y x y
+
+     + 
+ +
Áp dụng BĐT
1 1 4
x y x y
+ 
+
, BĐT Côsi 2
x y xy
+  cho hai số dương ,
x y và giả
thiết ta có:
( )
2 2 2 2 2
1 1 4 1 4 1 2 2 1
4
; 2 . 4;
2 2 2 4
( ) (2 2)
ab ab
ab ab ab
ab
a b a b
a b
+   +  =   =
+ +
+
Suy ra
19
4
P 
0,25
Dấu “=” xảy ra khi
4
2
2 2
a b
ab a b
ab
a b
 =


=  = =


 + =

Vậy giá tri nhỏ nhất của biểu thức
19
4
P = khi 2
a b
 = =
0,25
…Hết…
PHÒNG GIÁO DUC VÀ ĐÀO TẠO
QUẬN HÀ ĐÔNG
ĐỀ CHÍNH THỨC
ĐỀ KHẢO SÁT CHẤT LƯỢNG LỚP 9 (LẦN 1)
Năm học 2023 - 2024
Môn: TOÁN
Thời gian làm bài: 120 phút
(Không kể thời gian giao đề)
(Đề gồm có 01 trang)
Câu I. (2,0 điểm)
Cho hai biểu thức
3 1 5 2
;
4
2 2
x x x
P Q
x
x x
+ − −
= = −
−
− +
với 0, 4
x x
  .
1. Tính giá trị của biểu thức P khi 25
x =
2. Chứng minh
2
x
Q
x
=
−
3. Tìm giá trị của x để biểu thức
P
Q
đạt giá trị nhỏ nhất.
Câu II. (2,0 điểm)
1. Giải bài toàn bằng cách lập phương trình hoặc hệ phương trình
Hai bạn Linh và Chi ở hai địa điểm cách nhau 18 km đạp xe đi ngược chiều nhau để gặp nhau. Nếu hai bạn
khởi hành cùng một lúc thì sẽ gặp nhau sau 40 phút. Nhưng nếu Linh khởi hành trước 18 phút thì các bạn sẽ
gặp nhau sau 30 phút tính từ lúc Chi bắt đầu đi. Tính vận tốc của mỗi bạn?
2. Một chiếc cốc có dạng hình trụ với chiều cao 8 cm, bán kính đáy là 3 cm. Hỏi chiếc cốc này có đựng
được 200ml sữa không? (Lấy 3,14
  và bỏ qua bề dày của chiếc cốc).
Câu III. (2,5 điểm)
1. Giải hệ phương trình:
2 2 1 8
2 3 1 3
x y
x y
 − + − =


− − − = −


2. Cho Parabol (
2
P):
2
x
y = và đường thằng (d): 1
y mx m
= − + (với m là tham số).
a) Tìm m để ( d) cắt ( )
P tại 2 điểm phân biệt A và B nằm về hai phía của trục tung.
b) Gọi K là giao điểm của (d) và trục Oy. Tìm m để Δ
2
KOA
KOB
S
S
=
Câu IV. (3,0 điểm)
Từ điểm A ở ngoài đường tròn ( )
;
O R vẽ hai tiếp tuyến , ( ,
AB AC B C là các tiếp điểm) và cát tuyến ADE
thuộc nửa mặt phẳng bờ là đường thẳng OA không chứa điểm B của đường tròn ( )
O . Gọi H là giao điểm
của OA và BC .
1. Chứng minh bốn điểm A,B,O,C cùng thuộc một đường tròn.
2. Chứng minh AO BC
⊥ tại H và AH.AO AD.AE
=
3. Đường thẳng đi qua điểm D và song song với đường thẳng BE cắt ,
AB BC lần lượ tại I, K. Chứng
minh tứ giác OHDE nội tiếp và D là trung điểm của IK .
Câu V. (0,5 điểm)
Với các giá trị của m để phương trình ( )
2 2
2 1 9 0
x m x m
− + + + = có nghiệm, hãy tìm giá trị nhỏ nhất của
biểu thức
( )( )
2
2 2 4
m m m
B
m
+ − +
= .
-----------------------Hết -----------------------------
PHÒNG GIÁO DỤC VÀ ĐÀO TẠO QUẬN HÀ ĐÔNG
HƯỚNG DẪN CHẤM MÔN TOÁN KHẢO SÁT CHẤT LƯỢNG LỚP 9 (LẦN 1)
Năm học 2023 - 2024
Câu Nội dung Điểm
Câu I
(2,0
điềm)
1) Tính giá trị của P khi 25
x = với 0, 4
x x
  . 0,5
Ta có ( )
x 25 TMCK
= thay vào biểu thức P được
25 3
25 2
P
+
=
−
0,25
Tính được
28 28
5 2 3
P = =
−
0,25
2) Chứng minh
2
x
Q
x
=
−
với 0, 4
x x
  . 1,0
1 5 2 1 5 2
4 4
2 2
x x x x
x x
Q
x
x
− − − −
− = +
+
=
− −
+
( )( )
( )( )
1 2 5 2
2 2
x x x
x x
− − + −
=
+ −
0,25
( )( )
3 2 5 2
2 2
x x x
x x
− + + −
=
+ −
0,25
( )( )
2
2 2
x x
x x
+
=
+ −
0,25
( )
( )( )
2
2
2 2
x x x
x
x x
+
= =
−
+ −
0,25
3) Tìm giá trị của x để biểu thức
P
Q
đạt giá trị nhỏ nhất 0,5
* Với 0, 4
x x
  biến đổi
3 3 2 3 3
:
2 2 2
P x x x x x
x
Q x x x x x x
+ + − +
= =  = = +
− − −
*Vì
3
0, 4 0, 0
x x x
x
    
Áp dụng BĐT Cosi cho 2 số
3
;
x
x
ta có:
3 3
2 2 3
P
x x
Q x x
= +   =
Dấu "=" xảy ra khi ( )
3
3 7
x x M
x
=  =
Vậy Min 2 3 3
P
x
Q
=  =
0,25
Câu II
(2,0
điểm )
1) Gọi vận tốc của Linh là ( )
km / h
x , vận tốc của Chi là ( )
km / h
y .
Điều kiện: 0; 0
x y
 
Đổi: 40 phút
2
3
= giò; 18 phút
3
10
= giờ; 30 phút
1
2
= giờ.
0,25
Hai bạn khởi hành cùng một lúc thì sẽ gặp nhau sau 40 phút
2
3
= giờ
+ Quãng đường Linh đi được là: ( )
2
km
3
x

+ Quãng đường Chi đi được là: ( )
2
km
3
y

Linh và Chi ở hai địa điểm cách nhau 18 km đạp xe đi ngược chiều nhau
nên, ta có phương trình:
2 2
18 27
3 3
x y x y
 +  =  + =
0,25
+ Thời gian của Chi tính từ lúc đi đến lúc gặp nhau là
1
2
giờ, nên quãng
đường Chi đi được là: ( )
1
km
2
y
+ Thời gian của Linh tính từ lúc đi đến lúc gặp nhau là
3
10
giờ
1
2
+ giờ
4
5
= giờ, nên quãng đường Linh đi được là: ( )
4
km
5
x
0,25
Ta có phương trình:
4 1
18 8 5 180
5 2
x y x y
 +  =  + =
Từ (1) và (2) ta có hệ phương trình:
( )
( )
15 tm
27 5 5 135
12 tm
8 5 180 8 5 180
x
x y x y
y
x y x y
   =
+ = + =
  
 
  
=
+ = + = 
 
 

0,5
Vậy vận tốc của Linh là ( )
15 km / h , vận tốc của Chi là ( )
12 km / h . 0,25
2) Thể tích của chiếc cốc là: ( )
2 3
3 .8 226,08 cm
   0,25
Vì 3
226,08 cm 226,08ml 200ml
=  .
Nên chiếc cốc này có thể đựng được 200ml sữa.
0,25
Câu
III
(2,5
diêm )
1) ĐKXĐ: 2, 1
x y
 
Đặt ( )
2 ; 1 0, 0
x a y b a b
− = − =  
Ta có hệ phương trình
2 8 2 8 2 8 2 8 3
3 3 2 6 6 7 14 2 2
a b a b a b a b a
a b a b b b b
   
+ = + = + = + = =

  
   
    
− = − − = − = = =

 

 


Theo cách đặt có
2 3 2 9 11
1 4 5
1 2
x x x
y y
y
 − =  − = =


 
  
− = =
− = 



(tmđk)
Kết luận
0,25
0,5
0,25
2)
a) Tìm m để ( )
d cắt ( )
P tại 2 diểm phân biệt A và B nằm về hai phía so với
trục tung.
Xét phương trình hoành độ giao điểm ( )
d và ( )
P :
( )
2
2
1 2 2 2 0 *
2
x
mx m x mx m
= − +  − + − =
(d) cắt ( )
P tại hai điểm phân biệt A và B nằm ở hai phía của trục tung.
 Phương trình ( )
* có hai nghiệm 1 2
;
x x trái dấu 0
ac
 
2 2 0 1
m m
 −   
0,25
0,25
b) Gọi K là giao điểm của (d) và trục Oy. Tìm m để Δ
2
KOA
KOB
S
S
=
Với ( )
*
m 1,pt
 có hai nghiệm 1 2
x ,x . Giả sử 1 2
x 0 x
 
Theo định lý Viet, ta có 1 2
1 2
2 (1)
2 2 (2)
x x m
x x m
+ =


= −

Gọi H và I lần lượt là hình chiếu của A và B trên Oy.
1 2
AH ;BI
x x
 = =
Hai tam giác trên chung đáy OK nên tỉ số diện tích bằng tỉ số hai đường cao
Theo đề bài, ta có: 1
1 2
2
2 2 2
KOA
KOB
x
S
x x
S x
=  =  =
Mà 1 2 1 2
0 2
x x x x
   − =
Từ (1) và (3) 1 2
4 ; 2
x m x m
 = = −
Thay vào (2), ta có: ( ) 2
4 2 2 2 4 1 0
m m m m m
− = −  + − =
Giải ra
( )
( )
1
2
1 17
8
1 17
8
m tmdk
m tmdk
 − +
=



− −
 =


Vậy
1 17 1 17
;
8 8
m
 
− + − −
 
 
 
 
0,5
0,25
0,25
0,25
1) Xét ( )
O có AB,AC là hai tiếp tuyến cắt nhau ( B,C là các tiếp điểm)
90
ABO ACO
 = =
B,C
 thuộc đường tròn đường kính OA
 Bốn điểm A,B,O,C cùng thuộc đường tròn đường kính OA.
0,75
2) Xét ( )
O có AB,AC là hai tiếp tuyến cắt nhau tại A,B và C là các tiếp điểm
AB AC A
 =  thuộc trung trực của đoạn BC
Có ( )
OB OC R O
= =  thuộc trung trụ
̂ c của đoạn BC
Do đó OA là trung trực của đoạn BC OA BC
 ⊥ tại H .
+) Xét ABO có 90 ,
ABO BH OA
= ⊥
2
AH AO AB
  = (hệ thức lượng) (1)
+) Xét ( )
O có ABD AEB
= (góc nội tiếp và góc tạo bời tia tiếp tuyến và dây cùng chắn
BD )
C/ m
 dược: ABL
 ( ) 2
g g AD.AE AB
AEB
 −  =
∽ (2)
Từ (1) và ( )
2 AH AO AD AE
  = 
0,25
0,25
0,25
0,25
3) Có .
AH AE
AH AO AD AE
AD AO
=   =
C/ m
 dược: ADH
 ( )
c.g.c
AOE ADH AOE
  =
∽
C/ m
 dược tứ giác DHOE nội tiếp
+) Có tứ giác DHOE nội tiếp (cmt) AHD OED
 =
Xét ( )
O có ( )
OD OE R ODE
= =  cân tại O OED ODE
 =
Lại có tứ giác DHOE nội tiếp ( )
cmt ODE OHE
 =
AHD OHE
 =
Gọi M là giao điểm của BC và AE
C/ m được HM là phân giác của DHE
Lại có HA HM
⊥ (vì OA BC
⊥ )
HA
 là phân giác của góc ngoài tại H của HDE

MD AD DH
ME AE EH
 
 = =
 
 
Có ( )
IK//BE gt DK//BE
DK DM
BE ME
  =
Có IK ( )
//BE gt DI//BE
DI AD
BE AE
  =
DK DI
DK DI
BE BE
 =  = hay D là trung điểm của IK.
0,25
0,25
0,25
0,25
Câu V.
(0,5
điểm)
PT có nghiệm ( ) ( )
2 2
Δ 0 [ 2 1 ] 4 9 0 4
m m m
   − + − +   
Biến đổi
( )
3
2 2
2
8 8 8 15
8 16 16
2 2
8 15
( 4) 16
2 2
m m
B m m m m
m m m
m
m m
m
+  
= = + = − + + + + −
 
 
 
= − + + + −
 
 
0,25
Với 4
m  chứng minh được 2
( 4) 0
m −  ,
8 15
4 và 16 14
2 2
m
m
m
 
+  − 
 
 
Dấu "=" xãy ra khi 4
m =
Vậy GTNN của 18
B = khi 4
m =
0,25
(HS giải theo cách khác nếu đủng vẫn cho điểm tối đa)
1
SỞ GIÁO DỤC VÀ ĐÀO TẠO
HẢI DƯƠNG
ĐỀ THI GIAO LƯU AN THANH-
QUANG KHẢI
ĐỀ THI THỬ VÀO LỚP 10 THPT
NĂM HỌC 2024 - 2025
Môn thi : TOÁN
Thời gian làm bài: 120 phút, không kể thời gian giao
đề
(Đề thi gồm 02 trang- 05 câu)
Câu 1 (2,0 điểm)
1. Giải phương trình sau: 2 x - 5 - 3= 0
2. Giải hệ phương trình sau:
2 7
.
2 1
x y
x y
= +


+ =

Câu II. (2,0 điểm)
1. Rút gọn biểu thức sau:
2
a+b a+b b a b a
M= : - + -
a-b
a+ b b- ab ab+a
−
 
 
 
(Với ).
2) Xác định hàm số y = ax + b. Biết rằng đồ thị hàm số song song với đường thẳng
y =
1
2
x – 3 và cắt trục hoành tại điểm có hoành độ là -2
Câu 3 (2 điểm)
1)Ông Ba đầu tư 600 triệu đồng để kinh doanh: đầu tư trồng rau sạch với lãi suất 10%
mỗi tháng và đầu tư vào một nhà hàng với lãi suất 12% mỗi tháng . Cuối tháng ông Ba
nhận được 64 triệu đồng tiền lãi. Hỏi khoản tiền mà ông Ba đã đầu tư vào mỗi khoản
là bao nhiêu triệu đồng?
2) Cho phương trình: 2 2
2 1 4 0
x m x m m
Tìm m để phương trình có hai nghiệm phân biệt 1 2
;
x x thỏa mãn: 1 2
3 2 16
x x m
Câu 4 (3,0 điểm)
0

 a
b
2
1)Trên mặt biển, khi khoảng cách AB từ
ca nô đến chân tháp hải đăng là 250 m,
một người đứng trên tháp hải đăng đó
nhìn về phía ca nô theo phương CA tạo
với phương nằm ngang Cx một góc là
0
32
ACx ( Hình vẽ). Tính chiều cao
của tháp hải đăng( làm tròn kết quả đên
chữ số thập phân thứ nhất, biết AB//Cx
và độ cao từ tầm mắt của người đó đến
đỉnh tháp hải đăng là 3,2 m.
2)Từ một điểm M nằm ngoài đường tròn (O ; R), kẻ tiếp tuyến MA, MB với
đường tròn (O) (A và B là tiếp điểm).
a) Chứng minh tứ giác MAOB nội tiếp. Xác định tâm đường tròn ngoại tiếp tứ
giác MAOB
b.1) Gọi E là trung điểm MA, AB cắt OM tại H và K là giao điểm của BE với
đường tròn (O). Chứng minh: 2
. 2
BK BE AH
b.2) Kẻ đường kính AC của đường tròn (O) , MC cắt đường tròn (O) tại D, kẻ
DI vuông góc với AC tại I. Gọi F là giao điểm của AB và DI. Chứng minh điểm C,
E, F thẳng hàng.
Câu 5(1 điểm). Cho các số thực dương x, y, z thỏa mãn: 2 2 2
3
x y z xyz
+ + =
Tìm giá trị lớn nhất của biểu thức
2 2 2
4 4 4
x y z
P
x yz y xz z xy
= + +
+ + +
----------- Hết ----------
Họ và tên thí sinh:......................................................... Số báo danh:........................
Chữ ký giám thị 1: ..................................... Chữ ký giám thị 2: .................................
3
SỞ GIÁO DỤC VÀ ĐÀO TẠO
HẢI DƯƠNG
ĐỀ GIAO LƯU AN THANH-
QUANG KHẢI
HƯỚNG DẪN CHẤM THI THỬ VÀO LỚP 10
THPT
NĂM HỌC 2024 - 2025
Môn thi : TOÁN
Thời gian làm bài: 120 phút, không kể thời gian giao
đề
(Đáp án gồm 08 trang- 05 câu)
Câu
Hướng dẫn chấm Biểu
điểm
1
1)2 x - 5 - 3= 0 . Điều kiện 5
x  0,25
2 x - 5 = 3

2
2
2 5 3
x 4 5 9
x 0,25
29
4
x (t/m) 0,25
Vậy phương trình có nghiệm
29
4
x 0,25
2
2)
2 7
2 1
x y
x y
= +



+ =

2x y 7 4x 2y 14
x 2y 1 x 2y 1
− = − =
 

 
+ = + =
 
0,25
5x 15
x 2y 1
=

 
+ =
 0,25
x 3
y 1
=

 
= −

0,25
Vậy hệ phương trình trên có nghiệm duy nhất (x,y)=(3; -1) 0,25
2
1)
2
a+b a+b b a b a
M= : - + -
a-b
a+ b b- ab ab+a
−
 
 
 
Với
 
+ + −
= + +
 
 
−
+ − +
 
a b a b b a a b
: _
a b
a b a b a b 2
0,25
+ + + + − + −
= +
−
+
a b a b b ab ab a a b
:
a b
a b 2
0,25
0

 a
b
4
+ + −
= +
−
+
− −
= + = −
a b (a b) a b
:
b a
a b
a b a b
a b
2
2
2 2
0,25
Vậy b
a
M −
= với 0

 a
b . 0,25
2)Xét hàm số y = ax + b (*)
Đồ thị (*) cắt trục hoành khi và chỉ khi: 0
a
0.25
Do đồ thị (*) song song với đường thẳng y =
1
2
x – 3
Khi và chỉ khi
( )
1
a TM a 0
2
b 3

= 


  −

Hàm số (*) trở thành:
1
3
2
y x b b (*)
0.25
Vì đồ thị hàm số (*) cắt trục hoành tại điểm có hoành độ là (-2) nên
đồ thị hàm số (*) đi qua điểm A(-2;0). Do đó ta có:
1
. 2 0 1
2
b b ( Thỏa mãn ĐK 3
b )
0.25
Vậy hàm số cần xác định là:
1
1
2
y x 0.25
3
1)Gọi số tiền ông Ba đầu tư trồng rau sạch là x (Triệu đồng)
Số tiền ông Ba đầu tư vào nhà hàng là y ( Triệu đồng)
ĐK: 0<x,y<600 (*)
+ Vì tổng số tiền ông Ba đầu tư là 600 triệu đồng nên ta có phương
trình: 600 1
x y
0.25
Sau một tháng :
Số tiền lãi ông Ba được hưởng khi đầu tư vào trồng rau sạch là :
10%.x ( Triệu đồng)
Số tiền lãi ông Ba được hưởng khi đầu tư vào nhà hàng là :
12%.y ( Triệu đồng)
0.25
5
Vì cuối tháng ông Ba nhận được 64 triệu đồng tiền lãi nên ta có
phương trình:
10%. 12%. 64
x y 5 6 3200
x y (2)
Từ (1) và (2) ta có hệ phương trình :
150 400
....
5 6 3200 200
x y x
x y y
( Thỏa mãn ĐK(*))
0.25
Vậy số tiền ông Ba đầu tư vào vào trồng rau sạch là : 400 (Triệu
đồng)
Số tiền lãi ông Ba được hưởng khi đầu tư vào nhà hàng là : 200
(Triệu đồng)
0.25
2) Xét phương trình: 2 2
2 1 4 0
x m x m m (*)
Ta có:
2
' 2
1 1. 4 3 3
m m m m
Phương trình (*) có hai nghiệm phân biệt khi và chỉ khi:
'
0 3 3 0 1
m m
0.25
Với m>1, áp dụng hệ thức Vi-ét ta có :
1 2
2
1 2
2 2 1
4 2
x x m
x x m m
0.25
Ta có :
2
2 1 15
4 0
2 4
m m m Với mọi giá trị của m
1 2 0
x x Với mọi giá trị của m
Phương trình (*) có hai nghiệm cùng dấu
Với 1 2 2 0
m m
1 2 0
x x
Phương trình (*) có hai nghiệm dương phân biệt khi m>1
Vì 1 2
0; 0
x x nên 1 1 2 2
;
x x x x
Theo bài ra : 1 2
3 2 16
x x m 1 2
3 2 16
x x m (3)
0.25
Từ (1) và (3) ta có hệ phương trình :
6
1 2 1
1 2 2
2 2 2
...
3 2 16 4
x x m x m
x x m x m
Thay 1
2
2
4
x m
x m
vào (2) ta được :
2
2 4 4
m m m m
………….
4
m ( Thỏa mãn ĐK : m>1)
Vậy m=4 là giá trị cần tìm 0.25
4
1)Hình vẽ :
O.25
3,2 m
x
250 m
320
B
A
C
7
(HS cần vẽ hình minh họa dưới)
Vì Cx//AB nên : BAC ACx mà 0
32
ACx nên : 0
32
BAC
Vì tam giác ABC vuông tại B nên áp dụng hệ thức giữa cạnh và góc
trong tam giác vuông ta có :
.tan
BC AB BAC
0
250.tan32
BC =156,217338 156,2
0.5
Chiều cao của tháp hải đăng là : 156,2+3,2=159,4 (m)
Vậy chiều cao tháp hải đăng xấp xỉ khoảng 159,4 m
0.25
2)
Hình vẽ câu a
0.25
a) Xét tứ giác MAOB có :
0
90
MAO ( Do MA là tiếp tuyến của đường tròn (O))
0
90
MBO ( Do MB là tiếp tuyến của đường tròn (O))
0 0 0
90 90 180
MAO MBO
Tứ giác MAOB nội tiếp đường tròn đường kính OM
( Tổng hai góc đối diện bằng 1800
)
Vậy tứ giác MAOB nội tiếp đường tròn đường kính OM.
Tâm đường tròn ngoại tiếp tứ giác MAOB là trung điểm của đoạn
thẳng OM
0.25
0.25
0.25
B
A
O M
8
b.1) Hình vẽ câu b.1
+ Chứng minh được OM là đường trung trực của AB
H là trung điểm của AB
Chứng minh được EH là đường trung bình của tam giác ABM
HE//BM
2 1
E B ( Cặp góc so le trong)
Mà : 2 1
A B ( Góc nội tiếp và góc tạo bởi tia tiếp tuyến và dây
cung cùng chắn BK của đường tròn (O))
2 2
E A
Tứ giác AEKH nội tiếp (hai đỉnh liên tiếp E và A cùng nhìn cạnh
KH dưới một góc bằng nhau)
1
K HAE
Hay 1
K BAE
0.25
+ Chứng minh được : BAE đồng dạng với BKH
BA BE
BK BH
. .
BA BH BK BE
Mà H là trung điểm của AB (cm trên)
1
2
2
1
H
K
E
B
A
O M
9
1
2
BH AH AB
Do đó : 2
. 2. . 2
BK BE AH AH AH
0.25
b.2) Hình vẽ
Kẻ tiếp tuyến tại C của đường tròn (O) cắt DE tại N,
Gọi giao điểm của NO với CD là Q.
Đường thẳng CN cắt đường thẳng AD tại P
+ Chứng minh được DE là tiếp tuyến của đường tròn (O)
Chỉ ra được ON là đường trung trực của CD
Chứng minh được ba điểm H, B, N thẳng hàng.
Chứng minh được N là trung điểm của CP ( Dựa vào định lí đường
trung bình trong tam giác ACP)
0.25
+ Chứng minh được F là trung điểm của ID
Gọi giao điểm của CF với MA là E’
. Chứng minh được E’
là trung
điểm của MA
Vậy điểm E trùng với điểm E’
, nghĩa là ba điểm C, F, E thẳng hàng.
0.25
5
2 2 2
3 3
x y z
x y z xyz
yz xz xy
+ + =  + + =
P
Q
N
F
I
D
C
H
E
B
A
O M
10
Áp dụng bất đẳng thức Cô-si cho hai số dương ;
x y
yz xz
ta có:
2
2 .
x y x y
yz xz yz x z
+  =
0.25
Tương tự ta cũng có:
2 2
;
y z z x
xz xy x xy yz y
+  + 
2 2 2
x y y z z x
yz xz xz xy xy yz z x y
     
 + + + + +  + +
     
     
1 1 1 1 1 1
3
x y z
yz zx xy x y z x y z
 + +  + +  + + 
0.25
Lại có:
2
4 4 2
4
1 1 1 1 1 1 1
2 2 .2. . ( )
4 4
2
x
x yz x yz x yz
x yz y z
yz y z
+  =   =  +
+
Tương tự
2 2
4 4
1 1 1 1 1 1
( ); ( )
4 4
y z
y xz x z z xy x y
 +  +
+ +
Suy ra
0.25
2 2 2
4 4 4
1 2 2 2 1 1 1 1 3
( ) ( )
4 2 2
3
2
x y z
P
x yz y xz z xy x y z x y z
P
= + +  + + = + + 
+ + +
= 
Vậy giá trị lớn nhất của P =
3
2
khi x = y = z = 1
0.25
Lưu ý: Học sinh làm theo cách khác đúng vẫn cho điểm tối đa.
SỞ GIÁO DỤC VÀ ĐÀO TẠO
TỈNH PHÚ YÊN
ĐỀ CHÍNH THỨC
KỲ THI CHỌN HỌC SINH GIỎI THCS CẤP TỈNH,
NĂM HỌC 2023 - 2024
Môn thi: TOÁN
Ngày thi: 06/3/2024
Thời gian: 150 phút (không kể thời gian giao đề)
Câu 1.(3,50 điểm) Cho các biểu thức:
1 1 1 1
;
1 2 2 3 3 4 8 9
A = + + ++
+ + + +
1 1 1 1
;
1 2 3 8
B = + + ++
1 2 3 8.
C = + + ++
a) Tính giá trị của biểu thức A.
b) So sánh giá trị các biểu thức A, B và C .
Câu 2. (3,50 điểm) Giài hệ phương trình:
( )
( )
2 2
2 2
4 5
4 3
x x y y
y y x x
 + =


− =


Câu 3. ( 4,00 điểm) Cho hình vuông ,
ABCD I là trung điểm của cạnh AB. Dựng đường tròn
tâm I , đường kính AB. Tiếp tuyến DE với đường tròn (I cắt cạnh BC tại F ( E là tiếp
điểm).
a) Biết 6,25 cm
EF = , tính cạnh của hình vuông.
b) Trên nửa đường tròn đường kỉnh AB (phần không cùng phía với hinh vuông ABCD )
lấy các điềm ,
M N sao cho 15 cm
BM MN
= = ( M nằm giữa B và N ). Tính chu vi tứ giác
BMNA.
Câu 4. (3,00 điểm)
a) Đặt ; ;
a b b c c a
x y z
a b b c c a
− − −
= = =
+ + +
, với , ,
a b c là các số tự nhiên dương.
Chứng minh rằng: ( )( )( ) ( )( )( )
1 1 1 1 1 1
x y z x y z
− − − = + + + .
b) Biết
( )( )( )
( )( )( )
12
a b b c c a
a b b c c a
+ + +
=
− − −
; tinh giá trị biểu thức
a b c
S
a b b c c a
= + +
+ + +
.
Câu 5. (3,00 điểm)
Cho ,
x y là hai số thực dương thỏa: 2 2
1 5 4
4
y x
= − .
Tìm giá trị nhò nhất của biểu thức 4 2024
P x y
= + + .
Câu 6. (3,00 điểm) Cho tam giác ABC vuông tại A. D là điểm di động trên cạnh AC . Đường
thẳng qua A và vuông góc với BD cắt đường thằng qua C và vuông góc với AC tại E .
Chứng minh rắng đường tròn đường kinh DE đi qua điểm cố định thứ hai (khác điểm C ).
------Hết------
Thí sinh không sử dụng tài liệu. Giám thị không giải thích gì thêm.
Họ và tên thi sinh:……………………....;Số báo danh:…………………….
Chữ ki giám thị 1:…………………………………. Chữ kí giám thị 2:……………………..
UBND TP. TAM KỲ
PHÒNG GIÁO DỤC VÀ ĐÀO TẠO
Đề chính thức
KỲ KHẢO SÁT HỌC SINH GIỎI LỚP 9 LẦN 3
NĂM HỌC 2023 - 2024
MÔN TOÁN
Ngày 06 tháng 03 năm 2024
Thời gian: 150 phút (không kể thời gian phát dề)
Câu 1. (4.0 điểm)
a) Cho ( )
2
2 1 2 1
f n
n n
=
+ + −
với n là số nguyên dương. Tính giá trị của biểu thức
( ) ( ) ( ) ( )
1 2 3 60 .
P f f f f
= + + ++
b) Tìm tất cả các giá trị của tham số m để phương trình ( )
2 2
2 3 1 4 0
x m x m m
− − + − − = có hai
nghiệm phân biệt 1 2
,
x x thỏa mãn 1 2 1 2 1 2 1 2
2024
x x x x x x x x
+ + = − + − .
Câu 2. (4.0 điểm)
a) Giải phương trình: ( ) 4 3 2
4 1 32 80 50 3
x x x x x
− − = − + − + .
b) Giải hệ phương trình:
3 2
2 1 3
4 1 9 8 52 4
x y
x x y x y xy
 − + =


− + − − = − −


Câu 3. (4.0 điểm). Cho tam giác ABC , kẻ các đường phân giác trong , ,
AD BE CF của tam giác
ABC . Chứng minh rằng
a) AD AB AC BD DC
=  − 
b)
1 1 1 1 1 1
AB AC BC AD BE CF
+ +  + +
Câu 4. (4.0 điểm). Cho đường tròn ( )
;
O R và điểm M cố định ở bên ngoài đường tròn. Từ M kẻ
các tiếp tuyến ,
MA MB và cát tuyến MCD dến đường tròn ( )
;
O R , với ,
A B là các tiếp điểm. ,
C D
thuộc đường tròn ( )
O sao cho , 2
MC MD CD R
  . Gọi E là trung điểm của CD.
a) Chứng minh bốn điểm , , ,
A E O B cùng nằm trên một đường tròn.
b) Gọi F là giao điểm của AB và OE . Chứng minh FC là tiếp tuyến của đường tròn
( )
;
O R .
c) Gọi T là điểm thay đổi trên cung nhỏ AB của đường tròn ( )
O . Tiếp tuyến tại T của
đường tròn ( )
O cắt ,
MA MB lần lượt tại các điểm ,
I K . Chứng minh rằng chu vi tam giác
MIK không đổi. Xác định vị trí của điểm T trên cung nhỏ AB sao cho tam giác MIK có
diện tích lớn nhất.
Câu 5. (5.0 điểm)
a) Tìm tất cả các bộ ba số nguyên tố ( )
, ,
p q r sao cho 200
p q r pqr
+ + = − .
b) Cho các số thực , , , 1
a b c d  − . Tìm tất cả các giá trị thực của k sao cho
( )
3 3 3 3
1
Hêt
a b c d k a b c d
+ + + +  + + +
=== ===
Lưu ý: Thí sinh không được phép sử dụng tài liệu. Giám thị không được gợi ý gì thêm.
Họ và tên thí sinh............................................... Số báo danh……………………………….
Hướng dẫn chấm môn Toán - Thi khảo sát HSG lớp 9 lần 3 năm học 2023-2024
UBND TP. TAM KỲ
PHÒNG GIÁO DỤC VÀ ĐÀO TẠO
KHẢO SÁT HỌC SINH GIỎI LỚP 9-LẦN 3
Năm hoc: 2023 - 2024
Ngày 06 tháng 03 năm 2024
Môn: TOÁN
HƯỚNG DÃN CHÁM
Câu Nội dung Điểm
1
(4.0
điểm )
a) (2.0 điểm)
Ta có
( )
( )
( )( )
2 2 1 2 1
2 1 2 1 2 1 2 1
n n
f n
n n n n
+ − −
=
+ + − + − −
(do 2 1 2 1 0)
n n
+ − − 
0.5
( )
( ) ( )
2 2 1 2 1
2 1 2 1
2 1 2 1
n n
n n
n n
+ − −
= = + − −
+ − −
0.5
Khi đó
( ) ( ) ( ) ( )
3 1 5 3 7 5 121 119
P = − + − + − ++ − 0.5
121 1 11 1 10
= − = − = 0.5
b) (2.0 điểm)
Phương trình ( ) ( )
2 2
2 3 1 4 0 1
x m x m m
− − + − − = có hai nghiệm
phân biệt 2
Δ 0 8 5 5 0
m m
  − +

 
0.25
2
5 135
8 0;
16 32
m m
 
 − +   
 
 
0.25
Theo định lí Vi-ét, ta có:
( )
1 2
2
1 2
2 3 1
4
x x m
x x m m
+ = −


= − −

1 2
x x xác định ( )
2
1 2
0 4 0 *
x x m m
   − − 
0.25
Đặt 1 2 1 2 1 2 1 2
;
u x x x x v x x x x
= + + = + −
Ta có ( )
2 2
2 2 2
1 2 1 2 1 1 2
3
. 0; ,
2 4
x x
u v x x x x x x x
 
= + − = + +  
 
 
Suy ra u và v luôn cùng dấu. Do đó u v u v
+ = +
0.25
Hướng dẫn chấm môn Toán - Thi khảo sát HSG lớp 9 lần 3 năm học 2023-2024
Ta có
1 2 1 2 1 2 1 2
2024
x x x x x x x x
+ + + + − =
1 2 1 2 1 2 1 2
2024
x x x x x x x x
 + + + + − =
1 2
1012
x x
 + =
0.5
2 3 1 1012
m
 − =
169
3 1 506
3 1 506 505
3 1 506
3
m
m
m
m m
=
 
− =
 
 − =  
 
− = − = −
 

(thỏa điều kiện ( ))
*
.
0.5
2
(4.0
điểm)
a) (2.0 điểm )
Điều kiện: 1
x 
0.25
Ta có ( )
2
1
4 1 4 1 3 3
2
x x x
 
− − = − − + 
 
 
0.5
Dấu đẳng thức xảy ra khi và chỉ khi
1 5
1
2 4
x x
− =  = . 0.25
Lại có
2
4 3 2 2 5
32 80 50 3 3 32 3
4
x x x x x
 
− + − + = − − 
 
 
0.5
Dấu đẳng thức xảy ra khi và chỉ khi
0
5
0 5
4
4
x
x x
x
=

  
− = 
   =
 

0.25
Qua hai đánh giá trên ta thấy, phương trình đã cho có nghiệm duy nhất
5
4
x = .
0.25
b) (2.0 điểm )
Điều kiện: 1
y  −
0.25
Ta có:
( ) 2 2
3 3
2 1 3 2 1 3
4 1 ( 3) 4 6 5
x x
x y y x
y x y x x
 
 
− + =  + = −  
 
+ = − = − +


0.75
Thay 2
4 6 5
y x x
= − + vào phương trình thứ hai của hệ phương trình ta
được: ( ) ( ) ( )
3 2 2 2
2 3 9 2 6 5 52 6 5
x x x x x x x x x
− − − − − + = − − − +
0.5
Hướng dẫn chấm môn Toán - Thi khảo sát HSG lớp 9 lần 3 năm học 2023-2024
2
3
4 21 0
7
x
x x
x
= −

 − − =   =

. So với điều kiện 3
x  , chọn nghiệm 7
x =
Thay vào phương trình 2
4 6 5
y x x
= − + ta được 3
y = .
Vậy hệ phương trình đã cho có nghiệm duy nhất là: ( )
7;3 .
0.5
3
(3.0
điểm)
a) (1.5 điểm)
Lấy K thuộc tia đối của tia DA sao cho AKB ACB
= .
Vì ( )
g g
AD AC
ACD AKB AB AC AD AK
AB AK
   =   =  (1)
0.5
Vì ( )
g.g .
DC AC AD
DAC DBK BD DC DK AD
DK BK BD
  = =   = (2) 0.5
Trừ (1) và (2) vế theo vế ta được
( ) 2
AB AC DC BD AD AK KD AD AD AD
 −  =  − =  =
hay AD AB AC BD DC
=  −  .
0.5
Hướng dẫn chấm môn Toán - Thi khảo sát HSG lớp 9 lần 3 năm học 2023-2024
b) (1.5 điểm)
Qua B kẻ đường thẳng song song với AD, cắt đường thẳng AC tại
M
ABM
 cân tại A AM AB
 = .
0.25
Theo bất đẳng thức tam giác: 2
MB AM AB MB AB
 +   0.25
Do / /
AD BM nên
AD CA AC AC
BM CM AC AM AC AB
= = =
+ +
(do CM=AC+AM; AM=AB)
AD AC
BM AC AB
 =
+
2 2
AC AC AB AB AC
AD BM
AC AB AC AB AC AB
 
 =   =
+ + +
1 1 1 1 1 1
2 2 2 2
AC AB
AD AB AC AC AB AC AB
+  
  = + =  +
 
  
(3)
0.5
Tương tự,
1 1 1 1
2
BE AB BC
 
  +
 
 
0.25
1 1 1 1
2
CF AC BC
 
  +
 
 
Cộng vế theo vế các bất đẳng thức (3), (4) và (5) ta được:
1 1 1 1 1 1
AD BE CF AB BC AC
+ +  + +
0.25
4
(4.0
điểm)
Hướng dẫn chấm môn Toán - Thi khảo sát HSG lớp 9 lần 3 năm học 2023-2024
a) (1.0 điểm)
Xét tứ giác AOBM có:
90
MAO = (vì MA là tiếp tuyến của đường tròn ( )
O )
90
MBO = (vì MB là tiếp tuyến của đường tròn ( ))
O
90 90 180
MAO MBO
 + = + =
Do đó tứ giác AOBM nôi tiếp đường tròn đường kính MO (1)
0.5
Vì E là trung điểm của đoạn thẳng CD nên OE CD
⊥
Xét MOE vuông tại E nên nội tiếp đường tròn đường kính ( )
2
MO
Từ (1) và (2) suy ra 5 điểm , , , ,
A E O B M cùng nằm trên đường tròn
đường kính MO.
Vậy bốn điểm , , ,
A E O B cùng nằm trên một đường tròn.
0.5
b) (1.0 điểm)
( )
g.g
OE OM
OEM OHF
OH OF
  =
2 2
OE OF OM OH OA OC
  =  = =
OE OC
OC OF
 =
0.5
Xét OEC và OCF có
OE OC
OC OF
= và EOC COF
= .
Do đó OEC OCF
 (c.g.c). Suy ra 90
OEC OCF
= = .
Mà ( )
D O
 nên FC là tiếp tuyến của đường tròn ( )
;
O R .
0.5
c) (2.0 điểm)
Do IK là tiếp tuyến của đường tròn ( )
O tại T nên IT IA
= và KT KB
=
IK IT KT IA KB
 = + = +
Kí hiệu MIK
C là chu vi ΔMIK
Ta có: 2
MIK
C MI MK IK MI MK IA KB MA MB MA
= + + = + + + = + =
mà điểm A và M cố định nên MA không đổi. Do đó MIK
C không đổi
0.5
Qua O kẻ đường thẳng song song AB cắt MA tại 1
A , cắt MB tại 1
B 
Ta có:
1 1 1 1
1 1 1
2 2 2
KOI KOT TOI BOT TOA AOB AOM MAB MB A
= + = + = = = =
Từ đó suy ra 1 1
Δ Δ
IOA IOK OB K
  (g.g)
1 1
1 1
IA OB
OA B K
 = hay
2
1 1
1 1 1 1
4
AB
IA B K OA OB
 =  = (không đổi)
0.5
Hướng dẫn chấm môn Toán - Thi khảo sát HSG lớp 9 lần 3 năm học 2023-2024
Ta có: ( )
1 1 1 1 1 1 1 1
1
2
MIK MA B KOI IOA KOB
S S S S S MO AB R IK IA KB
= − − − =  − + +
 
 
Mà 2
IK KM MA
+ = (chứng minh trên) nên
( ) ( )
1 1 1 1
2 2
IK MA MI MK MA MA IA MB KB
= − − = − − − −
1 1 1
2 2
MA MA IA KB
= − + +
( )
1 1 1 1 1
1
2 2 2 2
2
IMK
S MO AB R MA MA IA KB
 =  − − + +
 
 
( )
1 1 1 1 1
1
2
MO AB R AA R IA KB
=  +  − +
0.5
Áp dụng bất đẳng thức AM GM
− , ta có:
2
1 1
1 1 1 1 1 1
2 2
4
AB
IA KB IA KB AB
+   = =
1 1 1 1 1
1
2
IMK
S MO AB R AA RAB
   +  − .
Dấu "=" xảy ra 1 1 n
IA KB T AB
 =   sao cho / /
IK AB.
0.5
5
(5.0
diêm
)
a) (2.5 điểm)
Không mất tính tổng quát, giả sử p q r
  . Phương trình đã cho tương
đương với ( )( ) ( )( )
1 1 1 1 202
qr p r q
− − + − − = .
Nếu p lẻ thì ,
q r cũng lẻ. Khi đó vế trái của phương trình trên chia hết
cho 4 nhưng vế phải thì không (vô lí). Do đó 2
p = .
0.5
Thay 2
p = vào phương trình trên ta được
( )( ) 4
2 202 4 2 2 1 405 2 1 2 1 5.3
qr q r qr q r q r
− − =  − − + =  − − =
Do 3 2 1 2 1
q r
 −  − nên ( )( )
2
9 (2 1) 2 1 2 1 405
q q r
 −  − − =
Suy ra 3 2 1 20
q
 −  .
0.5
Mà 2 1
q − lẻ nên  
2 1 3;5;9;15
q −  . 0.25
Nếu 2 1 3 2
q q
− =  = thì 68
r = (không phải là số nguyên tố nên loại). 0.25
Nếu 2 1 5 3
q q
− =  = thì 41
r = (là số nguyên tố nên thỏa). 0.25
Nếu 2 1 9 5
q r
− =  = thì 23
r = (là số nguyên tố nên thỏa). 0.25
Nếu 2 1 15 8
q q
− =  = thì loại vì không phải là số nguyên tố. 0.25
Vậy tất cả các bộ ba số nguyên tố cần tìm là ( ) ( )
2,5,23 , 2,3,41 cùng các
hoán vị của nó.
Lưu ý:
Nếu thiếu cụm từ “các hoán vị của nó” thì trừ 0.25 điểm.
0.25
Hướng dẫn chấm môn Toán - Thi khảo sát HSG lớp 9 lần 3 năm học 2023-2024
b) (2.5 điểm)
Nếu 1
a b c d
= = = = − thì ( )
3 . 4
k
−  − . Do đó
3
4
k  .
0.5
Nếu
1
2
a b c d
= = = = thì
1 1
4 1 4
8 2
k
 
 +   
 
 
. Suy
3
4
k  . Vì vậy
3
4
k = . 0.5
Với
3
4
k = , ta sẽ chứng minh bất đẳng thức
( )
3 3 3 3 3
1 , , , , 1.
4
a b c d a b c d a b c d
+ + + +  + + +   −
Đầu tiên, ta chứng minh 3
4 1 3 , 1
x x x
+    − .
Thật vậy
( )
3 2
4 1 3 1 (2 1) 0
x x x x
+   + −  luôn đúng với mọi 1
x  − .
0.5
Áp dụng bất đẳng thức trên ta có
3
4 1 3
a a
+  ,
3
4 1 3
b b
+  ,
3
4 1 3
c c
+  ,
3
4 1 3
d d
+  .
0.5
Cộng vế theo vế bốn bất đẳng thức trên ta được
( )
3 3 3 3 3
1 , , , , 1.
4
a b c d a b c d a b c d
+ + + +  + + +   −
Vậy
3
4
k = là giá trị cần tìm.
0.25
===Hết===
Lưu ý :
• Nhũng câu, nhũng ý nào thí sinh làm khác với đáp án nhung lập luận họpp lý và đáp
số đúng vẫn đạt điểm tối đa ở câu đó, ý đó.
• Nhũng kiến thức mới thí sinh đưa vào bài làm nếu đúng vẫn được chấp nhận bởi vì
đây là đối tượng học sinh giỏi có năng khiếu Toán.
Khuyến khích nhũng cách giải sáng tạo, độc đáo và có cộng điểm thưởng cho nhüng cách
giải này.
45o
4m
20 m
A B
H
C
UBND THÀNH PHỐ HẢI DƯƠNG
PHÒNG GIÁO DỤC VÀ ĐÀO TẠO
ĐỀ KHẢO SÁT HỌC SINH LỚP 9 - LẦN 2
NĂM HỌC 2023 - 2024
MÔN TOÁN
Thời gian làm bài: 120 phút
(Đề gồm 06 câu, 01 trang)
Câu 1. (2,0 điểm)
a) Giải phương trình
2 1
5.
3
x −
=
b) Giải hệ phương trình
( )
2 1 3
3 1 0
x y
x y
 − + =

− + =

.
Câu 2. (2,0 điểm)
a) Rút gọn biểu thức ( )
2 5 1
: 4
3 6 2
x
A x
x x x x
 
+
= − + −
 
 
+ + − −
 
với 0, 4, 16
x x x
   .
b) Cho hai đường thẳng ( ) ( )
2
: 1 2 3
d y m x m
= + + − và ( )
' : 10 6
d y x m
= + − , với m là tham số. Tìm m
để hai đường thẳng đã cho song song với nhau.
Câu 3. (2,0 điểm)
a) Công ty HD xây dựng kế hoạch cho 2 phân xưởng sản xuất với tổng sản phẩm làm được là là 520 sản
phẩm. Tuy nhiên, các phân xưởng đều rất trách nhiệm và áp dụng kĩ thuật tốt nên đã nâng cao hiệu quả so với
kế hoạch là 20% và tổng số sản phẩm sản xuất được của 2 phân xưởng là 596 sản phẩm. Hỏi theo kế hoạch ban
đầu, mỗi phân xưởng làm bao nhiêu sản phẩm?
b) Cho phương trình ( )
2 2
2 1 2 0
x m x m
− + + + = , với m là tham số. Tìm m để phương trình đã cho có
hai nghiệm phân biệt 1 2
,
x x thỏa mãn: ( )
2 2 2
1 2 1 2
3 2 1
x x m x x
− = + − .
Câu 4 (1,0 điểm) Một người đứng ở vị trí A trên nóc một ngôi nhà cao
4 m đang quan sát một cây cao, cách ngôi nhà 20 m và đo được
45
BAC = (tham khảo hình vẽ). Tính chiều cao của cây đó (theo đơn vị
mét, làm tròn kết quả đến hàng phần mười), biết rằng nếu góc  nhọn
và thỏa mãn
1
tan
5
 = thì ta chọn 11
 = .
Câu 5 (2,0 điểm) Cho tam giác nhọn ABC có AB AC
 , các đường cao BD và CE cắt nhau tại H .
a) Chứng minh rằng tứ giác ADHE nội tiếp đường tròn.
b) Gọi M là trung điểm của BC , đường thẳng DE cắt BC tại N , AH cắt BC tại .
K Chứng minh
rằng DEK DMC
= và NH AM
⊥ .
Câu 6 (1,0 điểm) Cho ,
x y là các số thực không âm, thỏa mãn 2
x  và ( )
2 2
xy y
= + . Tìm giá trị lớn nhất của
biểu thức
2
2
2
2 1
1 1
y y
x x
B
x y x y
+
−
= + +
− + +
.
ĐỀ CHÍNH THỨC
UBND THÀNH PHỐ HẢI DƯƠNG
PHÒNG GIÁO DỤC VÀ ĐÀO TẠO
ĐỀ KHẢO SÁT HỌC SINH LỚP 9 - LẦN 2
NĂM HỌC 2023 - 2024
MÔN TOÁN
Thời gian làm bài: 120 phút
(Đề gồm 06 câu, 01 trang)
Câu Đáp án Điểm
1a
a) Giải phương trình
2 1
5.
3
x −
=
+ Ta có
2 1
5 2 1 15
3
x
x
−
=  − = 0,5
2 16 8
x x
 =  =
+ Vậy phương trình có nghiệm là 8
x =
0,5
1b
b) Giải hệ phương trình
( )
2 1 3
3 1 0
x y
x y
 − + =

− + =

.
+ Ta có:
( ) 2 3
2 1 3
3 1
3 1 0
x y
x y
x y
x y
+ =
 − + = 

 
− = −
− + = 

0,25
6 3 15 7 14 2
3 1 3 1 3 1
x y x x
x y x y x y
+ = = =
  
  
  
− = − − = − − = −
  
0,5
2 2
3 3 1
x x
y y
= =
 
 
 
= =
 
+ Vậy hệ phương trình có nghiệm ( ) ( )
, 2; 1
x y = .
0,25
2a
a) Rút gọn biểu thức ( )
2 5 1
: 4
3 6 2
x
A x
x x x x
 
+
= − + −
 
 
+ + − −
 
với
0, 4, 16
x x x
   .
+ Ta có: ( )
2 5 1
: 4
3 6 2
x
A x
x x x x
 
+
= − + −
 
 
+ + − −
 
( )( )
( )( ) ( )( ) ( )( )
( )( )
2 2 5 3 1
.
4
3 2 3 2 3 2
12 1
.
4
3 2
x x x
x
x x x x x x
x x
x
x x
 
+ − +
 
= − −
  −
+ − + − + −
 
− −
=
−
+ −
0,5
( )( )
( )( )
3 4 1 1
.
4 2
3 2
x x
x x
x x
+ −
= =
− −
+ −
+ Vậy
1
2
A
x
=
−
0,5
HƯỚNG DẪN CHẤM
2b
b) Cho hai đường thẳng ( ) ( )
2
: 1 2 3
d y m x m
= + + − và ( )
' : 10 6
d y x m
= + − , với
m là tham số. Tìm m để hai đường thẳng đã cho song song với nhau.
+ Điều kiện để hai đường thẳng song song là
2
1 10
2 3 6
m
m m
 + =

−  −

0,5
2
9
3
3 9
m
m
m
 =
  = −



Vậy 3
m = −
0,5
Câu 3a
a) Công ty HD xây dựng kế hoạch cho 2 phân xưởng sản xuất với tổng sản phẩm
làm được là là 520 sản phẩm. Tuy nhiên, các phân xưởng đều rất trách nhiệm và
áp dụng kĩ thuật tốt nên đã nâng cao hiệu quả so với kế hoạch là 20% và tổng số
sản phẩm sản xuất được của 2 phân xưởng là 596 sản phẩm. Hỏi theo kế hoạch
ban đầu, mỗi phân xưởng làm bao nhiêu sản phẩm?
+ Gọi số sản phẩm theo kế hoạch mà phân xưởng thứ nhất và phân xưởng thứ hai
làm được lần lượt là ,
x y (sản phẩm) ( )
,
x y 
+ Tổng sản phẩm làm được là 520 sản phẩm nên 520
x y
+ =
0,25
+ Thực tế, số sản phẩm phân xưởng thứ nhất làm được là 10% 1,1
x x x
+ = , số sản
phẩm phân xưởng thứ hai làm được là 20% 1,2
y y y
+ =
0,25
+ Số sản phẩm mà 2 phân xưởng làm được là 596 nên 1,1 1,2 596
x y
+ =
Ta có hệ phương trình:
520
1,1 1,2 596
x y
x y
+ =


+ =

0,25
280
( )
240
x
tmdk
y
=

 
=

Vậy theo kế hoạch, số sản phẩm 2 phân xưởng làm là 280 và 240 sản phẩm.
0,25
3b
b) Cho phương trình ( )
2 2
2 1 2 0
x m x m
− + + + = , với m là tham số. Tìm m để
phương trình đã cho có hai nghiệm phân biệt 1 2
,
x x thỏa mãn:
( )
2 2 2
1 2 1 2
3 2 1
x x m x x
− = + − .
+ Điều kiện để phương trình có hai nghiệm phân biệt là
( ) ( )
2 2
' 0 1 2 0
m m
   + − + 
0,25
1
2 1 0
2
m m
 −   
+ Vậy
1
2
m  th ì phương trình có hai nghiệm phân biệt 1 2
,
x x
0,25
+ Theo Vi – et:
( ) ( )
( )
1 2
2
1 2
2 1 1
1 2
x x m
x x m
+ = +



= +


T ừ gi ả thi ết ( ) ( )
2 2 2 2 2 2
1 2 1 2 1 1 2 2
3 2 1 2 3 2 1
x x m x x x x x x m
− = + −  − − = −
( )( ) ( )( ) ( )( ) ( )( )
1 2 1 2 1 2
3 2 1 1 2 1 3 2 1 1
x x x x m m m x x m m
 + − = + −  + − = + −
Vì
1
2
m  nên ( )
2 1 0
m +  , do đó ( )
1 2
3 1 3
x x m
− = −
0,25
45o
4m
20 m
K
A B
H
C
+ Từ (1), (3) ta được
1
2
7 5
4
3
4
m
x
m
x
+

=



+
 =


Thay vào (2):
( )( ) 2 2
1
7 5 3
2 9 26 17 0 17
16
9
m
m m
m m m
m
=

+ + 
= +  − + = 
 =

Kết hợp điều kiện
1
2
m  , ta có 1
m = thỏa mãn yêu cầu bài toán. Vậy 1
m = .
0,25
4
Một người đứng ở vị trí A trên nóc một ngôi nhà cao 4 m đang quan sát một cây
cao, cách ngôi nhà 20 m và đo được 45
BAC =  (tham khảo hình vẽ). Tính chiều
cao của cây đó (theo đơn vị mét, làm tròn kết quả đến hàng phần mười), biết
rằng nếu góc  nhọn và thỏa mãn
1
tan
5
 = thì ta chọn 11
 = .
+ Ta có
1
tan 11
5
AH
ABH ABH
BH
= =  =  0,25
+ Kẻ AK vuông góc với BC tại K . Vì AK BH
∥ nên 11
KAB ABH
= = .
Do đó 45 11 34
CAK = −  = 
0,25
+ Xét tam giác ACK có ( )
tan .tan34 13,5
CK
CAK CK AK m
AK
=  =   0,25
+ Chiều cao của cây là: ( )
4 13,5 17,5
BC BK KC m
= +  + = 0,25
5a
Cho tam giác nhọn ABC có AB AC
 , các đường cao BD và CE cắt nhau tại
H .
a) Chứng minh rằng tứ giác ADHE nội tiếp đường tròn.
H
E
D
B C
A
+ Do ,
BD CE là các đường cao cắt nhau tại H nên 90
AEH ADH
= =  0,5
+ Tứ giác ADHE có 180
AEH ADH
+ =  nên tứ giác ADHE nội tiếp đường
tròn.
0,5
5b
b) Gọi M là trung điểm của BC , đường thẳng DE cắt BC tại N , AH cắt BC
tại .
K Chứng minh rằng DEK DMC
= và NH AM
⊥ .
+ Tứ giác ADHE nội tiếp nên DAH DEH
=
Tứ giác BKHE nội tiếp nên HBK HEK
=
Mà 90
DAH C HBK C DAH HBK
+ = + =  =
Do đó 2 2
DEK DEH HEK HBK DBM
= + = =
0,25
+ Tam giác DBC vuông tại D v à c ó DM l à trung tuy ến DM BM MC
 = =
Có ( )
2 2
DMC DBM BDM DBM
= + =
+ Từ (1) và (2) ta có: DEK DMC
=
0,25
+ Gọi I là giao điểm của đường thẳng NH và đường tròn ngoại tiếp tứ giác
ADHE.
Ta sẽ chứng minh , ,
A I M thẳng hàng.
Ta có tứ giác ADHE nội tiếp đường tròn đường kính AH , do đó 90
AIH =  (3)
Có ( . ) . .
NE NI
NEI NHD g g NE ND NH NI
NH ND
   =  =
∽
0,25
+ Do DEK DMC
= nên tứ giác DEKM nội tiếp
Có ( . ) . .
NE NK
NEK NMD g g NE ND NM NK
NM ND
   =  =
∽
Vậy . .
NH NK
NH NI NK NM
NM NI
=  = . Tam giác NHK và NMI có chung góc
N .
Từ đó suy ra ( . . )
NHK NMI c g c
 
∽ Do đó NHK NMI
= nên tứ giác HIMK nội
tiếp 90
HIM =  (4)
+ Từ (3), (4) ta có 180
AIH HIM
+ = , tức , ,
A I M thẳng hàng. Do vậy
NH AM
⊥ .
0,25
I
K
N
M
H
E
D
B C
A
6
Cho ,
x y là các số thực không âm, thỏa mãn 2
x  và ( )
2 2
xy y
= + . Tìm giá trị
lớn nhất của biểu thức
2
2
2
2 1
1 1
y y
x x
B
x y x y
+
−
= + +
− + +
.
+ Từ giả thiết ( ) ( ) ( ) ( )( )
2 2 1 1 3 1 1
xy y x y x y
= +  − + + + = − +
( )( )
1 1 3
1
1 1 1 1
x y x y
 + + =
− + − +
Ta cũng có
( ) ( )
2 2
1 1 1
1 1
1 1
B
x y
x y
= − + − +
+
− +
0,25
+ Đặt ( )
1 1
; , 0
1 1
a b a b
x y
= = 
− +
Khi đó ,
a b thỏa mãn 3 1
a b ab
+ + = và 2 2
1 1
ab
B a b
a b
= − + − +
+
+ Ta có ( )
2
3
1 3
4
a b ab a b a b
= + +  + + + .
Do đó ( ) ( ) ( )( )
2 2
3 4 4 0 2 3 3 2 0
3
a b a b a b a b a b
+ + + −   + + + −   + 
(vì 2 0
a b
+ +  , với mọi , 0
a b  )
0,25
+ Lại có ( ) ( )
2
2 2 2 2 4 2
1 1 2 2 2 2
2 3
a b
a b a b
 
+
− + −  − −  − 
 
 
 
( )
( ) ( )
1 1 1 1 3 1 1
.
3 3 3 3 2 3 6
a b
ab
a b a b a b
− +
= = −  − =
+ + +
Dấu “=” xảy ra khi
1
3
a b
= = .
0,25
+ Vậy 2 2 4 2 1 8 2 1
1 1
3 6 6
ab
B a b
a b
+
= − + − +  + =
+
Giá trị lớn nhất của B là
8 2 1
6
+
. Dấu “=” xảy ra khi
1
3
a b
= = , khi đó
4
2
x
y
=


=

0,25
PHÒNG GIÁO DỤC VÀ ĐẠO TẠO
HUYỆN BÁ THƯỚC
ĐỀ KHẢO SÁT CHẤT LƯỢNG LỚP 9
NĂM HỌC 2023 – 2024
Môn: Toán
Thời gian: 120 phút (không kể thời gian giao đề)
Ngày thi: …../03/2024
Đề thi có: 01 trang gồm 05 câu.
Câu 1. (2,0 điểm):
1. Giải phương trình: 2
3 4 1 0
x x
− + =
2. Giải hệ phương trình:
 − =

− =

2 7
3 6
x y
x y
Câu 2. (2,0 điểm): Cho biểu thức
4 8 2
: 3 ,
4
2 2
x x x
P
x
x x
   
+
= − +
   
−
+ −
   
với 0, 1
x x
  và 4
x 
1. Rút gọn biểu thức
2. Tìm các giá trị của x để 4
P = −
Câu 3. (2,0 điểm):
1. Cho đường thẳng ( ):
d y ax b
= + . Tìm ,
a b để đường thẳng ( )
d song song với đường thẳng
( )
' : 2 3
d y x
= + và đi qua điểm ( )
1; 1
A − .
2. Cho phương trình 2
1 0
− + − =
x nx n (n là tham số, x là biến số). Tìm các giá trị
của n để phương trình có hai nghiệm phân biệt.
Câu 4. (3,0 điểm): Cho nửa đường tròn đường kính AB = 2R. Từ A và B kẻ hai tiếp tuyến Ax,
By. Qua điểm M thuộc nửa đường tròn kẻ tiếp tuyến thứ ba cắt các tiếp tuyến Ax , By lần lượt
ở C và D. Các đường thẳng AD và BC cắt nhau tại N.
1. Chứng minh AC + BD = CD.
2. Chứng minh OC // BM
3. Xác định vị trí của M để chu vi tứ giác ACDB đạt giá trị nhỏ nhất.
Câu 5. (1,0 điểm): Cho các số thực dương , ,
x y z thỏa mãn 3.
x y z
+ + =
Chứng minh rằng:
1 1 1
3
xy x y yz y z zx z x
+ + 
+ + + + + +
……….. Hết ………
Cán bộ coi thi không giải thích gì thêm.
Họ và tên thí sinh …………………………………….Số báo danh………
ĐỀ CHÍNH THỨC
2
PGD&ĐT BÁ THƯỚC HƯỚNG DẪN VÀ BIỂU CHẤM TOÁN 9
NĂM HỌC 2023 – 2024
Câu NỘI DUNG Điểm
Câu 1
(2,0
điểm)
1. Giải phương trình: 2
3 4 1 0
x x
− + = 1,0
Ta có: ( )
+ + = + − + =
3 4 1 0
a b c
Nên phương trình đã cho có hai nghiệm phân biệt 1
1
x = và =
2
1
3
x
0,75
Vậy phương trình có hai nghiệm phân biệt 1
1
x = và =
2
1
3
x 0,25
2. Giải hệ phương trình:
2 7
3 6
x y
x y
 − =
 
− =

1,0
 − =

− =

2 7
3 6
x y
x y
2 7
2 6 12
x y
x y
 − =
 
− =

2 7
5 5
x y
y
 − =
 
= −

0,5
2 1 7
1
x
y
 + =
 
= −

3
1
x
y
 =
 
= −

0,25
Vậy hệ phương trình có nghiệm duy nhất là
 =

= −

3
1
x
y
0,25
Câu 2
(2,0
điểm)
Câu2. (2,0 điểm): Cho biểu thức
4 8 2
: 3 ,
4
2 2
x x x
P
x
x x
   
+
= − +
   
−
+ −
   
với
0, 1
x x
  và 4
x 
1. Rút gọn biểu thức
2. Tìm các giá trị của x để 4
P = −
2
1. Rút gọn biểu thức P. 1
3
Với 0, 1, 4
x x x
  
( ) ( )
( ) ( )
( )
( ) ( ) ( )
4 8 2
: 3
4
2 2
4 8 2 3 6
:
2 2 2
2 . 2
4 8 8 4 4
:
2
2 . 2
4 2 2
.
2 . 2 4 1
1
x x x
P
x
x x
x x x x
x x x
x x
x x x x
x
x x
x x x
x x x
x
x
   
+
= − +
   
   
−
+ −
   
   
+ −
 
= − +
 
 
 
+ − −
+ −  
 
 
− − −
 
=
  −
+ −
 
− + −
=
+ − −
−
=
−
Vậy với 0, 1, 4
x x x
   thì 1
x
A
x
−
=
−
0,25
0,25
0,25
0,25
2.Tìm các giá trị của x để 4
P = −
1
Với 0, 1, 4
x x x
  
4
4 4 4 4 3 4
3
1
x
P x x x x
x
−
= −  = −  = −  − = −  =
−
16
9
x
 = (thỏa mãn điều kiện).
0,5
0,25
Vậy
16
9
x = thì 4
P = − . 0,25
Câu 3
2,0
điểm
1. Cho đường thẳng ( ):
d y ax b
= + . Tìm ,
a b để đường thẳng ( )
d song song
với đường thẳng ( )
' : 2 3
d y x
= + và đi qua điểm ( )
1; 1
A − . 1
Do ( ) ( )
'
// d
d nên
2
3
a
b
=




.
0,25
Do ( )
d đi qua điểm ( )
1; 1
A − nên:
1 2.1 3
b b
− = +  = − (thỏa mãn điều kiện 3
b  ) .
Vậy 2
a = , 3
b = − .
0,5
Vậy 2
a = , 3
b = − .
0,25
4
2. Cho phương trình 2
1 0
− + − =
x nx n (n là tham số, x là biến số). Tìm
các giá trị của n để phương trình có hai nghiệm phân biệt.
1,0
Ta có : ( ) ( ) ( )
2 2
4 1 2 0, 2
n n n n
 = − − − = −   
Suy ra với mọi 2

n phương trình có hai nghiệm phân biệt 1 2
,
x x
1
Câu 4
3,0
điểm
Câu 4 : Cho nửa đường tròn đường kính AB = 2R. Từ A và B kẻ hai
tiếp tuyến Ax, By. Qua điểm M thuộc nửa đường tròn kẻ tiếp tuyến thứ
ba cắt các tiếp tuyến Ax , By lần lượt ở C và D. Các đường thẳng AD và
BC cắt nhau tại N.
1. Chứng minh AC + BD = CD.
1,0
1/ Theo tính chất hai tiếp tuyến cắt nhau ta có:
CA = CM; DB = DM => AC + BD = CM + DM.
Mà CM + DM = CD => AC + BD = CD
0,5
0,5
2. Chứng minh OC // BM 1,0
2/ Theo tính chất hai tiếp tuyến cắt nhau ta có: OC là tia phân giác của
góc AOM; OD là tia phân giác của góc BOM, mà AOM và BOM là
hai góc kề bù => COD = 900
.
Theo trên COD = 900
nên OC ⊥ OD . (1)
Theo tính chất hai tiếp tuyến cắt nhau ta có: DB = DM; lại có OM
= OB =R
=> OD là trung trực của BM => BM ⊥ OD . (2).
Từ (1) Và (2) => OC // BM ( Vì cùng vuông góc với OD).
0,25
0,25
0,25
0,25
3. Xác định vị trí của M để chu vi tứ giác ACDB đạt giá trị nhỏ nhất. 1,0
3/ Ta có chu vi tứ giác ACDB = AB + AC + CD + BD mà AC + BD = CD
=> Chu vi tứ giác ACDB = AB + 2CD mà AB không đổi 0,25
0,25
/
/
y
x
N
C
D
I
M
B
O
A
5
Chú ý:
- Các cách làm khác nếu đúng vẫn cho điểm tối đa
- Bài hình nếu hình vẽ sai cơ bản hoặc không có hình thì không chấm bài hình..
- Các trường hợp khác tổ chấm thống nhất phương án chấm.
=> Chu vi tứ giác ACDB nhỏ nhất khi CD nhỏ nhất , mà CD nhỏ
nhất khi CD là khoảng cách giữa Ax và By tức là CD vuông góc với Ax
và By. Khi đó CD // AB
=> M phải là trung điểm của cung AB.
0,25
0,25
Câu 5
1,0
điểm
Câu 5. Cho các số thực dương , ,
x y zthỏa mãn 3.
x y z
+ + = Chứng minh
rằng :
1 1 1
3
xy x y yz y z zx z x
+ + 
+ + + + + +
1
Ta chứng minh ( ) ( )
2
1 3
x y xy x y
+ +  + + với mọi ,
x y
Thật vậy, bất đẳng thức trên tương đương với :
( ) ( )
2 2
2 1 2 2 2 6
x y xy x y xy x y
+ + + + +  + +
( ) ( ) ( )
2 2 2
1 1
x y x y
 − + − + − . Dấu " "
= xảy ra 1
x y
 = =
Do đó
1 3
1
x y
xy x y

+ +
+ +
, với , 0
x y  . Dấu " "
= xảy ra 1
x y
 = =
Tương tự ta suy ra
1 1 1 3 3 3
1 1 1
x y y z z x
xy x y yz y z zx z x
+ +  + +
+ + + + + +
+ + + + + +
Dấu " "
= xảy ra ( )
1 1
x y z
 = = =
Ta chứng minh
1 1 1 9
,
m n p m n p
+ + 
+ +
với mọi , , 0
m n p 
Thật vậy, bất đẳng thức trên tương đương với :
1 1 1 9
n p m p m n
m m n n p p
+ + + + + + + + 
6
n m p m p n
m n m p n p
   
 
 + + + + + 
     
     
Theo bđt Cô si ta thấy bất đẳng thức trên luôn đúng. Dấu " "
= xảy ra
m n p
 = =
Do đó :
( )
( )
3 3 3 9 3
3 2
1 1 1 2 3
x y y z z x x y z
+ +  =
+ + + + + + + + +
Từ (1) và (2) suy ra điều phải chứng minh .
Dấu " "
= xảy ra 1
x y z
 = = =
0,25
0,25
0,25
0,25
PHÒNG GIÁO DỤC VÀ ĐÀO TẠO
QUẬN HÀ ĐÔNG
ĐỀ CHÍNH THỨC
ĐỀ KHẢO SÁT CHẤT LƯỢNG LỚP 9 (LẦN 1)
Năm học 2023-2024
Môn: TOÁN
Thời gian làm bài:120 phút (không kể thời gian giao đề)
(Đề gồm có 01 trang)
Câu I. (2,0 điểm)
Cho hai biểu thức
3 1 5 2
;
4
2 2
x x x
P Q
x
x x
+ − −
= = −
−
− +
với 0, 4
x x
  .
1) Tính giá trị của biều thức P khi 25
x =
2) Chứng minh
2
x
Q
x
=
−
3) Tìm giá trị của x để biểu thức
P
Q
đạt giá trị nhỏ nhất.
Câu II. (2,0 điểm)
1) Giải bài toán bằng cách lập phương trình hoặc hệ phương trình
Hai bạn Linh và Chi ở hai địa điểm cách nhau 18 km đạp xe đi ngược chiều nhau để gặp nhau.
Nếu hai bạn khởi hành cùng một lúc thì sẽ gặp nhau sau 40 phút. Nhưng nếu Linh khởi hành trước
18 phút thi các bạn sẽ gặp nhau sau 30 phút tính từ lúc Chi bắt đầu đi. Tính vận tốc của mỗi bạn?
2) Một chiếc cốc có dạng hình trụ với chiều cao 8 cm, bán kính đáy là 3 cm. Hỏi chiếc cốc này có
đựng được 200ml sữa không? (Lấy 3,14
  và bỏ qua bề dày của chiếc cốc).
Câu II. (2,0 điểm)
1) Giài hệ phương trình:
2 2 1 8
2 3 1 3
x y
x y
 − + − =


− − − = −


2) Cho Parabol ( )
2
P :
2
x
y = và đường thẳng (d): 1
y mx m
= − + (với m là tham số).
a) Tìm m để (d) cắt ( )
P tại 2 điểm phân biệt A và B nằm về hai phía của trục tung.
b) Gọi K là giao điểm của (d) và trục Oy . Tìm m để 2
KOA
KOB
S
S
=
Câu IV. (3,0 điểm)
Từ điểm A ở ngoài đường tròn ( )
O;R vẽ hai tiếp tuyến AB,AC(B,C là các tiếp điểm) và cát tuyến
ADE thuộc nửa mặt phẳng bờ là đường thẳng OA không chứa điểm B của đường tròn ( )
O . Gọi H là
giao điểm của OA và BC.
1) Chứng minh bốn điểm A,B,O,C cùng thuộc một đường tròn.
2) Chứng minh AO BC
⊥ tại H và AH.AO AD.AE
=
3) Đường thẳng đi qua điểm D và song song với đường thẳng BE cắt AB,BC lần lượt tại I,K .
Chứng minh tứ giác OHDE nội tiếp và D là trung điểm của IK .
Câu V. (0,5 điểm)
Với các giá trị của m để phương trình ( )
2 2
2 1 9 0
x m x m
− + + + = có nghiệm, hãy tìm giá trị nhỏ
nhất của biểu thức
( )( )
2
2 2 4
m m m
B
m
+ − +
=
---------------Hết---------------
PHÒNG GIÁO DỤC VÀ ĐÀO TẠO
QUẬN HÀ ĐÔNG
HƯỚNG DẪN CHẤM MÔN TOÁN
KHẢO SÁT CHẤT LƯỢNG LỚP 9 (LẦN 1)
Năm học 2023-2024
Câu Nội dung Điểm
Câu I
(2.0
điềm)
1) Tính giá trị của P khi 25
x = với 0, 4
x x
  . 0,5
Ta có ( )
x 25 TMC
= thay vào biểu thức P được
25 3
25 2
P
+
=
−
0,25
Tính được
28 28
5 2 3
P = =
−
0,25
2) Chứng minh
2
x
Q
x
=
−
với 0, 4
x x
  1,0
( )( )
( )( )
1 2 5 2
1 5 2 1 5 2
4 4
2 2 2 2
x x x
x x x x
Q
x x
x x x x
− − + −
− − − −
= − = + =
− −
+ + + −
0,25
( )( )
3 2 5 2
2 2
x x x
x x
− + + −
=
+ −
0,25
( )( )
2
2 2
x x
x x
+
=
+ −
0,25
( )
( )( )
2
2
2 2
x x x
x
x x
+
= =
−
+ −
0,25
3) Tìm giá trị của x để biểu thức
P
Q
đạt giá trị nhỏ nhất 0,5
* Với 0, 4
x x
  biến dối
3 3 2 3 3
:
2 2 2
P x x x x x
x
Q x x x x x x
+ + − +
= =  = = +
− − −
* Vì
3
0, 4 0, 0
x x x
x
    
Áp dụng BĐT Cosi cho 2 số
3
;
x
x
ta có:
3 3
2 2 3
P
x x
Q x x
= +   =
Dấu "=" xảy ra khi ( )
3
3
x x TM
x
=  =
Vậy Min 2 3 3
P
x
Q
=  =
0,25
ĐỀ THI THỬ TUYỂN SINH VÀO LỚP 10 THPT MÔN TOÁN CÁC TỈNH NĂM HỌC 2023-2024 CÓ LỜI GIẢI CHI TIẾT.pdf
ĐỀ THI THỬ TUYỂN SINH VÀO LỚP 10 THPT MÔN TOÁN CÁC TỈNH NĂM HỌC 2023-2024 CÓ LỜI GIẢI CHI TIẾT.pdf
ĐỀ THI THỬ TUYỂN SINH VÀO LỚP 10 THPT MÔN TOÁN CÁC TỈNH NĂM HỌC 2023-2024 CÓ LỜI GIẢI CHI TIẾT.pdf
ĐỀ THI THỬ TUYỂN SINH VÀO LỚP 10 THPT MÔN TOÁN CÁC TỈNH NĂM HỌC 2023-2024 CÓ LỜI GIẢI CHI TIẾT.pdf
ĐỀ THI THỬ TUYỂN SINH VÀO LỚP 10 THPT MÔN TOÁN CÁC TỈNH NĂM HỌC 2023-2024 CÓ LỜI GIẢI CHI TIẾT.pdf
ĐỀ THI THỬ TUYỂN SINH VÀO LỚP 10 THPT MÔN TOÁN CÁC TỈNH NĂM HỌC 2023-2024 CÓ LỜI GIẢI CHI TIẾT.pdf
ĐỀ THI THỬ TUYỂN SINH VÀO LỚP 10 THPT MÔN TOÁN CÁC TỈNH NĂM HỌC 2023-2024 CÓ LỜI GIẢI CHI TIẾT.pdf
ĐỀ THI THỬ TUYỂN SINH VÀO LỚP 10 THPT MÔN TOÁN CÁC TỈNH NĂM HỌC 2023-2024 CÓ LỜI GIẢI CHI TIẾT.pdf
ĐỀ THI THỬ TUYỂN SINH VÀO LỚP 10 THPT MÔN TOÁN CÁC TỈNH NĂM HỌC 2023-2024 CÓ LỜI GIẢI CHI TIẾT.pdf
ĐỀ THI THỬ TUYỂN SINH VÀO LỚP 10 THPT MÔN TOÁN CÁC TỈNH NĂM HỌC 2023-2024 CÓ LỜI GIẢI CHI TIẾT.pdf
ĐỀ THI THỬ TUYỂN SINH VÀO LỚP 10 THPT MÔN TOÁN CÁC TỈNH NĂM HỌC 2023-2024 CÓ LỜI GIẢI CHI TIẾT.pdf
ĐỀ THI THỬ TUYỂN SINH VÀO LỚP 10 THPT MÔN TOÁN CÁC TỈNH NĂM HỌC 2023-2024 CÓ LỜI GIẢI CHI TIẾT.pdf
ĐỀ THI THỬ TUYỂN SINH VÀO LỚP 10 THPT MÔN TOÁN CÁC TỈNH NĂM HỌC 2023-2024 CÓ LỜI GIẢI CHI TIẾT.pdf
ĐỀ THI THỬ TUYỂN SINH VÀO LỚP 10 THPT MÔN TOÁN CÁC TỈNH NĂM HỌC 2023-2024 CÓ LỜI GIẢI CHI TIẾT.pdf
ĐỀ THI THỬ TUYỂN SINH VÀO LỚP 10 THPT MÔN TOÁN CÁC TỈNH NĂM HỌC 2023-2024 CÓ LỜI GIẢI CHI TIẾT.pdf
ĐỀ THI THỬ TUYỂN SINH VÀO LỚP 10 THPT MÔN TOÁN CÁC TỈNH NĂM HỌC 2023-2024 CÓ LỜI GIẢI CHI TIẾT.pdf
ĐỀ THI THỬ TUYỂN SINH VÀO LỚP 10 THPT MÔN TOÁN CÁC TỈNH NĂM HỌC 2023-2024 CÓ LỜI GIẢI CHI TIẾT.pdf
ĐỀ THI THỬ TUYỂN SINH VÀO LỚP 10 THPT MÔN TOÁN CÁC TỈNH NĂM HỌC 2023-2024 CÓ LỜI GIẢI CHI TIẾT.pdf
ĐỀ THI THỬ TUYỂN SINH VÀO LỚP 10 THPT MÔN TOÁN CÁC TỈNH NĂM HỌC 2023-2024 CÓ LỜI GIẢI CHI TIẾT.pdf
ĐỀ THI THỬ TUYỂN SINH VÀO LỚP 10 THPT MÔN TOÁN CÁC TỈNH NĂM HỌC 2023-2024 CÓ LỜI GIẢI CHI TIẾT.pdf
ĐỀ THI THỬ TUYỂN SINH VÀO LỚP 10 THPT MÔN TOÁN CÁC TỈNH NĂM HỌC 2023-2024 CÓ LỜI GIẢI CHI TIẾT.pdf
ĐỀ THI THỬ TUYỂN SINH VÀO LỚP 10 THPT MÔN TOÁN CÁC TỈNH NĂM HỌC 2023-2024 CÓ LỜI GIẢI CHI TIẾT.pdf
ĐỀ THI THỬ TUYỂN SINH VÀO LỚP 10 THPT MÔN TOÁN CÁC TỈNH NĂM HỌC 2023-2024 CÓ LỜI GIẢI CHI TIẾT.pdf
ĐỀ THI THỬ TUYỂN SINH VÀO LỚP 10 THPT MÔN TOÁN CÁC TỈNH NĂM HỌC 2023-2024 CÓ LỜI GIẢI CHI TIẾT.pdf
ĐỀ THI THỬ TUYỂN SINH VÀO LỚP 10 THPT MÔN TOÁN CÁC TỈNH NĂM HỌC 2023-2024 CÓ LỜI GIẢI CHI TIẾT.pdf
ĐỀ THI THỬ TUYỂN SINH VÀO LỚP 10 THPT MÔN TOÁN CÁC TỈNH NĂM HỌC 2023-2024 CÓ LỜI GIẢI CHI TIẾT.pdf
ĐỀ THI THỬ TUYỂN SINH VÀO LỚP 10 THPT MÔN TOÁN CÁC TỈNH NĂM HỌC 2023-2024 CÓ LỜI GIẢI CHI TIẾT.pdf
ĐỀ THI THỬ TUYỂN SINH VÀO LỚP 10 THPT MÔN TOÁN CÁC TỈNH NĂM HỌC 2023-2024 CÓ LỜI GIẢI CHI TIẾT.pdf
ĐỀ THI THỬ TUYỂN SINH VÀO LỚP 10 THPT MÔN TOÁN CÁC TỈNH NĂM HỌC 2023-2024 CÓ LỜI GIẢI CHI TIẾT.pdf
ĐỀ THI THỬ TUYỂN SINH VÀO LỚP 10 THPT MÔN TOÁN CÁC TỈNH NĂM HỌC 2023-2024 CÓ LỜI GIẢI CHI TIẾT.pdf
ĐỀ THI THỬ TUYỂN SINH VÀO LỚP 10 THPT MÔN TOÁN CÁC TỈNH NĂM HỌC 2023-2024 CÓ LỜI GIẢI CHI TIẾT.pdf
ĐỀ THI THỬ TUYỂN SINH VÀO LỚP 10 THPT MÔN TOÁN CÁC TỈNH NĂM HỌC 2023-2024 CÓ LỜI GIẢI CHI TIẾT.pdf
ĐỀ THI THỬ TUYỂN SINH VÀO LỚP 10 THPT MÔN TOÁN CÁC TỈNH NĂM HỌC 2023-2024 CÓ LỜI GIẢI CHI TIẾT.pdf
ĐỀ THI THỬ TUYỂN SINH VÀO LỚP 10 THPT MÔN TOÁN CÁC TỈNH NĂM HỌC 2023-2024 CÓ LỜI GIẢI CHI TIẾT.pdf
ĐỀ THI THỬ TUYỂN SINH VÀO LỚP 10 THPT MÔN TOÁN CÁC TỈNH NĂM HỌC 2023-2024 CÓ LỜI GIẢI CHI TIẾT.pdf
ĐỀ THI THỬ TUYỂN SINH VÀO LỚP 10 THPT MÔN TOÁN CÁC TỈNH NĂM HỌC 2023-2024 CÓ LỜI GIẢI CHI TIẾT.pdf
ĐỀ THI THỬ TUYỂN SINH VÀO LỚP 10 THPT MÔN TOÁN CÁC TỈNH NĂM HỌC 2023-2024 CÓ LỜI GIẢI CHI TIẾT.pdf
ĐỀ THI THỬ TUYỂN SINH VÀO LỚP 10 THPT MÔN TOÁN CÁC TỈNH NĂM HỌC 2023-2024 CÓ LỜI GIẢI CHI TIẾT.pdf
ĐỀ THI THỬ TUYỂN SINH VÀO LỚP 10 THPT MÔN TOÁN CÁC TỈNH NĂM HỌC 2023-2024 CÓ LỜI GIẢI CHI TIẾT.pdf
ĐỀ THI THỬ TUYỂN SINH VÀO LỚP 10 THPT MÔN TOÁN CÁC TỈNH NĂM HỌC 2023-2024 CÓ LỜI GIẢI CHI TIẾT.pdf
ĐỀ THI THỬ TUYỂN SINH VÀO LỚP 10 THPT MÔN TOÁN CÁC TỈNH NĂM HỌC 2023-2024 CÓ LỜI GIẢI CHI TIẾT.pdf
ĐỀ THI THỬ TUYỂN SINH VÀO LỚP 10 THPT MÔN TOÁN CÁC TỈNH NĂM HỌC 2023-2024 CÓ LỜI GIẢI CHI TIẾT.pdf
ĐỀ THI THỬ TUYỂN SINH VÀO LỚP 10 THPT MÔN TOÁN CÁC TỈNH NĂM HỌC 2023-2024 CÓ LỜI GIẢI CHI TIẾT.pdf
ĐỀ THI THỬ TUYỂN SINH VÀO LỚP 10 THPT MÔN TOÁN CÁC TỈNH NĂM HỌC 2023-2024 CÓ LỜI GIẢI CHI TIẾT.pdf
ĐỀ THI THỬ TUYỂN SINH VÀO LỚP 10 THPT MÔN TOÁN CÁC TỈNH NĂM HỌC 2023-2024 CÓ LỜI GIẢI CHI TIẾT.pdf
ĐỀ THI THỬ TUYỂN SINH VÀO LỚP 10 THPT MÔN TOÁN CÁC TỈNH NĂM HỌC 2023-2024 CÓ LỜI GIẢI CHI TIẾT.pdf
ĐỀ THI THỬ TUYỂN SINH VÀO LỚP 10 THPT MÔN TOÁN CÁC TỈNH NĂM HỌC 2023-2024 CÓ LỜI GIẢI CHI TIẾT.pdf
ĐỀ THI THỬ TUYỂN SINH VÀO LỚP 10 THPT MÔN TOÁN CÁC TỈNH NĂM HỌC 2023-2024 CÓ LỜI GIẢI CHI TIẾT.pdf
ĐỀ THI THỬ TUYỂN SINH VÀO LỚP 10 THPT MÔN TOÁN CÁC TỈNH NĂM HỌC 2023-2024 CÓ LỜI GIẢI CHI TIẾT.pdf
ĐỀ THI THỬ TUYỂN SINH VÀO LỚP 10 THPT MÔN TOÁN CÁC TỈNH NĂM HỌC 2023-2024 CÓ LỜI GIẢI CHI TIẾT.pdf
ĐỀ THI THỬ TUYỂN SINH VÀO LỚP 10 THPT MÔN TOÁN CÁC TỈNH NĂM HỌC 2023-2024 CÓ LỜI GIẢI CHI TIẾT.pdf
ĐỀ THI THỬ TUYỂN SINH VÀO LỚP 10 THPT MÔN TOÁN CÁC TỈNH NĂM HỌC 2023-2024 CÓ LỜI GIẢI CHI TIẾT.pdf
ĐỀ THI THỬ TUYỂN SINH VÀO LỚP 10 THPT MÔN TOÁN CÁC TỈNH NĂM HỌC 2023-2024 CÓ LỜI GIẢI CHI TIẾT.pdf
ĐỀ THI THỬ TUYỂN SINH VÀO LỚP 10 THPT MÔN TOÁN CÁC TỈNH NĂM HỌC 2023-2024 CÓ LỜI GIẢI CHI TIẾT.pdf
ĐỀ THI THỬ TUYỂN SINH VÀO LỚP 10 THPT MÔN TOÁN CÁC TỈNH NĂM HỌC 2023-2024 CÓ LỜI GIẢI CHI TIẾT.pdf
ĐỀ THI THỬ TUYỂN SINH VÀO LỚP 10 THPT MÔN TOÁN CÁC TỈNH NĂM HỌC 2023-2024 CÓ LỜI GIẢI CHI TIẾT.pdf
ĐỀ THI THỬ TUYỂN SINH VÀO LỚP 10 THPT MÔN TOÁN CÁC TỈNH NĂM HỌC 2023-2024 CÓ LỜI GIẢI CHI TIẾT.pdf
ĐỀ THI THỬ TUYỂN SINH VÀO LỚP 10 THPT MÔN TOÁN CÁC TỈNH NĂM HỌC 2023-2024 CÓ LỜI GIẢI CHI TIẾT.pdf
ĐỀ THI THỬ TUYỂN SINH VÀO LỚP 10 THPT MÔN TOÁN CÁC TỈNH NĂM HỌC 2023-2024 CÓ LỜI GIẢI CHI TIẾT.pdf
ĐỀ THI THỬ TUYỂN SINH VÀO LỚP 10 THPT MÔN TOÁN CÁC TỈNH NĂM HỌC 2023-2024 CÓ LỜI GIẢI CHI TIẾT.pdf
ĐỀ THI THỬ TUYỂN SINH VÀO LỚP 10 THPT MÔN TOÁN CÁC TỈNH NĂM HỌC 2023-2024 CÓ LỜI GIẢI CHI TIẾT.pdf
ĐỀ THI THỬ TUYỂN SINH VÀO LỚP 10 THPT MÔN TOÁN CÁC TỈNH NĂM HỌC 2023-2024 CÓ LỜI GIẢI CHI TIẾT.pdf
ĐỀ THI THỬ TUYỂN SINH VÀO LỚP 10 THPT MÔN TOÁN CÁC TỈNH NĂM HỌC 2023-2024 CÓ LỜI GIẢI CHI TIẾT.pdf
ĐỀ THI THỬ TUYỂN SINH VÀO LỚP 10 THPT MÔN TOÁN CÁC TỈNH NĂM HỌC 2023-2024 CÓ LỜI GIẢI CHI TIẾT.pdf
ĐỀ THI THỬ TUYỂN SINH VÀO LỚP 10 THPT MÔN TOÁN CÁC TỈNH NĂM HỌC 2023-2024 CÓ LỜI GIẢI CHI TIẾT.pdf
ĐỀ THI THỬ TUYỂN SINH VÀO LỚP 10 THPT MÔN TOÁN CÁC TỈNH NĂM HỌC 2023-2024 CÓ LỜI GIẢI CHI TIẾT.pdf
ĐỀ THI THỬ TUYỂN SINH VÀO LỚP 10 THPT MÔN TOÁN CÁC TỈNH NĂM HỌC 2023-2024 CÓ LỜI GIẢI CHI TIẾT.pdf
ĐỀ THI THỬ TUYỂN SINH VÀO LỚP 10 THPT MÔN TOÁN CÁC TỈNH NĂM HỌC 2023-2024 CÓ LỜI GIẢI CHI TIẾT.pdf
ĐỀ THI THỬ TUYỂN SINH VÀO LỚP 10 THPT MÔN TOÁN CÁC TỈNH NĂM HỌC 2023-2024 CÓ LỜI GIẢI CHI TIẾT.pdf
ĐỀ THI THỬ TUYỂN SINH VÀO LỚP 10 THPT MÔN TOÁN CÁC TỈNH NĂM HỌC 2023-2024 CÓ LỜI GIẢI CHI TIẾT.pdf
ĐỀ THI THỬ TUYỂN SINH VÀO LỚP 10 THPT MÔN TOÁN CÁC TỈNH NĂM HỌC 2023-2024 CÓ LỜI GIẢI CHI TIẾT.pdf
ĐỀ THI THỬ TUYỂN SINH VÀO LỚP 10 THPT MÔN TOÁN CÁC TỈNH NĂM HỌC 2023-2024 CÓ LỜI GIẢI CHI TIẾT.pdf
ĐỀ THI THỬ TUYỂN SINH VÀO LỚP 10 THPT MÔN TOÁN CÁC TỈNH NĂM HỌC 2023-2024 CÓ LỜI GIẢI CHI TIẾT.pdf
ĐỀ THI THỬ TUYỂN SINH VÀO LỚP 10 THPT MÔN TOÁN CÁC TỈNH NĂM HỌC 2023-2024 CÓ LỜI GIẢI CHI TIẾT.pdf
ĐỀ THI THỬ TUYỂN SINH VÀO LỚP 10 THPT MÔN TOÁN CÁC TỈNH NĂM HỌC 2023-2024 CÓ LỜI GIẢI CHI TIẾT.pdf
ĐỀ THI THỬ TUYỂN SINH VÀO LỚP 10 THPT MÔN TOÁN CÁC TỈNH NĂM HỌC 2023-2024 CÓ LỜI GIẢI CHI TIẾT.pdf
ĐỀ THI THỬ TUYỂN SINH VÀO LỚP 10 THPT MÔN TOÁN CÁC TỈNH NĂM HỌC 2023-2024 CÓ LỜI GIẢI CHI TIẾT.pdf
ĐỀ THI THỬ TUYỂN SINH VÀO LỚP 10 THPT MÔN TOÁN CÁC TỈNH NĂM HỌC 2023-2024 CÓ LỜI GIẢI CHI TIẾT.pdf
ĐỀ THI THỬ TUYỂN SINH VÀO LỚP 10 THPT MÔN TOÁN CÁC TỈNH NĂM HỌC 2023-2024 CÓ LỜI GIẢI CHI TIẾT.pdf
ĐỀ THI THỬ TUYỂN SINH VÀO LỚP 10 THPT MÔN TOÁN CÁC TỈNH NĂM HỌC 2023-2024 CÓ LỜI GIẢI CHI TIẾT.pdf
ĐỀ THI THỬ TUYỂN SINH VÀO LỚP 10 THPT MÔN TOÁN CÁC TỈNH NĂM HỌC 2023-2024 CÓ LỜI GIẢI CHI TIẾT.pdf
ĐỀ THI THỬ TUYỂN SINH VÀO LỚP 10 THPT MÔN TOÁN CÁC TỈNH NĂM HỌC 2023-2024 CÓ LỜI GIẢI CHI TIẾT.pdf
ĐỀ THI THỬ TUYỂN SINH VÀO LỚP 10 THPT MÔN TOÁN CÁC TỈNH NĂM HỌC 2023-2024 CÓ LỜI GIẢI CHI TIẾT.pdf
ĐỀ THI THỬ TUYỂN SINH VÀO LỚP 10 THPT MÔN TOÁN CÁC TỈNH NĂM HỌC 2023-2024 CÓ LỜI GIẢI CHI TIẾT.pdf
ĐỀ THI THỬ TUYỂN SINH VÀO LỚP 10 THPT MÔN TOÁN CÁC TỈNH NĂM HỌC 2023-2024 CÓ LỜI GIẢI CHI TIẾT.pdf
ĐỀ THI THỬ TUYỂN SINH VÀO LỚP 10 THPT MÔN TOÁN CÁC TỈNH NĂM HỌC 2023-2024 CÓ LỜI GIẢI CHI TIẾT.pdf
ĐỀ THI THỬ TUYỂN SINH VÀO LỚP 10 THPT MÔN TOÁN CÁC TỈNH NĂM HỌC 2023-2024 CÓ LỜI GIẢI CHI TIẾT.pdf
ĐỀ THI THỬ TUYỂN SINH VÀO LỚP 10 THPT MÔN TOÁN CÁC TỈNH NĂM HỌC 2023-2024 CÓ LỜI GIẢI CHI TIẾT.pdf
ĐỀ THI THỬ TUYỂN SINH VÀO LỚP 10 THPT MÔN TOÁN CÁC TỈNH NĂM HỌC 2023-2024 CÓ LỜI GIẢI CHI TIẾT.pdf
ĐỀ THI THỬ TUYỂN SINH VÀO LỚP 10 THPT MÔN TOÁN CÁC TỈNH NĂM HỌC 2023-2024 CÓ LỜI GIẢI CHI TIẾT.pdf
ĐỀ THI THỬ TUYỂN SINH VÀO LỚP 10 THPT MÔN TOÁN CÁC TỈNH NĂM HỌC 2023-2024 CÓ LỜI GIẢI CHI TIẾT.pdf
ĐỀ THI THỬ TUYỂN SINH VÀO LỚP 10 THPT MÔN TOÁN CÁC TỈNH NĂM HỌC 2023-2024 CÓ LỜI GIẢI CHI TIẾT.pdf
ĐỀ THI THỬ TUYỂN SINH VÀO LỚP 10 THPT MÔN TOÁN CÁC TỈNH NĂM HỌC 2023-2024 CÓ LỜI GIẢI CHI TIẾT.pdf
ĐỀ THI THỬ TUYỂN SINH VÀO LỚP 10 THPT MÔN TOÁN CÁC TỈNH NĂM HỌC 2023-2024 CÓ LỜI GIẢI CHI TIẾT.pdf
ĐỀ THI THỬ TUYỂN SINH VÀO LỚP 10 THPT MÔN TOÁN CÁC TỈNH NĂM HỌC 2023-2024 CÓ LỜI GIẢI CHI TIẾT.pdf
ĐỀ THI THỬ TUYỂN SINH VÀO LỚP 10 THPT MÔN TOÁN CÁC TỈNH NĂM HỌC 2023-2024 CÓ LỜI GIẢI CHI TIẾT.pdf
ĐỀ THI THỬ TUYỂN SINH VÀO LỚP 10 THPT MÔN TOÁN CÁC TỈNH NĂM HỌC 2023-2024 CÓ LỜI GIẢI CHI TIẾT.pdf
ĐỀ THI THỬ TUYỂN SINH VÀO LỚP 10 THPT MÔN TOÁN CÁC TỈNH NĂM HỌC 2023-2024 CÓ LỜI GIẢI CHI TIẾT.pdf
ĐỀ THI THỬ TUYỂN SINH VÀO LỚP 10 THPT MÔN TOÁN CÁC TỈNH NĂM HỌC 2023-2024 CÓ LỜI GIẢI CHI TIẾT.pdf
ĐỀ THI THỬ TUYỂN SINH VÀO LỚP 10 THPT MÔN TOÁN CÁC TỈNH NĂM HỌC 2023-2024 CÓ LỜI GIẢI CHI TIẾT.pdf
ĐỀ THI THỬ TUYỂN SINH VÀO LỚP 10 THPT MÔN TOÁN CÁC TỈNH NĂM HỌC 2023-2024 CÓ LỜI GIẢI CHI TIẾT.pdf
ĐỀ THI THỬ TUYỂN SINH VÀO LỚP 10 THPT MÔN TOÁN CÁC TỈNH NĂM HỌC 2023-2024 CÓ LỜI GIẢI CHI TIẾT.pdf
ĐỀ THI THỬ TUYỂN SINH VÀO LỚP 10 THPT MÔN TOÁN CÁC TỈNH NĂM HỌC 2023-2024 CÓ LỜI GIẢI CHI TIẾT.pdf
ĐỀ THI THỬ TUYỂN SINH VÀO LỚP 10 THPT MÔN TOÁN CÁC TỈNH NĂM HỌC 2023-2024 CÓ LỜI GIẢI CHI TIẾT.pdf
ĐỀ THI THỬ TUYỂN SINH VÀO LỚP 10 THPT MÔN TOÁN CÁC TỈNH NĂM HỌC 2023-2024 CÓ LỜI GIẢI CHI TIẾT.pdf
ĐỀ THI THỬ TUYỂN SINH VÀO LỚP 10 THPT MÔN TOÁN CÁC TỈNH NĂM HỌC 2023-2024 CÓ LỜI GIẢI CHI TIẾT.pdf
ĐỀ THI THỬ TUYỂN SINH VÀO LỚP 10 THPT MÔN TOÁN CÁC TỈNH NĂM HỌC 2023-2024 CÓ LỜI GIẢI CHI TIẾT.pdf
ĐỀ THI THỬ TUYỂN SINH VÀO LỚP 10 THPT MÔN TOÁN CÁC TỈNH NĂM HỌC 2023-2024 CÓ LỜI GIẢI CHI TIẾT.pdf
ĐỀ THI THỬ TUYỂN SINH VÀO LỚP 10 THPT MÔN TOÁN CÁC TỈNH NĂM HỌC 2023-2024 CÓ LỜI GIẢI CHI TIẾT.pdf
ĐỀ THI THỬ TUYỂN SINH VÀO LỚP 10 THPT MÔN TOÁN CÁC TỈNH NĂM HỌC 2023-2024 CÓ LỜI GIẢI CHI TIẾT.pdf
ĐỀ THI THỬ TUYỂN SINH VÀO LỚP 10 THPT MÔN TOÁN CÁC TỈNH NĂM HỌC 2023-2024 CÓ LỜI GIẢI CHI TIẾT.pdf
ĐỀ THI THỬ TUYỂN SINH VÀO LỚP 10 THPT MÔN TOÁN CÁC TỈNH NĂM HỌC 2023-2024 CÓ LỜI GIẢI CHI TIẾT.pdf
ĐỀ THI THỬ TUYỂN SINH VÀO LỚP 10 THPT MÔN TOÁN CÁC TỈNH NĂM HỌC 2023-2024 CÓ LỜI GIẢI CHI TIẾT.pdf
ĐỀ THI THỬ TUYỂN SINH VÀO LỚP 10 THPT MÔN TOÁN CÁC TỈNH NĂM HỌC 2023-2024 CÓ LỜI GIẢI CHI TIẾT.pdf
ĐỀ THI THỬ TUYỂN SINH VÀO LỚP 10 THPT MÔN TOÁN CÁC TỈNH NĂM HỌC 2023-2024 CÓ LỜI GIẢI CHI TIẾT.pdf
ĐỀ THI THỬ TUYỂN SINH VÀO LỚP 10 THPT MÔN TOÁN CÁC TỈNH NĂM HỌC 2023-2024 CÓ LỜI GIẢI CHI TIẾT.pdf
ĐỀ THI THỬ TUYỂN SINH VÀO LỚP 10 THPT MÔN TOÁN CÁC TỈNH NĂM HỌC 2023-2024 CÓ LỜI GIẢI CHI TIẾT.pdf
ĐỀ THI THỬ TUYỂN SINH VÀO LỚP 10 THPT MÔN TOÁN CÁC TỈNH NĂM HỌC 2023-2024 CÓ LỜI GIẢI CHI TIẾT.pdf
ĐỀ THI THỬ TUYỂN SINH VÀO LỚP 10 THPT MÔN TOÁN CÁC TỈNH NĂM HỌC 2023-2024 CÓ LỜI GIẢI CHI TIẾT.pdf
ĐỀ THI THỬ TUYỂN SINH VÀO LỚP 10 THPT MÔN TOÁN CÁC TỈNH NĂM HỌC 2023-2024 CÓ LỜI GIẢI CHI TIẾT.pdf
ĐỀ THI THỬ TUYỂN SINH VÀO LỚP 10 THPT MÔN TOÁN CÁC TỈNH NĂM HỌC 2023-2024 CÓ LỜI GIẢI CHI TIẾT.pdf
ĐỀ THI THỬ TUYỂN SINH VÀO LỚP 10 THPT MÔN TOÁN CÁC TỈNH NĂM HỌC 2023-2024 CÓ LỜI GIẢI CHI TIẾT.pdf
ĐỀ THI THỬ TUYỂN SINH VÀO LỚP 10 THPT MÔN TOÁN CÁC TỈNH NĂM HỌC 2023-2024 CÓ LỜI GIẢI CHI TIẾT.pdf
ĐỀ THI THỬ TUYỂN SINH VÀO LỚP 10 THPT MÔN TOÁN CÁC TỈNH NĂM HỌC 2023-2024 CÓ LỜI GIẢI CHI TIẾT.pdf
ĐỀ THI THỬ TUYỂN SINH VÀO LỚP 10 THPT MÔN TOÁN CÁC TỈNH NĂM HỌC 2023-2024 CÓ LỜI GIẢI CHI TIẾT.pdf
ĐỀ THI THỬ TUYỂN SINH VÀO LỚP 10 THPT MÔN TOÁN CÁC TỈNH NĂM HỌC 2023-2024 CÓ LỜI GIẢI CHI TIẾT.pdf
ĐỀ THI THỬ TUYỂN SINH VÀO LỚP 10 THPT MÔN TOÁN CÁC TỈNH NĂM HỌC 2023-2024 CÓ LỜI GIẢI CHI TIẾT.pdf
ĐỀ THI THỬ TUYỂN SINH VÀO LỚP 10 THPT MÔN TOÁN CÁC TỈNH NĂM HỌC 2023-2024 CÓ LỜI GIẢI CHI TIẾT.pdf
ĐỀ THI THỬ TUYỂN SINH VÀO LỚP 10 THPT MÔN TOÁN CÁC TỈNH NĂM HỌC 2023-2024 CÓ LỜI GIẢI CHI TIẾT.pdf
ĐỀ THI THỬ TUYỂN SINH VÀO LỚP 10 THPT MÔN TOÁN CÁC TỈNH NĂM HỌC 2023-2024 CÓ LỜI GIẢI CHI TIẾT.pdf
ĐỀ THI THỬ TUYỂN SINH VÀO LỚP 10 THPT MÔN TOÁN CÁC TỈNH NĂM HỌC 2023-2024 CÓ LỜI GIẢI CHI TIẾT.pdf
ĐỀ THI THỬ TUYỂN SINH VÀO LỚP 10 THPT MÔN TOÁN CÁC TỈNH NĂM HỌC 2023-2024 CÓ LỜI GIẢI CHI TIẾT.pdf
ĐỀ THI THỬ TUYỂN SINH VÀO LỚP 10 THPT MÔN TOÁN CÁC TỈNH NĂM HỌC 2023-2024 CÓ LỜI GIẢI CHI TIẾT.pdf
ĐỀ THI THỬ TUYỂN SINH VÀO LỚP 10 THPT MÔN TOÁN CÁC TỈNH NĂM HỌC 2023-2024 CÓ LỜI GIẢI CHI TIẾT.pdf
ĐỀ THI THỬ TUYỂN SINH VÀO LỚP 10 THPT MÔN TOÁN CÁC TỈNH NĂM HỌC 2023-2024 CÓ LỜI GIẢI CHI TIẾT.pdf
ĐỀ THI THỬ TUYỂN SINH VÀO LỚP 10 THPT MÔN TOÁN CÁC TỈNH NĂM HỌC 2023-2024 CÓ LỜI GIẢI CHI TIẾT.pdf
ĐỀ THI THỬ TUYỂN SINH VÀO LỚP 10 THPT MÔN TOÁN CÁC TỈNH NĂM HỌC 2023-2024 CÓ LỜI GIẢI CHI TIẾT.pdf
ĐỀ THI THỬ TUYỂN SINH VÀO LỚP 10 THPT MÔN TOÁN CÁC TỈNH NĂM HỌC 2023-2024 CÓ LỜI GIẢI CHI TIẾT.pdf
ĐỀ THI THỬ TUYỂN SINH VÀO LỚP 10 THPT MÔN TOÁN CÁC TỈNH NĂM HỌC 2023-2024 CÓ LỜI GIẢI CHI TIẾT.pdf
ĐỀ THI THỬ TUYỂN SINH VÀO LỚP 10 THPT MÔN TOÁN CÁC TỈNH NĂM HỌC 2023-2024 CÓ LỜI GIẢI CHI TIẾT.pdf
ĐỀ THI THỬ TUYỂN SINH VÀO LỚP 10 THPT MÔN TOÁN CÁC TỈNH NĂM HỌC 2023-2024 CÓ LỜI GIẢI CHI TIẾT.pdf
ĐỀ THI THỬ TUYỂN SINH VÀO LỚP 10 THPT MÔN TOÁN CÁC TỈNH NĂM HỌC 2023-2024 CÓ LỜI GIẢI CHI TIẾT.pdf
ĐỀ THI THỬ TUYỂN SINH VÀO LỚP 10 THPT MÔN TOÁN CÁC TỈNH NĂM HỌC 2023-2024 CÓ LỜI GIẢI CHI TIẾT.pdf
ĐỀ THI THỬ TUYỂN SINH VÀO LỚP 10 THPT MÔN TOÁN CÁC TỈNH NĂM HỌC 2023-2024 CÓ LỜI GIẢI CHI TIẾT.pdf
ĐỀ THI THỬ TUYỂN SINH VÀO LỚP 10 THPT MÔN TOÁN CÁC TỈNH NĂM HỌC 2023-2024 CÓ LỜI GIẢI CHI TIẾT.pdf
ĐỀ THI THỬ TUYỂN SINH VÀO LỚP 10 THPT MÔN TOÁN CÁC TỈNH NĂM HỌC 2023-2024 CÓ LỜI GIẢI CHI TIẾT.pdf
ĐỀ THI THỬ TUYỂN SINH VÀO LỚP 10 THPT MÔN TOÁN CÁC TỈNH NĂM HỌC 2023-2024 CÓ LỜI GIẢI CHI TIẾT.pdf
ĐỀ THI THỬ TUYỂN SINH VÀO LỚP 10 THPT MÔN TOÁN CÁC TỈNH NĂM HỌC 2023-2024 CÓ LỜI GIẢI CHI TIẾT.pdf
ĐỀ THI THỬ TUYỂN SINH VÀO LỚP 10 THPT MÔN TOÁN CÁC TỈNH NĂM HỌC 2023-2024 CÓ LỜI GIẢI CHI TIẾT.pdf
ĐỀ THI THỬ TUYỂN SINH VÀO LỚP 10 THPT MÔN TOÁN CÁC TỈNH NĂM HỌC 2023-2024 CÓ LỜI GIẢI CHI TIẾT.pdf
ĐỀ THI THỬ TUYỂN SINH VÀO LỚP 10 THPT MÔN TOÁN CÁC TỈNH NĂM HỌC 2023-2024 CÓ LỜI GIẢI CHI TIẾT.pdf
ĐỀ THI THỬ TUYỂN SINH VÀO LỚP 10 THPT MÔN TOÁN CÁC TỈNH NĂM HỌC 2023-2024 CÓ LỜI GIẢI CHI TIẾT.pdf
ĐỀ THI THỬ TUYỂN SINH VÀO LỚP 10 THPT MÔN TOÁN CÁC TỈNH NĂM HỌC 2023-2024 CÓ LỜI GIẢI CHI TIẾT.pdf
ĐỀ THI THỬ TUYỂN SINH VÀO LỚP 10 THPT MÔN TOÁN CÁC TỈNH NĂM HỌC 2023-2024 CÓ LỜI GIẢI CHI TIẾT.pdf
ĐỀ THI THỬ TUYỂN SINH VÀO LỚP 10 THPT MÔN TOÁN CÁC TỈNH NĂM HỌC 2023-2024 CÓ LỜI GIẢI CHI TIẾT.pdf
ĐỀ THI THỬ TUYỂN SINH VÀO LỚP 10 THPT MÔN TOÁN CÁC TỈNH NĂM HỌC 2023-2024 CÓ LỜI GIẢI CHI TIẾT.pdf
ĐỀ THI THỬ TUYỂN SINH VÀO LỚP 10 THPT MÔN TOÁN CÁC TỈNH NĂM HỌC 2023-2024 CÓ LỜI GIẢI CHI TIẾT.pdf
ĐỀ THI THỬ TUYỂN SINH VÀO LỚP 10 THPT MÔN TOÁN CÁC TỈNH NĂM HỌC 2023-2024 CÓ LỜI GIẢI CHI TIẾT.pdf
ĐỀ THI THỬ TUYỂN SINH VÀO LỚP 10 THPT MÔN TOÁN CÁC TỈNH NĂM HỌC 2023-2024 CÓ LỜI GIẢI CHI TIẾT.pdf
ĐỀ THI THỬ TUYỂN SINH VÀO LỚP 10 THPT MÔN TOÁN CÁC TỈNH NĂM HỌC 2023-2024 CÓ LỜI GIẢI CHI TIẾT.pdf
ĐỀ THI THỬ TUYỂN SINH VÀO LỚP 10 THPT MÔN TOÁN CÁC TỈNH NĂM HỌC 2023-2024 CÓ LỜI GIẢI CHI TIẾT.pdf
ĐỀ THI THỬ TUYỂN SINH VÀO LỚP 10 THPT MÔN TOÁN CÁC TỈNH NĂM HỌC 2023-2024 CÓ LỜI GIẢI CHI TIẾT.pdf
ĐỀ THI THỬ TUYỂN SINH VÀO LỚP 10 THPT MÔN TOÁN CÁC TỈNH NĂM HỌC 2023-2024 CÓ LỜI GIẢI CHI TIẾT.pdf
ĐỀ THI THỬ TUYỂN SINH VÀO LỚP 10 THPT MÔN TOÁN CÁC TỈNH NĂM HỌC 2023-2024 CÓ LỜI GIẢI CHI TIẾT.pdf
ĐỀ THI THỬ TUYỂN SINH VÀO LỚP 10 THPT MÔN TOÁN CÁC TỈNH NĂM HỌC 2023-2024 CÓ LỜI GIẢI CHI TIẾT.pdf
ĐỀ THI THỬ TUYỂN SINH VÀO LỚP 10 THPT MÔN TOÁN CÁC TỈNH NĂM HỌC 2023-2024 CÓ LỜI GIẢI CHI TIẾT.pdf
ĐỀ THI THỬ TUYỂN SINH VÀO LỚP 10 THPT MÔN TOÁN CÁC TỈNH NĂM HỌC 2023-2024 CÓ LỜI GIẢI CHI TIẾT.pdf
ĐỀ THI THỬ TUYỂN SINH VÀO LỚP 10 THPT MÔN TOÁN CÁC TỈNH NĂM HỌC 2023-2024 CÓ LỜI GIẢI CHI TIẾT.pdf
ĐỀ THI THỬ TUYỂN SINH VÀO LỚP 10 THPT MÔN TOÁN CÁC TỈNH NĂM HỌC 2023-2024 CÓ LỜI GIẢI CHI TIẾT.pdf
ĐỀ THI THỬ TUYỂN SINH VÀO LỚP 10 THPT MÔN TOÁN CÁC TỈNH NĂM HỌC 2023-2024 CÓ LỜI GIẢI CHI TIẾT.pdf
ĐỀ THI THỬ TUYỂN SINH VÀO LỚP 10 THPT MÔN TOÁN CÁC TỈNH NĂM HỌC 2023-2024 CÓ LỜI GIẢI CHI TIẾT.pdf
ĐỀ THI THỬ TUYỂN SINH VÀO LỚP 10 THPT MÔN TOÁN CÁC TỈNH NĂM HỌC 2023-2024 CÓ LỜI GIẢI CHI TIẾT.pdf
ĐỀ THI THỬ TUYỂN SINH VÀO LỚP 10 THPT MÔN TOÁN CÁC TỈNH NĂM HỌC 2023-2024 CÓ LỜI GIẢI CHI TIẾT.pdf
ĐỀ THI THỬ TUYỂN SINH VÀO LỚP 10 THPT MÔN TOÁN CÁC TỈNH NĂM HỌC 2023-2024 CÓ LỜI GIẢI CHI TIẾT.pdf
ĐỀ THI THỬ TUYỂN SINH VÀO LỚP 10 THPT MÔN TOÁN CÁC TỈNH NĂM HỌC 2023-2024 CÓ LỜI GIẢI CHI TIẾT.pdf
ĐỀ THI THỬ TUYỂN SINH VÀO LỚP 10 THPT MÔN TOÁN CÁC TỈNH NĂM HỌC 2023-2024 CÓ LỜI GIẢI CHI TIẾT.pdf
ĐỀ THI THỬ TUYỂN SINH VÀO LỚP 10 THPT MÔN TOÁN CÁC TỈNH NĂM HỌC 2023-2024 CÓ LỜI GIẢI CHI TIẾT.pdf
ĐỀ THI THỬ TUYỂN SINH VÀO LỚP 10 THPT MÔN TOÁN CÁC TỈNH NĂM HỌC 2023-2024 CÓ LỜI GIẢI CHI TIẾT.pdf
ĐỀ THI THỬ TUYỂN SINH VÀO LỚP 10 THPT MÔN TOÁN CÁC TỈNH NĂM HỌC 2023-2024 CÓ LỜI GIẢI CHI TIẾT.pdf
ĐỀ THI THỬ TUYỂN SINH VÀO LỚP 10 THPT MÔN TOÁN CÁC TỈNH NĂM HỌC 2023-2024 CÓ LỜI GIẢI CHI TIẾT.pdf
ĐỀ THI THỬ TUYỂN SINH VÀO LỚP 10 THPT MÔN TOÁN CÁC TỈNH NĂM HỌC 2023-2024 CÓ LỜI GIẢI CHI TIẾT.pdf
ĐỀ THI THỬ TUYỂN SINH VÀO LỚP 10 THPT MÔN TOÁN CÁC TỈNH NĂM HỌC 2023-2024 CÓ LỜI GIẢI CHI TIẾT.pdf
ĐỀ THI THỬ TUYỂN SINH VÀO LỚP 10 THPT MÔN TOÁN CÁC TỈNH NĂM HỌC 2023-2024 CÓ LỜI GIẢI CHI TIẾT.pdf
ĐỀ THI THỬ TUYỂN SINH VÀO LỚP 10 THPT MÔN TOÁN CÁC TỈNH NĂM HỌC 2023-2024 CÓ LỜI GIẢI CHI TIẾT.pdf
ĐỀ THI THỬ TUYỂN SINH VÀO LỚP 10 THPT MÔN TOÁN CÁC TỈNH NĂM HỌC 2023-2024 CÓ LỜI GIẢI CHI TIẾT.pdf
ĐỀ THI THỬ TUYỂN SINH VÀO LỚP 10 THPT MÔN TOÁN CÁC TỈNH NĂM HỌC 2023-2024 CÓ LỜI GIẢI CHI TIẾT.pdf
ĐỀ THI THỬ TUYỂN SINH VÀO LỚP 10 THPT MÔN TOÁN CÁC TỈNH NĂM HỌC 2023-2024 CÓ LỜI GIẢI CHI TIẾT.pdf
ĐỀ THI THỬ TUYỂN SINH VÀO LỚP 10 THPT MÔN TOÁN CÁC TỈNH NĂM HỌC 2023-2024 CÓ LỜI GIẢI CHI TIẾT.pdf
ĐỀ THI THỬ TUYỂN SINH VÀO LỚP 10 THPT MÔN TOÁN CÁC TỈNH NĂM HỌC 2023-2024 CÓ LỜI GIẢI CHI TIẾT.pdf
ĐỀ THI THỬ TUYỂN SINH VÀO LỚP 10 THPT MÔN TOÁN CÁC TỈNH NĂM HỌC 2023-2024 CÓ LỜI GIẢI CHI TIẾT.pdf
ĐỀ THI THỬ TUYỂN SINH VÀO LỚP 10 THPT MÔN TOÁN CÁC TỈNH NĂM HỌC 2023-2024 CÓ LỜI GIẢI CHI TIẾT.pdf
ĐỀ THI THỬ TUYỂN SINH VÀO LỚP 10 THPT MÔN TOÁN CÁC TỈNH NĂM HỌC 2023-2024 CÓ LỜI GIẢI CHI TIẾT.pdf
ĐỀ THI THỬ TUYỂN SINH VÀO LỚP 10 THPT MÔN TOÁN CÁC TỈNH NĂM HỌC 2023-2024 CÓ LỜI GIẢI CHI TIẾT.pdf
ĐỀ THI THỬ TUYỂN SINH VÀO LỚP 10 THPT MÔN TOÁN CÁC TỈNH NĂM HỌC 2023-2024 CÓ LỜI GIẢI CHI TIẾT.pdf
ĐỀ THI THỬ TUYỂN SINH VÀO LỚP 10 THPT MÔN TOÁN CÁC TỈNH NĂM HỌC 2023-2024 CÓ LỜI GIẢI CHI TIẾT.pdf
ĐỀ THI THỬ TUYỂN SINH VÀO LỚP 10 THPT MÔN TOÁN CÁC TỈNH NĂM HỌC 2023-2024 CÓ LỜI GIẢI CHI TIẾT.pdf
ĐỀ THI THỬ TUYỂN SINH VÀO LỚP 10 THPT MÔN TOÁN CÁC TỈNH NĂM HỌC 2023-2024 CÓ LỜI GIẢI CHI TIẾT.pdf
ĐỀ THI THỬ TUYỂN SINH VÀO LỚP 10 THPT MÔN TOÁN CÁC TỈNH NĂM HỌC 2023-2024 CÓ LỜI GIẢI CHI TIẾT.pdf
ĐỀ THI THỬ TUYỂN SINH VÀO LỚP 10 THPT MÔN TOÁN CÁC TỈNH NĂM HỌC 2023-2024 CÓ LỜI GIẢI CHI TIẾT.pdf
ĐỀ THI THỬ TUYỂN SINH VÀO LỚP 10 THPT MÔN TOÁN CÁC TỈNH NĂM HỌC 2023-2024 CÓ LỜI GIẢI CHI TIẾT.pdf
ĐỀ THI THỬ TUYỂN SINH VÀO LỚP 10 THPT MÔN TOÁN CÁC TỈNH NĂM HỌC 2023-2024 CÓ LỜI GIẢI CHI TIẾT.pdf
ĐỀ THI THỬ TUYỂN SINH VÀO LỚP 10 THPT MÔN TOÁN CÁC TỈNH NĂM HỌC 2023-2024 CÓ LỜI GIẢI CHI TIẾT.pdf
ĐỀ THI THỬ TUYỂN SINH VÀO LỚP 10 THPT MÔN TOÁN CÁC TỈNH NĂM HỌC 2023-2024 CÓ LỜI GIẢI CHI TIẾT.pdf
ĐỀ THI THỬ TUYỂN SINH VÀO LỚP 10 THPT MÔN TOÁN CÁC TỈNH NĂM HỌC 2023-2024 CÓ LỜI GIẢI CHI TIẾT.pdf
ĐỀ THI THỬ TUYỂN SINH VÀO LỚP 10 THPT MÔN TOÁN CÁC TỈNH NĂM HỌC 2023-2024 CÓ LỜI GIẢI CHI TIẾT.pdf
ĐỀ THI THỬ TUYỂN SINH VÀO LỚP 10 THPT MÔN TOÁN CÁC TỈNH NĂM HỌC 2023-2024 CÓ LỜI GIẢI CHI TIẾT.pdf
ĐỀ THI THỬ TUYỂN SINH VÀO LỚP 10 THPT MÔN TOÁN CÁC TỈNH NĂM HỌC 2023-2024 CÓ LỜI GIẢI CHI TIẾT.pdf
ĐỀ THI THỬ TUYỂN SINH VÀO LỚP 10 THPT MÔN TOÁN CÁC TỈNH NĂM HỌC 2023-2024 CÓ LỜI GIẢI CHI TIẾT.pdf
ĐỀ THI THỬ TUYỂN SINH VÀO LỚP 10 THPT MÔN TOÁN CÁC TỈNH NĂM HỌC 2023-2024 CÓ LỜI GIẢI CHI TIẾT.pdf
ĐỀ THI THỬ TUYỂN SINH VÀO LỚP 10 THPT MÔN TOÁN CÁC TỈNH NĂM HỌC 2023-2024 CÓ LỜI GIẢI CHI TIẾT.pdf
ĐỀ THI THỬ TUYỂN SINH VÀO LỚP 10 THPT MÔN TOÁN CÁC TỈNH NĂM HỌC 2023-2024 CÓ LỜI GIẢI CHI TIẾT.pdf
ĐỀ THI THỬ TUYỂN SINH VÀO LỚP 10 THPT MÔN TOÁN CÁC TỈNH NĂM HỌC 2023-2024 CÓ LỜI GIẢI CHI TIẾT.pdf
ĐỀ THI THỬ TUYỂN SINH VÀO LỚP 10 THPT MÔN TOÁN CÁC TỈNH NĂM HỌC 2023-2024 CÓ LỜI GIẢI CHI TIẾT.pdf
ĐỀ THI THỬ TUYỂN SINH VÀO LỚP 10 THPT MÔN TOÁN CÁC TỈNH NĂM HỌC 2023-2024 CÓ LỜI GIẢI CHI TIẾT.pdf
ĐỀ THI THỬ TUYỂN SINH VÀO LỚP 10 THPT MÔN TOÁN CÁC TỈNH NĂM HỌC 2023-2024 CÓ LỜI GIẢI CHI TIẾT.pdf
ĐỀ THI THỬ TUYỂN SINH VÀO LỚP 10 THPT MÔN TOÁN CÁC TỈNH NĂM HỌC 2023-2024 CÓ LỜI GIẢI CHI TIẾT.pdf
ĐỀ THI THỬ TUYỂN SINH VÀO LỚP 10 THPT MÔN TOÁN CÁC TỈNH NĂM HỌC 2023-2024 CÓ LỜI GIẢI CHI TIẾT.pdf
ĐỀ THI THỬ TUYỂN SINH VÀO LỚP 10 THPT MÔN TOÁN CÁC TỈNH NĂM HỌC 2023-2024 CÓ LỜI GIẢI CHI TIẾT.pdf
ĐỀ THI THỬ TUYỂN SINH VÀO LỚP 10 THPT MÔN TOÁN CÁC TỈNH NĂM HỌC 2023-2024 CÓ LỜI GIẢI CHI TIẾT.pdf
ĐỀ THI THỬ TUYỂN SINH VÀO LỚP 10 THPT MÔN TOÁN CÁC TỈNH NĂM HỌC 2023-2024 CÓ LỜI GIẢI CHI TIẾT.pdf
ĐỀ THI THỬ TUYỂN SINH VÀO LỚP 10 THPT MÔN TOÁN CÁC TỈNH NĂM HỌC 2023-2024 CÓ LỜI GIẢI CHI TIẾT.pdf
ĐỀ THI THỬ TUYỂN SINH VÀO LỚP 10 THPT MÔN TOÁN CÁC TỈNH NĂM HỌC 2023-2024 CÓ LỜI GIẢI CHI TIẾT.pdf
ĐỀ THI THỬ TUYỂN SINH VÀO LỚP 10 THPT MÔN TOÁN CÁC TỈNH NĂM HỌC 2023-2024 CÓ LỜI GIẢI CHI TIẾT.pdf
ĐỀ THI THỬ TUYỂN SINH VÀO LỚP 10 THPT MÔN TOÁN CÁC TỈNH NĂM HỌC 2023-2024 CÓ LỜI GIẢI CHI TIẾT.pdf
ĐỀ THI THỬ TUYỂN SINH VÀO LỚP 10 THPT MÔN TOÁN CÁC TỈNH NĂM HỌC 2023-2024 CÓ LỜI GIẢI CHI TIẾT.pdf
ĐỀ THI THỬ TUYỂN SINH VÀO LỚP 10 THPT MÔN TOÁN CÁC TỈNH NĂM HỌC 2023-2024 CÓ LỜI GIẢI CHI TIẾT.pdf
ĐỀ THI THỬ TUYỂN SINH VÀO LỚP 10 THPT MÔN TOÁN CÁC TỈNH NĂM HỌC 2023-2024 CÓ LỜI GIẢI CHI TIẾT.pdf
ĐỀ THI THỬ TUYỂN SINH VÀO LỚP 10 THPT MÔN TOÁN CÁC TỈNH NĂM HỌC 2023-2024 CÓ LỜI GIẢI CHI TIẾT.pdf
ĐỀ THI THỬ TUYỂN SINH VÀO LỚP 10 THPT MÔN TOÁN CÁC TỈNH NĂM HỌC 2023-2024 CÓ LỜI GIẢI CHI TIẾT.pdf
ĐỀ THI THỬ TUYỂN SINH VÀO LỚP 10 THPT MÔN TOÁN CÁC TỈNH NĂM HỌC 2023-2024 CÓ LỜI GIẢI CHI TIẾT.pdf
ĐỀ THI THỬ TUYỂN SINH VÀO LỚP 10 THPT MÔN TOÁN CÁC TỈNH NĂM HỌC 2023-2024 CÓ LỜI GIẢI CHI TIẾT.pdf
ĐỀ THI THỬ TUYỂN SINH VÀO LỚP 10 THPT MÔN TOÁN CÁC TỈNH NĂM HỌC 2023-2024 CÓ LỜI GIẢI CHI TIẾT.pdf
ĐỀ THI THỬ TUYỂN SINH VÀO LỚP 10 THPT MÔN TOÁN CÁC TỈNH NĂM HỌC 2023-2024 CÓ LỜI GIẢI CHI TIẾT.pdf
ĐỀ THI THỬ TUYỂN SINH VÀO LỚP 10 THPT MÔN TOÁN CÁC TỈNH NĂM HỌC 2023-2024 CÓ LỜI GIẢI CHI TIẾT.pdf
ĐỀ THI THỬ TUYỂN SINH VÀO LỚP 10 THPT MÔN TOÁN CÁC TỈNH NĂM HỌC 2023-2024 CÓ LỜI GIẢI CHI TIẾT.pdf
ĐỀ THI THỬ TUYỂN SINH VÀO LỚP 10 THPT MÔN TOÁN CÁC TỈNH NĂM HỌC 2023-2024 CÓ LỜI GIẢI CHI TIẾT.pdf
ĐỀ THI THỬ TUYỂN SINH VÀO LỚP 10 THPT MÔN TOÁN CÁC TỈNH NĂM HỌC 2023-2024 CÓ LỜI GIẢI CHI TIẾT.pdf
ĐỀ THI THỬ TUYỂN SINH VÀO LỚP 10 THPT MÔN TOÁN CÁC TỈNH NĂM HỌC 2023-2024 CÓ LỜI GIẢI CHI TIẾT.pdf
ĐỀ THI THỬ TUYỂN SINH VÀO LỚP 10 THPT MÔN TOÁN CÁC TỈNH NĂM HỌC 2023-2024 CÓ LỜI GIẢI CHI TIẾT.pdf
ĐỀ THI THỬ TUYỂN SINH VÀO LỚP 10 THPT MÔN TOÁN CÁC TỈNH NĂM HỌC 2023-2024 CÓ LỜI GIẢI CHI TIẾT.pdf
ĐỀ THI THỬ TUYỂN SINH VÀO LỚP 10 THPT MÔN TOÁN CÁC TỈNH NĂM HỌC 2023-2024 CÓ LỜI GIẢI CHI TIẾT.pdf
ĐỀ THI THỬ TUYỂN SINH VÀO LỚP 10 THPT MÔN TOÁN CÁC TỈNH NĂM HỌC 2023-2024 CÓ LỜI GIẢI CHI TIẾT.pdf
ĐỀ THI THỬ TUYỂN SINH VÀO LỚP 10 THPT MÔN TOÁN CÁC TỈNH NĂM HỌC 2023-2024 CÓ LỜI GIẢI CHI TIẾT.pdf
ĐỀ THI THỬ TUYỂN SINH VÀO LỚP 10 THPT MÔN TOÁN CÁC TỈNH NĂM HỌC 2023-2024 CÓ LỜI GIẢI CHI TIẾT.pdf
ĐỀ THI THỬ TUYỂN SINH VÀO LỚP 10 THPT MÔN TOÁN CÁC TỈNH NĂM HỌC 2023-2024 CÓ LỜI GIẢI CHI TIẾT.pdf
ĐỀ THI THỬ TUYỂN SINH VÀO LỚP 10 THPT MÔN TOÁN CÁC TỈNH NĂM HỌC 2023-2024 CÓ LỜI GIẢI CHI TIẾT.pdf
ĐỀ THI THỬ TUYỂN SINH VÀO LỚP 10 THPT MÔN TOÁN CÁC TỈNH NĂM HỌC 2023-2024 CÓ LỜI GIẢI CHI TIẾT.pdf
ĐỀ THI THỬ TUYỂN SINH VÀO LỚP 10 THPT MÔN TOÁN CÁC TỈNH NĂM HỌC 2023-2024 CÓ LỜI GIẢI CHI TIẾT.pdf
ĐỀ THI THỬ TUYỂN SINH VÀO LỚP 10 THPT MÔN TOÁN CÁC TỈNH NĂM HỌC 2023-2024 CÓ LỜI GIẢI CHI TIẾT.pdf
ĐỀ THI THỬ TUYỂN SINH VÀO LỚP 10 THPT MÔN TOÁN CÁC TỈNH NĂM HỌC 2023-2024 CÓ LỜI GIẢI CHI TIẾT.pdf
ĐỀ THI THỬ TUYỂN SINH VÀO LỚP 10 THPT MÔN TOÁN CÁC TỈNH NĂM HỌC 2023-2024 CÓ LỜI GIẢI CHI TIẾT.pdf
ĐỀ THI THỬ TUYỂN SINH VÀO LỚP 10 THPT MÔN TOÁN CÁC TỈNH NĂM HỌC 2023-2024 CÓ LỜI GIẢI CHI TIẾT.pdf
ĐỀ THI THỬ TUYỂN SINH VÀO LỚP 10 THPT MÔN TOÁN CÁC TỈNH NĂM HỌC 2023-2024 CÓ LỜI GIẢI CHI TIẾT.pdf
ĐỀ THI THỬ TUYỂN SINH VÀO LỚP 10 THPT MÔN TOÁN CÁC TỈNH NĂM HỌC 2023-2024 CÓ LỜI GIẢI CHI TIẾT.pdf
ĐỀ THI THỬ TUYỂN SINH VÀO LỚP 10 THPT MÔN TOÁN CÁC TỈNH NĂM HỌC 2023-2024 CÓ LỜI GIẢI CHI TIẾT.pdf
ĐỀ THI THỬ TUYỂN SINH VÀO LỚP 10 THPT MÔN TOÁN CÁC TỈNH NĂM HỌC 2023-2024 CÓ LỜI GIẢI CHI TIẾT.pdf
ĐỀ THI THỬ TUYỂN SINH VÀO LỚP 10 THPT MÔN TOÁN CÁC TỈNH NĂM HỌC 2023-2024 CÓ LỜI GIẢI CHI TIẾT.pdf
ĐỀ THI THỬ TUYỂN SINH VÀO LỚP 10 THPT MÔN TOÁN CÁC TỈNH NĂM HỌC 2023-2024 CÓ LỜI GIẢI CHI TIẾT.pdf
ĐỀ THI THỬ TUYỂN SINH VÀO LỚP 10 THPT MÔN TOÁN CÁC TỈNH NĂM HỌC 2023-2024 CÓ LỜI GIẢI CHI TIẾT.pdf
ĐỀ THI THỬ TUYỂN SINH VÀO LỚP 10 THPT MÔN TOÁN CÁC TỈNH NĂM HỌC 2023-2024 CÓ LỜI GIẢI CHI TIẾT.pdf
ĐỀ THI THỬ TUYỂN SINH VÀO LỚP 10 THPT MÔN TOÁN CÁC TỈNH NĂM HỌC 2023-2024 CÓ LỜI GIẢI CHI TIẾT.pdf
ĐỀ THI THỬ TUYỂN SINH VÀO LỚP 10 THPT MÔN TOÁN CÁC TỈNH NĂM HỌC 2023-2024 CÓ LỜI GIẢI CHI TIẾT.pdf
ĐỀ THI THỬ TUYỂN SINH VÀO LỚP 10 THPT MÔN TOÁN CÁC TỈNH NĂM HỌC 2023-2024 CÓ LỜI GIẢI CHI TIẾT.pdf
ĐỀ THI THỬ TUYỂN SINH VÀO LỚP 10 THPT MÔN TOÁN CÁC TỈNH NĂM HỌC 2023-2024 CÓ LỜI GIẢI CHI TIẾT.pdf
ĐỀ THI THỬ TUYỂN SINH VÀO LỚP 10 THPT MÔN TOÁN CÁC TỈNH NĂM HỌC 2023-2024 CÓ LỜI GIẢI CHI TIẾT.pdf
ĐỀ THI THỬ TUYỂN SINH VÀO LỚP 10 THPT MÔN TOÁN CÁC TỈNH NĂM HỌC 2023-2024 CÓ LỜI GIẢI CHI TIẾT.pdf
ĐỀ THI THỬ TUYỂN SINH VÀO LỚP 10 THPT MÔN TOÁN CÁC TỈNH NĂM HỌC 2023-2024 CÓ LỜI GIẢI CHI TIẾT.pdf
ĐỀ THI THỬ TUYỂN SINH VÀO LỚP 10 THPT MÔN TOÁN CÁC TỈNH NĂM HỌC 2023-2024 CÓ LỜI GIẢI CHI TIẾT.pdf
ĐỀ THI THỬ TUYỂN SINH VÀO LỚP 10 THPT MÔN TOÁN CÁC TỈNH NĂM HỌC 2023-2024 CÓ LỜI GIẢI CHI TIẾT.pdf
ĐỀ THI THỬ TUYỂN SINH VÀO LỚP 10 THPT MÔN TOÁN CÁC TỈNH NĂM HỌC 2023-2024 CÓ LỜI GIẢI CHI TIẾT.pdf
ĐỀ THI THỬ TUYỂN SINH VÀO LỚP 10 THPT MÔN TOÁN CÁC TỈNH NĂM HỌC 2023-2024 CÓ LỜI GIẢI CHI TIẾT.pdf
ĐỀ THI THỬ TUYỂN SINH VÀO LỚP 10 THPT MÔN TOÁN CÁC TỈNH NĂM HỌC 2023-2024 CÓ LỜI GIẢI CHI TIẾT.pdf
ĐỀ THI THỬ TUYỂN SINH VÀO LỚP 10 THPT MÔN TOÁN CÁC TỈNH NĂM HỌC 2023-2024 CÓ LỜI GIẢI CHI TIẾT.pdf
ĐỀ THI THỬ TUYỂN SINH VÀO LỚP 10 THPT MÔN TOÁN CÁC TỈNH NĂM HỌC 2023-2024 CÓ LỜI GIẢI CHI TIẾT.pdf
ĐỀ THI THỬ TUYỂN SINH VÀO LỚP 10 THPT MÔN TOÁN CÁC TỈNH NĂM HỌC 2023-2024 CÓ LỜI GIẢI CHI TIẾT.pdf
ĐỀ THI THỬ TUYỂN SINH VÀO LỚP 10 THPT MÔN TOÁN CÁC TỈNH NĂM HỌC 2023-2024 CÓ LỜI GIẢI CHI TIẾT.pdf
ĐỀ THI THỬ TUYỂN SINH VÀO LỚP 10 THPT MÔN TOÁN CÁC TỈNH NĂM HỌC 2023-2024 CÓ LỜI GIẢI CHI TIẾT.pdf
ĐỀ THI THỬ TUYỂN SINH VÀO LỚP 10 THPT MÔN TOÁN CÁC TỈNH NĂM HỌC 2023-2024 CÓ LỜI GIẢI CHI TIẾT.pdf
ĐỀ THI THỬ TUYỂN SINH VÀO LỚP 10 THPT MÔN TOÁN CÁC TỈNH NĂM HỌC 2023-2024 CÓ LỜI GIẢI CHI TIẾT.pdf
ĐỀ THI THỬ TUYỂN SINH VÀO LỚP 10 THPT MÔN TOÁN CÁC TỈNH NĂM HỌC 2023-2024 CÓ LỜI GIẢI CHI TIẾT.pdf
ĐỀ THI THỬ TUYỂN SINH VÀO LỚP 10 THPT MÔN TOÁN CÁC TỈNH NĂM HỌC 2023-2024 CÓ LỜI GIẢI CHI TIẾT.pdf
ĐỀ THI THỬ TUYỂN SINH VÀO LỚP 10 THPT MÔN TOÁN CÁC TỈNH NĂM HỌC 2023-2024 CÓ LỜI GIẢI CHI TIẾT.pdf
ĐỀ THI THỬ TUYỂN SINH VÀO LỚP 10 THPT MÔN TOÁN CÁC TỈNH NĂM HỌC 2023-2024 CÓ LỜI GIẢI CHI TIẾT.pdf
ĐỀ THI THỬ TUYỂN SINH VÀO LỚP 10 THPT MÔN TOÁN CÁC TỈNH NĂM HỌC 2023-2024 CÓ LỜI GIẢI CHI TIẾT.pdf
ĐỀ THI THỬ TUYỂN SINH VÀO LỚP 10 THPT MÔN TOÁN CÁC TỈNH NĂM HỌC 2023-2024 CÓ LỜI GIẢI CHI TIẾT.pdf
ĐỀ THI THỬ TUYỂN SINH VÀO LỚP 10 THPT MÔN TOÁN CÁC TỈNH NĂM HỌC 2023-2024 CÓ LỜI GIẢI CHI TIẾT.pdf
ĐỀ THI THỬ TUYỂN SINH VÀO LỚP 10 THPT MÔN TOÁN CÁC TỈNH NĂM HỌC 2023-2024 CÓ LỜI GIẢI CHI TIẾT.pdf
ĐỀ THI THỬ TUYỂN SINH VÀO LỚP 10 THPT MÔN TOÁN CÁC TỈNH NĂM HỌC 2023-2024 CÓ LỜI GIẢI CHI TIẾT.pdf
ĐỀ THI THỬ TUYỂN SINH VÀO LỚP 10 THPT MÔN TOÁN CÁC TỈNH NĂM HỌC 2023-2024 CÓ LỜI GIẢI CHI TIẾT.pdf
ĐỀ THI THỬ TUYỂN SINH VÀO LỚP 10 THPT MÔN TOÁN CÁC TỈNH NĂM HỌC 2023-2024 CÓ LỜI GIẢI CHI TIẾT.pdf
ĐỀ THI THỬ TUYỂN SINH VÀO LỚP 10 THPT MÔN TOÁN CÁC TỈNH NĂM HỌC 2023-2024 CÓ LỜI GIẢI CHI TIẾT.pdf
ĐỀ THI THỬ TUYỂN SINH VÀO LỚP 10 THPT MÔN TOÁN CÁC TỈNH NĂM HỌC 2023-2024 CÓ LỜI GIẢI CHI TIẾT.pdf
ĐỀ THI THỬ TUYỂN SINH VÀO LỚP 10 THPT MÔN TOÁN CÁC TỈNH NĂM HỌC 2023-2024 CÓ LỜI GIẢI CHI TIẾT.pdf
ĐỀ THI THỬ TUYỂN SINH VÀO LỚP 10 THPT MÔN TOÁN CÁC TỈNH NĂM HỌC 2023-2024 CÓ LỜI GIẢI CHI TIẾT.pdf
ĐỀ THI THỬ TUYỂN SINH VÀO LỚP 10 THPT MÔN TOÁN CÁC TỈNH NĂM HỌC 2023-2024 CÓ LỜI GIẢI CHI TIẾT.pdf
ĐỀ THI THỬ TUYỂN SINH VÀO LỚP 10 THPT MÔN TOÁN CÁC TỈNH NĂM HỌC 2023-2024 CÓ LỜI GIẢI CHI TIẾT.pdf
ĐỀ THI THỬ TUYỂN SINH VÀO LỚP 10 THPT MÔN TOÁN CÁC TỈNH NĂM HỌC 2023-2024 CÓ LỜI GIẢI CHI TIẾT.pdf
ĐỀ THI THỬ TUYỂN SINH VÀO LỚP 10 THPT MÔN TOÁN CÁC TỈNH NĂM HỌC 2023-2024 CÓ LỜI GIẢI CHI TIẾT.pdf
ĐỀ THI THỬ TUYỂN SINH VÀO LỚP 10 THPT MÔN TOÁN CÁC TỈNH NĂM HỌC 2023-2024 CÓ LỜI GIẢI CHI TIẾT.pdf
ĐỀ THI THỬ TUYỂN SINH VÀO LỚP 10 THPT MÔN TOÁN CÁC TỈNH NĂM HỌC 2023-2024 CÓ LỜI GIẢI CHI TIẾT.pdf
ĐỀ THI THỬ TUYỂN SINH VÀO LỚP 10 THPT MÔN TOÁN CÁC TỈNH NĂM HỌC 2023-2024 CÓ LỜI GIẢI CHI TIẾT.pdf
ĐỀ THI THỬ TUYỂN SINH VÀO LỚP 10 THPT MÔN TOÁN CÁC TỈNH NĂM HỌC 2023-2024 CÓ LỜI GIẢI CHI TIẾT.pdf
ĐỀ THI THỬ TUYỂN SINH VÀO LỚP 10 THPT MÔN TOÁN CÁC TỈNH NĂM HỌC 2023-2024 CÓ LỜI GIẢI CHI TIẾT.pdf
ĐỀ THI THỬ TUYỂN SINH VÀO LỚP 10 THPT MÔN TOÁN CÁC TỈNH NĂM HỌC 2023-2024 CÓ LỜI GIẢI CHI TIẾT.pdf
ĐỀ THI THỬ TUYỂN SINH VÀO LỚP 10 THPT MÔN TOÁN CÁC TỈNH NĂM HỌC 2023-2024 CÓ LỜI GIẢI CHI TIẾT.pdf
ĐỀ THI THỬ TUYỂN SINH VÀO LỚP 10 THPT MÔN TOÁN CÁC TỈNH NĂM HỌC 2023-2024 CÓ LỜI GIẢI CHI TIẾT.pdf
ĐỀ THI THỬ TUYỂN SINH VÀO LỚP 10 THPT MÔN TOÁN CÁC TỈNH NĂM HỌC 2023-2024 CÓ LỜI GIẢI CHI TIẾT.pdf
ĐỀ THI THỬ TUYỂN SINH VÀO LỚP 10 THPT MÔN TOÁN CÁC TỈNH NĂM HỌC 2023-2024 CÓ LỜI GIẢI CHI TIẾT.pdf
ĐỀ THI THỬ TUYỂN SINH VÀO LỚP 10 THPT MÔN TOÁN CÁC TỈNH NĂM HỌC 2023-2024 CÓ LỜI GIẢI CHI TIẾT.pdf
ĐỀ THI THỬ TUYỂN SINH VÀO LỚP 10 THPT MÔN TOÁN CÁC TỈNH NĂM HỌC 2023-2024 CÓ LỜI GIẢI CHI TIẾT.pdf
ĐỀ THI THỬ TUYỂN SINH VÀO LỚP 10 THPT MÔN TOÁN CÁC TỈNH NĂM HỌC 2023-2024 CÓ LỜI GIẢI CHI TIẾT.pdf
ĐỀ THI THỬ TUYỂN SINH VÀO LỚP 10 THPT MÔN TOÁN CÁC TỈNH NĂM HỌC 2023-2024 CÓ LỜI GIẢI CHI TIẾT.pdf
ĐỀ THI THỬ TUYỂN SINH VÀO LỚP 10 THPT MÔN TOÁN CÁC TỈNH NĂM HỌC 2023-2024 CÓ LỜI GIẢI CHI TIẾT.pdf
ĐỀ THI THỬ TUYỂN SINH VÀO LỚP 10 THPT MÔN TOÁN CÁC TỈNH NĂM HỌC 2023-2024 CÓ LỜI GIẢI CHI TIẾT.pdf
ĐỀ THI THỬ TUYỂN SINH VÀO LỚP 10 THPT MÔN TOÁN CÁC TỈNH NĂM HỌC 2023-2024 CÓ LỜI GIẢI CHI TIẾT.pdf
ĐỀ THI THỬ TUYỂN SINH VÀO LỚP 10 THPT MÔN TOÁN CÁC TỈNH NĂM HỌC 2023-2024 CÓ LỜI GIẢI CHI TIẾT.pdf
ĐỀ THI THỬ TUYỂN SINH VÀO LỚP 10 THPT MÔN TOÁN CÁC TỈNH NĂM HỌC 2023-2024 CÓ LỜI GIẢI CHI TIẾT.pdf
ĐỀ THI THỬ TUYỂN SINH VÀO LỚP 10 THPT MÔN TOÁN CÁC TỈNH NĂM HỌC 2023-2024 CÓ LỜI GIẢI CHI TIẾT.pdf
ĐỀ THI THỬ TUYỂN SINH VÀO LỚP 10 THPT MÔN TOÁN CÁC TỈNH NĂM HỌC 2023-2024 CÓ LỜI GIẢI CHI TIẾT.pdf
ĐỀ THI THỬ TUYỂN SINH VÀO LỚP 10 THPT MÔN TOÁN CÁC TỈNH NĂM HỌC 2023-2024 CÓ LỜI GIẢI CHI TIẾT.pdf
ĐỀ THI THỬ TUYỂN SINH VÀO LỚP 10 THPT MÔN TOÁN CÁC TỈNH NĂM HỌC 2023-2024 CÓ LỜI GIẢI CHI TIẾT.pdf
ĐỀ THI THỬ TUYỂN SINH VÀO LỚP 10 THPT MÔN TOÁN CÁC TỈNH NĂM HỌC 2023-2024 CÓ LỜI GIẢI CHI TIẾT.pdf
ĐỀ THI THỬ TUYỂN SINH VÀO LỚP 10 THPT MÔN TOÁN CÁC TỈNH NĂM HỌC 2023-2024 CÓ LỜI GIẢI CHI TIẾT.pdf
ĐỀ THI THỬ TUYỂN SINH VÀO LỚP 10 THPT MÔN TOÁN CÁC TỈNH NĂM HỌC 2023-2024 CÓ LỜI GIẢI CHI TIẾT.pdf
ĐỀ THI THỬ TUYỂN SINH VÀO LỚP 10 THPT MÔN TOÁN CÁC TỈNH NĂM HỌC 2023-2024 CÓ LỜI GIẢI CHI TIẾT.pdf
ĐỀ THI THỬ TUYỂN SINH VÀO LỚP 10 THPT MÔN TOÁN CÁC TỈNH NĂM HỌC 2023-2024 CÓ LỜI GIẢI CHI TIẾT.pdf
ĐỀ THI THỬ TUYỂN SINH VÀO LỚP 10 THPT MÔN TOÁN CÁC TỈNH NĂM HỌC 2023-2024 CÓ LỜI GIẢI CHI TIẾT.pdf
ĐỀ THI THỬ TUYỂN SINH VÀO LỚP 10 THPT MÔN TOÁN CÁC TỈNH NĂM HỌC 2023-2024 CÓ LỜI GIẢI CHI TIẾT.pdf
ĐỀ THI THỬ TUYỂN SINH VÀO LỚP 10 THPT MÔN TOÁN CÁC TỈNH NĂM HỌC 2023-2024 CÓ LỜI GIẢI CHI TIẾT.pdf
ĐỀ THI THỬ TUYỂN SINH VÀO LỚP 10 THPT MÔN TOÁN CÁC TỈNH NĂM HỌC 2023-2024 CÓ LỜI GIẢI CHI TIẾT.pdf
ĐỀ THI THỬ TUYỂN SINH VÀO LỚP 10 THPT MÔN TOÁN CÁC TỈNH NĂM HỌC 2023-2024 CÓ LỜI GIẢI CHI TIẾT.pdf
ĐỀ THI THỬ TUYỂN SINH VÀO LỚP 10 THPT MÔN TOÁN CÁC TỈNH NĂM HỌC 2023-2024 CÓ LỜI GIẢI CHI TIẾT.pdf
ĐỀ THI THỬ TUYỂN SINH VÀO LỚP 10 THPT MÔN TOÁN CÁC TỈNH NĂM HỌC 2023-2024 CÓ LỜI GIẢI CHI TIẾT.pdf
ĐỀ THI THỬ TUYỂN SINH VÀO LỚP 10 THPT MÔN TOÁN CÁC TỈNH NĂM HỌC 2023-2024 CÓ LỜI GIẢI CHI TIẾT.pdf
ĐỀ THI THỬ TUYỂN SINH VÀO LỚP 10 THPT MÔN TOÁN CÁC TỈNH NĂM HỌC 2023-2024 CÓ LỜI GIẢI CHI TIẾT.pdf

More Related Content

What's hot

20 ĐỀ ĐỀ XUẤT THI VÀO 10 THPT THÀNH PHỐ HẢI PHÒNG MÔN TIẾNG ANH 9 NĂM HỌC 202...
20 ĐỀ ĐỀ XUẤT THI VÀO 10 THPT THÀNH PHỐ HẢI PHÒNG MÔN TIẾNG ANH 9 NĂM HỌC 202...20 ĐỀ ĐỀ XUẤT THI VÀO 10 THPT THÀNH PHỐ HẢI PHÒNG MÔN TIẾNG ANH 9 NĂM HỌC 202...
20 ĐỀ ĐỀ XUẤT THI VÀO 10 THPT THÀNH PHỐ HẢI PHÒNG MÔN TIẾNG ANH 9 NĂM HỌC 202...
Nguyen Thanh Tu Collection
 
BÀI TẬP DẠY THÊM TOÁN 11 - SÁCH CÁNH DIỀU - CẢ NĂM - CHUYÊN ĐỀ 1 - HÀM SỐ LƯỢ...
BÀI TẬP DẠY THÊM TOÁN 11 - SÁCH CÁNH DIỀU - CẢ NĂM - CHUYÊN ĐỀ 1 - HÀM SỐ LƯỢ...BÀI TẬP DẠY THÊM TOÁN 11 - SÁCH CÁNH DIỀU - CẢ NĂM - CHUYÊN ĐỀ 1 - HÀM SỐ LƯỢ...
BÀI TẬP DẠY THÊM TOÁN 11 - SÁCH CÁNH DIỀU - CẢ NĂM - CHUYÊN ĐỀ 1 - HÀM SỐ LƯỢ...
Nguyen Thanh Tu Collection
 
TÀI LIỆU DẠY THÊM HÓA HỌC 11 THEO MỨC ĐỘ SÁCH MỚI - KẾT NỐI TRI THỨC, CÁNH DI...
TÀI LIỆU DẠY THÊM HÓA HỌC 11 THEO MỨC ĐỘ SÁCH MỚI - KẾT NỐI TRI THỨC, CÁNH DI...TÀI LIỆU DẠY THÊM HÓA HỌC 11 THEO MỨC ĐỘ SÁCH MỚI - KẾT NỐI TRI THỨC, CÁNH DI...
TÀI LIỆU DẠY THÊM HÓA HỌC 11 THEO MỨC ĐỘ SÁCH MỚI - KẾT NỐI TRI THỨC, CÁNH DI...
Nguyen Thanh Tu Collection
 
ĐỀ THAM KHẢO THEO HƯỚNG MINH HỌA 2025 KIỂM TRA GIỮA HỌC KÌ 2 NĂM HỌC 2023-202...
ĐỀ THAM KHẢO THEO HƯỚNG MINH HỌA 2025 KIỂM TRA GIỮA HỌC KÌ 2 NĂM HỌC 2023-202...ĐỀ THAM KHẢO THEO HƯỚNG MINH HỌA 2025 KIỂM TRA GIỮA HỌC KÌ 2 NĂM HỌC 2023-202...
ĐỀ THAM KHẢO THEO HƯỚNG MINH HỌA 2025 KIỂM TRA GIỮA HỌC KÌ 2 NĂM HỌC 2023-202...
Nguyen Thanh Tu Collection
 
TỔNG HỢP ĐỀ THAM KHẢO TUYỂN SINH 10 NĂM HỌC 2024 - 2025 MÔN TOÁN 9 SỞ GIÁO DỤ...
TỔNG HỢP ĐỀ THAM KHẢO TUYỂN SINH 10 NĂM HỌC 2024 - 2025 MÔN TOÁN 9 SỞ GIÁO DỤ...TỔNG HỢP ĐỀ THAM KHẢO TUYỂN SINH 10 NĂM HỌC 2024 - 2025 MÔN TOÁN 9 SỞ GIÁO DỤ...
TỔNG HỢP ĐỀ THAM KHẢO TUYỂN SINH 10 NĂM HỌC 2024 - 2025 MÔN TOÁN 9 SỞ GIÁO DỤ...
Nguyen Thanh Tu Collection
 
20 ĐỀ DỰ ĐOÁN - PHÁT TRIỂN ĐỀ MINH HỌA BGD KỲ THI TỐT NGHIỆP THPT NĂM 2023 MÔ...
20 ĐỀ DỰ ĐOÁN - PHÁT TRIỂN ĐỀ MINH HỌA BGD KỲ THI TỐT NGHIỆP THPT NĂM 2023 MÔ...20 ĐỀ DỰ ĐOÁN - PHÁT TRIỂN ĐỀ MINH HỌA BGD KỲ THI TỐT NGHIỆP THPT NĂM 2023 MÔ...
20 ĐỀ DỰ ĐOÁN - PHÁT TRIỂN ĐỀ MINH HỌA BGD KỲ THI TỐT NGHIỆP THPT NĂM 2023 MÔ...
Nguyen Thanh Tu Collection
 
ĐỀ ÔN TẬP THAM KHẢO THI THỬ GIỮA HỌC KÌ 2 HÓA HỌC 11 - DÙNG CHUNG 3 SÁCH - TH...
ĐỀ ÔN TẬP THAM KHẢO THI THỬ GIỮA HỌC KÌ 2 HÓA HỌC 11 - DÙNG CHUNG 3 SÁCH - TH...ĐỀ ÔN TẬP THAM KHẢO THI THỬ GIỮA HỌC KÌ 2 HÓA HỌC 11 - DÙNG CHUNG 3 SÁCH - TH...
ĐỀ ÔN TẬP THAM KHẢO THI THỬ GIỮA HỌC KÌ 2 HÓA HỌC 11 - DÙNG CHUNG 3 SÁCH - TH...
Nguyen Thanh Tu Collection
 
TÀI LIỆU DẠY THÊM ĐẦY ĐỦ TOÁN 11 NĂM 2024 - HỌC KÌ 1 (KẾT NỐI TRI THỨC) - LÝ ...
TÀI LIỆU DẠY THÊM ĐẦY ĐỦ TOÁN 11 NĂM 2024 - HỌC KÌ 1 (KẾT NỐI TRI THỨC) - LÝ ...TÀI LIỆU DẠY THÊM ĐẦY ĐỦ TOÁN 11 NĂM 2024 - HỌC KÌ 1 (KẾT NỐI TRI THỨC) - LÝ ...
TÀI LIỆU DẠY THÊM ĐẦY ĐỦ TOÁN 11 NĂM 2024 - HỌC KÌ 1 (KẾT NỐI TRI THỨC) - LÝ ...
Nguyen Thanh Tu Collection
 
10 ĐỀ KIỂM TRA + 6 ĐỀ ÔN TẬP CUỐI KÌ 2 VẬT LÝ 11 - KẾT NỐI TRI THỨC - THEO C...
10 ĐỀ KIỂM TRA + 6 ĐỀ ÔN TẬP CUỐI KÌ 2 VẬT LÝ 11 - KẾT NỐI TRI THỨC - THEO C...10 ĐỀ KIỂM TRA + 6 ĐỀ ÔN TẬP CUỐI KÌ 2 VẬT LÝ 11 - KẾT NỐI TRI THỨC - THEO C...
10 ĐỀ KIỂM TRA + 6 ĐỀ ÔN TẬP CUỐI KÌ 2 VẬT LÝ 11 - KẾT NỐI TRI THỨC - THEO C...
Nguyen Thanh Tu Collection
 
ĐỀ THI THỬ GIỮA HỌC KÌ 1 MÔN TIẾNG ANH I-LEARN SMART WORLD 11 - NĂM HỌC 2023-...
ĐỀ THI THỬ GIỮA HỌC KÌ 1 MÔN TIẾNG ANH I-LEARN SMART WORLD 11 - NĂM HỌC 2023-...ĐỀ THI THỬ GIỮA HỌC KÌ 1 MÔN TIẾNG ANH I-LEARN SMART WORLD 11 - NĂM HỌC 2023-...
ĐỀ THI THỬ GIỮA HỌC KÌ 1 MÔN TIẾNG ANH I-LEARN SMART WORLD 11 - NĂM HỌC 2023-...
Nguyen Thanh Tu Collection
 
CHUYÊN ĐỀ DẠY THÊM HÓA HỌC 11 - SÁCH MỚI NĂM 2023 (CHƯƠNG 1 CÂN BẰNG HÓA HỌC)...
CHUYÊN ĐỀ DẠY THÊM HÓA HỌC 11 - SÁCH MỚI NĂM 2023 (CHƯƠNG 1 CÂN BẰNG HÓA HỌC)...CHUYÊN ĐỀ DẠY THÊM HÓA HỌC 11 - SÁCH MỚI NĂM 2023 (CHƯƠNG 1 CÂN BẰNG HÓA HỌC)...
CHUYÊN ĐỀ DẠY THÊM HÓA HỌC 11 - SÁCH MỚI NĂM 2023 (CHƯƠNG 1 CÂN BẰNG HÓA HỌC)...
Nguyen Thanh Tu Collection
 
BÀI TẬP DẠY THÊM TOÁN 11 - CHÂN TRỜI SÁNG TẠO - CẢ NĂM - CHƯƠNG 4 - ĐƯỜNG THẲ...
BÀI TẬP DẠY THÊM TOÁN 11 - CHÂN TRỜI SÁNG TẠO - CẢ NĂM - CHƯƠNG 4 - ĐƯỜNG THẲ...BÀI TẬP DẠY THÊM TOÁN 11 - CHÂN TRỜI SÁNG TẠO - CẢ NĂM - CHƯƠNG 4 - ĐƯỜNG THẲ...
BÀI TẬP DẠY THÊM TOÁN 11 - CHÂN TRỜI SÁNG TẠO - CẢ NĂM - CHƯƠNG 4 - ĐƯỜNG THẲ...
Nguyen Thanh Tu Collection
 
BẤT ĐẲNG THỨC LUYỆN THI VÀO LỚP 10 CHUYÊN 2023-2024 (15 BẤT ĐẲNG THỨC VÀO LỚP...
BẤT ĐẲNG THỨC LUYỆN THI VÀO LỚP 10 CHUYÊN 2023-2024 (15 BẤT ĐẲNG THỨC VÀO LỚP...BẤT ĐẲNG THỨC LUYỆN THI VÀO LỚP 10 CHUYÊN 2023-2024 (15 BẤT ĐẲNG THỨC VÀO LỚP...
BẤT ĐẲNG THỨC LUYỆN THI VÀO LỚP 10 CHUYÊN 2023-2024 (15 BẤT ĐẲNG THỨC VÀO LỚP...
Nguyen Thanh Tu Collection
 
BỘ CÂU HỎI ÔN TẬP KIỂM TRA VÀ ĐÁNH GIÁ MÔN HÓA HỌC 11 (KẾT NỐI TRI THỨC) - TÀ...
BỘ CÂU HỎI ÔN TẬP KIỂM TRA VÀ ĐÁNH GIÁ MÔN HÓA HỌC 11 (KẾT NỐI TRI THỨC) - TÀ...BỘ CÂU HỎI ÔN TẬP KIỂM TRA VÀ ĐÁNH GIÁ MÔN HÓA HỌC 11 (KẾT NỐI TRI THỨC) - TÀ...
BỘ CÂU HỎI ÔN TẬP KIỂM TRA VÀ ĐÁNH GIÁ MÔN HÓA HỌC 11 (KẾT NỐI TRI THỨC) - TÀ...
Nguyen Thanh Tu Collection
 
BÀI TẬP DẠY THÊM TOÁN 11 - CHÂN TRỜI SÁNG TẠO - CẢ NĂM - CHƯƠNG 1 - HÀM SỐ LƯ...
BÀI TẬP DẠY THÊM TOÁN 11 - CHÂN TRỜI SÁNG TẠO - CẢ NĂM - CHƯƠNG 1 - HÀM SỐ LƯ...BÀI TẬP DẠY THÊM TOÁN 11 - CHÂN TRỜI SÁNG TẠO - CẢ NĂM - CHƯƠNG 1 - HÀM SỐ LƯ...
BÀI TẬP DẠY THÊM TOÁN 11 - CHÂN TRỜI SÁNG TẠO - CẢ NĂM - CHƯƠNG 1 - HÀM SỐ LƯ...
Nguyen Thanh Tu Collection
 
15 ĐỀ THI THỬ TUYỂN SINH VÀO LỚP 10 MÔN TIẾNG ANH NĂM HỌC 2023 - 2024 SỞ GIÁO...
15 ĐỀ THI THỬ TUYỂN SINH VÀO LỚP 10 MÔN TIẾNG ANH NĂM HỌC 2023 - 2024 SỞ GIÁO...15 ĐỀ THI THỬ TUYỂN SINH VÀO LỚP 10 MÔN TIẾNG ANH NĂM HỌC 2023 - 2024 SỞ GIÁO...
15 ĐỀ THI THỬ TUYỂN SINH VÀO LỚP 10 MÔN TIẾNG ANH NĂM HỌC 2023 - 2024 SỞ GIÁO...
Nguyen Thanh Tu Collection
 
TÓM TẮT LÝ THUYẾT + BÀI TẬP TRẮC NGHIỆM TỪNG BÀI HỌC THEO 4 CẤP ĐỘ HÓA HỌC LỚ...
TÓM TẮT LÝ THUYẾT + BÀI TẬP TRẮC NGHIỆM TỪNG BÀI HỌC THEO 4 CẤP ĐỘ HÓA HỌC LỚ...TÓM TẮT LÝ THUYẾT + BÀI TẬP TRẮC NGHIỆM TỪNG BÀI HỌC THEO 4 CẤP ĐỘ HÓA HỌC LỚ...
TÓM TẮT LÝ THUYẾT + BÀI TẬP TRẮC NGHIỆM TỪNG BÀI HỌC THEO 4 CẤP ĐỘ HÓA HỌC LỚ...
Nguyen Thanh Tu Collection
 
GIÁO ÁN KẾ HOẠCH BÀI DẠY (SGK + CHUYÊN ĐỀ) HÓA HỌC 11 KẾT NỐI TRI THỨC THEO C...
GIÁO ÁN KẾ HOẠCH BÀI DẠY (SGK + CHUYÊN ĐỀ) HÓA HỌC 11 KẾT NỐI TRI THỨC THEO C...GIÁO ÁN KẾ HOẠCH BÀI DẠY (SGK + CHUYÊN ĐỀ) HÓA HỌC 11 KẾT NỐI TRI THỨC THEO C...
GIÁO ÁN KẾ HOẠCH BÀI DẠY (SGK + CHUYÊN ĐỀ) HÓA HỌC 11 KẾT NỐI TRI THỨC THEO C...
Nguyen Thanh Tu Collection
 
Đề Thi HK2 Toán 8 - THCS Nguyễn Văn Luông
Đề Thi HK2 Toán 8 - THCS Nguyễn Văn LuôngĐề Thi HK2 Toán 8 - THCS Nguyễn Văn Luông
Đề Thi HK2 Toán 8 - THCS Nguyễn Văn Luông
Công Ty TNHH VIETTRIGROUP
 
BÀI TẬP DẠY THÊM TOÁN 11 - SÁCH KẾT NỐI TRI THỨC - CẢ NĂM - CHƯƠNG 6 - HÀM SỐ...
BÀI TẬP DẠY THÊM TOÁN 11 - SÁCH KẾT NỐI TRI THỨC - CẢ NĂM - CHƯƠNG 6 - HÀM SỐ...BÀI TẬP DẠY THÊM TOÁN 11 - SÁCH KẾT NỐI TRI THỨC - CẢ NĂM - CHƯƠNG 6 - HÀM SỐ...
BÀI TẬP DẠY THÊM TOÁN 11 - SÁCH KẾT NỐI TRI THỨC - CẢ NĂM - CHƯƠNG 6 - HÀM SỐ...
Nguyen Thanh Tu Collection
 

What's hot (20)

20 ĐỀ ĐỀ XUẤT THI VÀO 10 THPT THÀNH PHỐ HẢI PHÒNG MÔN TIẾNG ANH 9 NĂM HỌC 202...
20 ĐỀ ĐỀ XUẤT THI VÀO 10 THPT THÀNH PHỐ HẢI PHÒNG MÔN TIẾNG ANH 9 NĂM HỌC 202...20 ĐỀ ĐỀ XUẤT THI VÀO 10 THPT THÀNH PHỐ HẢI PHÒNG MÔN TIẾNG ANH 9 NĂM HỌC 202...
20 ĐỀ ĐỀ XUẤT THI VÀO 10 THPT THÀNH PHỐ HẢI PHÒNG MÔN TIẾNG ANH 9 NĂM HỌC 202...
 
BÀI TẬP DẠY THÊM TOÁN 11 - SÁCH CÁNH DIỀU - CẢ NĂM - CHUYÊN ĐỀ 1 - HÀM SỐ LƯỢ...
BÀI TẬP DẠY THÊM TOÁN 11 - SÁCH CÁNH DIỀU - CẢ NĂM - CHUYÊN ĐỀ 1 - HÀM SỐ LƯỢ...BÀI TẬP DẠY THÊM TOÁN 11 - SÁCH CÁNH DIỀU - CẢ NĂM - CHUYÊN ĐỀ 1 - HÀM SỐ LƯỢ...
BÀI TẬP DẠY THÊM TOÁN 11 - SÁCH CÁNH DIỀU - CẢ NĂM - CHUYÊN ĐỀ 1 - HÀM SỐ LƯỢ...
 
TÀI LIỆU DẠY THÊM HÓA HỌC 11 THEO MỨC ĐỘ SÁCH MỚI - KẾT NỐI TRI THỨC, CÁNH DI...
TÀI LIỆU DẠY THÊM HÓA HỌC 11 THEO MỨC ĐỘ SÁCH MỚI - KẾT NỐI TRI THỨC, CÁNH DI...TÀI LIỆU DẠY THÊM HÓA HỌC 11 THEO MỨC ĐỘ SÁCH MỚI - KẾT NỐI TRI THỨC, CÁNH DI...
TÀI LIỆU DẠY THÊM HÓA HỌC 11 THEO MỨC ĐỘ SÁCH MỚI - KẾT NỐI TRI THỨC, CÁNH DI...
 
ĐỀ THAM KHẢO THEO HƯỚNG MINH HỌA 2025 KIỂM TRA GIỮA HỌC KÌ 2 NĂM HỌC 2023-202...
ĐỀ THAM KHẢO THEO HƯỚNG MINH HỌA 2025 KIỂM TRA GIỮA HỌC KÌ 2 NĂM HỌC 2023-202...ĐỀ THAM KHẢO THEO HƯỚNG MINH HỌA 2025 KIỂM TRA GIỮA HỌC KÌ 2 NĂM HỌC 2023-202...
ĐỀ THAM KHẢO THEO HƯỚNG MINH HỌA 2025 KIỂM TRA GIỮA HỌC KÌ 2 NĂM HỌC 2023-202...
 
TỔNG HỢP ĐỀ THAM KHẢO TUYỂN SINH 10 NĂM HỌC 2024 - 2025 MÔN TOÁN 9 SỞ GIÁO DỤ...
TỔNG HỢP ĐỀ THAM KHẢO TUYỂN SINH 10 NĂM HỌC 2024 - 2025 MÔN TOÁN 9 SỞ GIÁO DỤ...TỔNG HỢP ĐỀ THAM KHẢO TUYỂN SINH 10 NĂM HỌC 2024 - 2025 MÔN TOÁN 9 SỞ GIÁO DỤ...
TỔNG HỢP ĐỀ THAM KHẢO TUYỂN SINH 10 NĂM HỌC 2024 - 2025 MÔN TOÁN 9 SỞ GIÁO DỤ...
 
20 ĐỀ DỰ ĐOÁN - PHÁT TRIỂN ĐỀ MINH HỌA BGD KỲ THI TỐT NGHIỆP THPT NĂM 2023 MÔ...
20 ĐỀ DỰ ĐOÁN - PHÁT TRIỂN ĐỀ MINH HỌA BGD KỲ THI TỐT NGHIỆP THPT NĂM 2023 MÔ...20 ĐỀ DỰ ĐOÁN - PHÁT TRIỂN ĐỀ MINH HỌA BGD KỲ THI TỐT NGHIỆP THPT NĂM 2023 MÔ...
20 ĐỀ DỰ ĐOÁN - PHÁT TRIỂN ĐỀ MINH HỌA BGD KỲ THI TỐT NGHIỆP THPT NĂM 2023 MÔ...
 
ĐỀ ÔN TẬP THAM KHẢO THI THỬ GIỮA HỌC KÌ 2 HÓA HỌC 11 - DÙNG CHUNG 3 SÁCH - TH...
ĐỀ ÔN TẬP THAM KHẢO THI THỬ GIỮA HỌC KÌ 2 HÓA HỌC 11 - DÙNG CHUNG 3 SÁCH - TH...ĐỀ ÔN TẬP THAM KHẢO THI THỬ GIỮA HỌC KÌ 2 HÓA HỌC 11 - DÙNG CHUNG 3 SÁCH - TH...
ĐỀ ÔN TẬP THAM KHẢO THI THỬ GIỮA HỌC KÌ 2 HÓA HỌC 11 - DÙNG CHUNG 3 SÁCH - TH...
 
TÀI LIỆU DẠY THÊM ĐẦY ĐỦ TOÁN 11 NĂM 2024 - HỌC KÌ 1 (KẾT NỐI TRI THỨC) - LÝ ...
TÀI LIỆU DẠY THÊM ĐẦY ĐỦ TOÁN 11 NĂM 2024 - HỌC KÌ 1 (KẾT NỐI TRI THỨC) - LÝ ...TÀI LIỆU DẠY THÊM ĐẦY ĐỦ TOÁN 11 NĂM 2024 - HỌC KÌ 1 (KẾT NỐI TRI THỨC) - LÝ ...
TÀI LIỆU DẠY THÊM ĐẦY ĐỦ TOÁN 11 NĂM 2024 - HỌC KÌ 1 (KẾT NỐI TRI THỨC) - LÝ ...
 
10 ĐỀ KIỂM TRA + 6 ĐỀ ÔN TẬP CUỐI KÌ 2 VẬT LÝ 11 - KẾT NỐI TRI THỨC - THEO C...
10 ĐỀ KIỂM TRA + 6 ĐỀ ÔN TẬP CUỐI KÌ 2 VẬT LÝ 11 - KẾT NỐI TRI THỨC - THEO C...10 ĐỀ KIỂM TRA + 6 ĐỀ ÔN TẬP CUỐI KÌ 2 VẬT LÝ 11 - KẾT NỐI TRI THỨC - THEO C...
10 ĐỀ KIỂM TRA + 6 ĐỀ ÔN TẬP CUỐI KÌ 2 VẬT LÝ 11 - KẾT NỐI TRI THỨC - THEO C...
 
ĐỀ THI THỬ GIỮA HỌC KÌ 1 MÔN TIẾNG ANH I-LEARN SMART WORLD 11 - NĂM HỌC 2023-...
ĐỀ THI THỬ GIỮA HỌC KÌ 1 MÔN TIẾNG ANH I-LEARN SMART WORLD 11 - NĂM HỌC 2023-...ĐỀ THI THỬ GIỮA HỌC KÌ 1 MÔN TIẾNG ANH I-LEARN SMART WORLD 11 - NĂM HỌC 2023-...
ĐỀ THI THỬ GIỮA HỌC KÌ 1 MÔN TIẾNG ANH I-LEARN SMART WORLD 11 - NĂM HỌC 2023-...
 
CHUYÊN ĐỀ DẠY THÊM HÓA HỌC 11 - SÁCH MỚI NĂM 2023 (CHƯƠNG 1 CÂN BẰNG HÓA HỌC)...
CHUYÊN ĐỀ DẠY THÊM HÓA HỌC 11 - SÁCH MỚI NĂM 2023 (CHƯƠNG 1 CÂN BẰNG HÓA HỌC)...CHUYÊN ĐỀ DẠY THÊM HÓA HỌC 11 - SÁCH MỚI NĂM 2023 (CHƯƠNG 1 CÂN BẰNG HÓA HỌC)...
CHUYÊN ĐỀ DẠY THÊM HÓA HỌC 11 - SÁCH MỚI NĂM 2023 (CHƯƠNG 1 CÂN BẰNG HÓA HỌC)...
 
BÀI TẬP DẠY THÊM TOÁN 11 - CHÂN TRỜI SÁNG TẠO - CẢ NĂM - CHƯƠNG 4 - ĐƯỜNG THẲ...
BÀI TẬP DẠY THÊM TOÁN 11 - CHÂN TRỜI SÁNG TẠO - CẢ NĂM - CHƯƠNG 4 - ĐƯỜNG THẲ...BÀI TẬP DẠY THÊM TOÁN 11 - CHÂN TRỜI SÁNG TẠO - CẢ NĂM - CHƯƠNG 4 - ĐƯỜNG THẲ...
BÀI TẬP DẠY THÊM TOÁN 11 - CHÂN TRỜI SÁNG TẠO - CẢ NĂM - CHƯƠNG 4 - ĐƯỜNG THẲ...
 
BẤT ĐẲNG THỨC LUYỆN THI VÀO LỚP 10 CHUYÊN 2023-2024 (15 BẤT ĐẲNG THỨC VÀO LỚP...
BẤT ĐẲNG THỨC LUYỆN THI VÀO LỚP 10 CHUYÊN 2023-2024 (15 BẤT ĐẲNG THỨC VÀO LỚP...BẤT ĐẲNG THỨC LUYỆN THI VÀO LỚP 10 CHUYÊN 2023-2024 (15 BẤT ĐẲNG THỨC VÀO LỚP...
BẤT ĐẲNG THỨC LUYỆN THI VÀO LỚP 10 CHUYÊN 2023-2024 (15 BẤT ĐẲNG THỨC VÀO LỚP...
 
BỘ CÂU HỎI ÔN TẬP KIỂM TRA VÀ ĐÁNH GIÁ MÔN HÓA HỌC 11 (KẾT NỐI TRI THỨC) - TÀ...
BỘ CÂU HỎI ÔN TẬP KIỂM TRA VÀ ĐÁNH GIÁ MÔN HÓA HỌC 11 (KẾT NỐI TRI THỨC) - TÀ...BỘ CÂU HỎI ÔN TẬP KIỂM TRA VÀ ĐÁNH GIÁ MÔN HÓA HỌC 11 (KẾT NỐI TRI THỨC) - TÀ...
BỘ CÂU HỎI ÔN TẬP KIỂM TRA VÀ ĐÁNH GIÁ MÔN HÓA HỌC 11 (KẾT NỐI TRI THỨC) - TÀ...
 
BÀI TẬP DẠY THÊM TOÁN 11 - CHÂN TRỜI SÁNG TẠO - CẢ NĂM - CHƯƠNG 1 - HÀM SỐ LƯ...
BÀI TẬP DẠY THÊM TOÁN 11 - CHÂN TRỜI SÁNG TẠO - CẢ NĂM - CHƯƠNG 1 - HÀM SỐ LƯ...BÀI TẬP DẠY THÊM TOÁN 11 - CHÂN TRỜI SÁNG TẠO - CẢ NĂM - CHƯƠNG 1 - HÀM SỐ LƯ...
BÀI TẬP DẠY THÊM TOÁN 11 - CHÂN TRỜI SÁNG TẠO - CẢ NĂM - CHƯƠNG 1 - HÀM SỐ LƯ...
 
15 ĐỀ THI THỬ TUYỂN SINH VÀO LỚP 10 MÔN TIẾNG ANH NĂM HỌC 2023 - 2024 SỞ GIÁO...
15 ĐỀ THI THỬ TUYỂN SINH VÀO LỚP 10 MÔN TIẾNG ANH NĂM HỌC 2023 - 2024 SỞ GIÁO...15 ĐỀ THI THỬ TUYỂN SINH VÀO LỚP 10 MÔN TIẾNG ANH NĂM HỌC 2023 - 2024 SỞ GIÁO...
15 ĐỀ THI THỬ TUYỂN SINH VÀO LỚP 10 MÔN TIẾNG ANH NĂM HỌC 2023 - 2024 SỞ GIÁO...
 
TÓM TẮT LÝ THUYẾT + BÀI TẬP TRẮC NGHIỆM TỪNG BÀI HỌC THEO 4 CẤP ĐỘ HÓA HỌC LỚ...
TÓM TẮT LÝ THUYẾT + BÀI TẬP TRẮC NGHIỆM TỪNG BÀI HỌC THEO 4 CẤP ĐỘ HÓA HỌC LỚ...TÓM TẮT LÝ THUYẾT + BÀI TẬP TRẮC NGHIỆM TỪNG BÀI HỌC THEO 4 CẤP ĐỘ HÓA HỌC LỚ...
TÓM TẮT LÝ THUYẾT + BÀI TẬP TRẮC NGHIỆM TỪNG BÀI HỌC THEO 4 CẤP ĐỘ HÓA HỌC LỚ...
 
GIÁO ÁN KẾ HOẠCH BÀI DẠY (SGK + CHUYÊN ĐỀ) HÓA HỌC 11 KẾT NỐI TRI THỨC THEO C...
GIÁO ÁN KẾ HOẠCH BÀI DẠY (SGK + CHUYÊN ĐỀ) HÓA HỌC 11 KẾT NỐI TRI THỨC THEO C...GIÁO ÁN KẾ HOẠCH BÀI DẠY (SGK + CHUYÊN ĐỀ) HÓA HỌC 11 KẾT NỐI TRI THỨC THEO C...
GIÁO ÁN KẾ HOẠCH BÀI DẠY (SGK + CHUYÊN ĐỀ) HÓA HỌC 11 KẾT NỐI TRI THỨC THEO C...
 
Đề Thi HK2 Toán 8 - THCS Nguyễn Văn Luông
Đề Thi HK2 Toán 8 - THCS Nguyễn Văn LuôngĐề Thi HK2 Toán 8 - THCS Nguyễn Văn Luông
Đề Thi HK2 Toán 8 - THCS Nguyễn Văn Luông
 
BÀI TẬP DẠY THÊM TOÁN 11 - SÁCH KẾT NỐI TRI THỨC - CẢ NĂM - CHƯƠNG 6 - HÀM SỐ...
BÀI TẬP DẠY THÊM TOÁN 11 - SÁCH KẾT NỐI TRI THỨC - CẢ NĂM - CHƯƠNG 6 - HÀM SỐ...BÀI TẬP DẠY THÊM TOÁN 11 - SÁCH KẾT NỐI TRI THỨC - CẢ NĂM - CHƯƠNG 6 - HÀM SỐ...
BÀI TẬP DẠY THÊM TOÁN 11 - SÁCH KẾT NỐI TRI THỨC - CẢ NĂM - CHƯƠNG 6 - HÀM SỐ...
 

Similar to ĐỀ THI THỬ TUYỂN SINH VÀO LỚP 10 THPT MÔN TOÁN CÁC TỈNH NĂM HỌC 2023-2024 CÓ LỜI GIẢI CHI TIẾT.pdf

Đề Thi HK2 Toán 9 - THCS An Nhơn
Đề Thi HK2 Toán 9 - THCS  An NhơnĐề Thi HK2 Toán 9 - THCS  An Nhơn
Đề Thi HK2 Toán 9 - THCS An Nhơn
Trung Tâm Gia Sư Việt Trí
 
Đề Thi HK2 Toán 9 - THCS An Nhơn Tây
Đề Thi HK2 Toán 9 - THCS  An Nhơn TâyĐề Thi HK2 Toán 9 - THCS  An Nhơn Tây
Đề Thi HK2 Toán 9 - THCS An Nhơn Tây
Trung Tâm Gia Sư Việt Trí
 
Đề Thi HK2 Toán 9 - THCS Bình Chánh
Đề Thi HK2 Toán 9 - THCS  Bình ChánhĐề Thi HK2 Toán 9 - THCS  Bình Chánh
Đề Thi HK2 Toán 9 - THCS Bình Chánh
Trung Tâm Gia Sư Việt Trí
 
Đề Thi HK2 Toán 9 - THCS Bình Chánh
Đề Thi HK2 Toán 9 - THCS  Bình ChánhĐề Thi HK2 Toán 9 - THCS  Bình Chánh
Đề Thi HK2 Toán 9 - THCS Bình Chánh
Trung Tâm Gia Sư Việt Trí
 
75 de thi hoc sinh gioi toan 7 co dap an chi tiet
75 de thi hoc sinh gioi toan 7 co dap an chi tiet75 de thi hoc sinh gioi toan 7 co dap an chi tiet
75 de thi hoc sinh gioi toan 7 co dap an chi tiet
haic2hv.net
 
De thi-thu-thpt-quoc-gia-mon-toan-lan-3-nam-2015-truong-thpt-trieu-son-3
De thi-thu-thpt-quoc-gia-mon-toan-lan-3-nam-2015-truong-thpt-trieu-son-3De thi-thu-thpt-quoc-gia-mon-toan-lan-3-nam-2015-truong-thpt-trieu-son-3
De thi-thu-thpt-quoc-gia-mon-toan-lan-3-nam-2015-truong-thpt-trieu-son-3
Hồng Nguyễn
 
Mathvn.com 11. toan tran phu lan 12014
Mathvn.com   11. toan tran phu lan 12014Mathvn.com   11. toan tran phu lan 12014
Mathvn.com 11. toan tran phu lan 12014
Miễn Cưỡng
 
Đề Thi HK2 Toán 9 - THCS Kiến Thiết
Đề Thi HK2 Toán 9 - THCS  Kiến ThiếtĐề Thi HK2 Toán 9 - THCS  Kiến Thiết
Đề Thi HK2 Toán 9 - THCS Kiến Thiết
Trung Tâm Gia Sư Việt Trí
 
Đề Thi HK2 Toán 9 - THCS Chu Văn An
Đề Thi HK2 Toán 9 - THCS  Chu Văn AnĐề Thi HK2 Toán 9 - THCS  Chu Văn An
Đề Thi HK2 Toán 9 - THCS Chu Văn An
Trung Tâm Gia Sư Việt Trí
 
Dap an-de-thi-mon-toan-vao-lop-10-chuyen-nam-2014-tinh-hai-duong
Dap an-de-thi-mon-toan-vao-lop-10-chuyen-nam-2014-tinh-hai-duongDap an-de-thi-mon-toan-vao-lop-10-chuyen-nam-2014-tinh-hai-duong
Dap an-de-thi-mon-toan-vao-lop-10-chuyen-nam-2014-tinh-hai-duong
Linh Nguyễn
 
Dap an-de-thi-mon-toan-vao-lop-10-chuyen-nam-2014-tinh-hai-duong
Dap an-de-thi-mon-toan-vao-lop-10-chuyen-nam-2014-tinh-hai-duongDap an-de-thi-mon-toan-vao-lop-10-chuyen-nam-2014-tinh-hai-duong
Dap an-de-thi-mon-toan-vao-lop-10-chuyen-nam-2014-tinh-hai-duongLinh Nguyễn
 
Toan pt.de046.2010
Toan pt.de046.2010Toan pt.de046.2010
Toan pt.de046.2010
BẢO Hí
 
Đề thi tuyển sinh vào 10 - môn toán tỉnh Hải Dương - 2012-2013
Đề thi tuyển sinh vào 10 - môn toán tỉnh Hải Dương - 2012-2013Đề thi tuyển sinh vào 10 - môn toán tỉnh Hải Dương - 2012-2013
Đề thi tuyển sinh vào 10 - môn toán tỉnh Hải Dương - 2012-2013
tieuhocvn .info
 
Sach luyen de toan 2014 p1 (thay hung)
Sach luyen de toan 2014 p1 (thay hung)Sach luyen de toan 2014 p1 (thay hung)
Sach luyen de toan 2014 p1 (thay hung)
KyNang Toan
 
(Chuyendeonthi.wordpress.com) de thi toan on thi dh hay va dac sac phan 1
(Chuyendeonthi.wordpress.com) de thi toan on thi dh hay va dac sac phan 1(Chuyendeonthi.wordpress.com) de thi toan on thi dh hay va dac sac phan 1
(Chuyendeonthi.wordpress.com) de thi toan on thi dh hay va dac sac phan 1
Kings Kingsley
 
Đề thi vào 10 môn Toán chuyên Lạng Sơn năm 2013 - 2014
Đề thi vào 10 môn Toán chuyên Lạng Sơn năm 2013 - 2014Đề thi vào 10 môn Toán chuyên Lạng Sơn năm 2013 - 2014
Đề thi vào 10 môn Toán chuyên Lạng Sơn năm 2013 - 2014
Bồi Dưỡng HSG Toán Lớp 3
 
Đề thi thử ĐH và đáp án môn Toán học lần 1 (2014) trường THPT Trần Phú, Hà Tĩ...
Đề thi thử ĐH và đáp án môn Toán học lần 1 (2014) trường THPT Trần Phú, Hà Tĩ...Đề thi thử ĐH và đáp án môn Toán học lần 1 (2014) trường THPT Trần Phú, Hà Tĩ...
Đề thi thử ĐH và đáp án môn Toán học lần 1 (2014) trường THPT Trần Phú, Hà Tĩ...
Megabook
 
Dap an chi tiet cao dang tu 2002-2004
Dap an chi tiet  cao dang tu  2002-2004Dap an chi tiet  cao dang tu  2002-2004
Dap an chi tiet cao dang tu 2002-2004
Thiên Đường Tình Yêu
 
De l10-hai phong-2014-toan
De l10-hai phong-2014-toanDe l10-hai phong-2014-toan
De l10-hai phong-2014-toanngatb1989
 
Tai lieu luyen thi dai hoc de thi dh mon toan khoi b - nam 2009
Tai lieu luyen thi dai hoc   de thi dh mon toan khoi b - nam 2009Tai lieu luyen thi dai hoc   de thi dh mon toan khoi b - nam 2009
Tai lieu luyen thi dai hoc de thi dh mon toan khoi b - nam 2009
Trungtâmluyệnthi Qsc
 

Similar to ĐỀ THI THỬ TUYỂN SINH VÀO LỚP 10 THPT MÔN TOÁN CÁC TỈNH NĂM HỌC 2023-2024 CÓ LỜI GIẢI CHI TIẾT.pdf (20)

Đề Thi HK2 Toán 9 - THCS An Nhơn
Đề Thi HK2 Toán 9 - THCS  An NhơnĐề Thi HK2 Toán 9 - THCS  An Nhơn
Đề Thi HK2 Toán 9 - THCS An Nhơn
 
Đề Thi HK2 Toán 9 - THCS An Nhơn Tây
Đề Thi HK2 Toán 9 - THCS  An Nhơn TâyĐề Thi HK2 Toán 9 - THCS  An Nhơn Tây
Đề Thi HK2 Toán 9 - THCS An Nhơn Tây
 
Đề Thi HK2 Toán 9 - THCS Bình Chánh
Đề Thi HK2 Toán 9 - THCS  Bình ChánhĐề Thi HK2 Toán 9 - THCS  Bình Chánh
Đề Thi HK2 Toán 9 - THCS Bình Chánh
 
Đề Thi HK2 Toán 9 - THCS Bình Chánh
Đề Thi HK2 Toán 9 - THCS  Bình ChánhĐề Thi HK2 Toán 9 - THCS  Bình Chánh
Đề Thi HK2 Toán 9 - THCS Bình Chánh
 
75 de thi hoc sinh gioi toan 7 co dap an chi tiet
75 de thi hoc sinh gioi toan 7 co dap an chi tiet75 de thi hoc sinh gioi toan 7 co dap an chi tiet
75 de thi hoc sinh gioi toan 7 co dap an chi tiet
 
De thi-thu-thpt-quoc-gia-mon-toan-lan-3-nam-2015-truong-thpt-trieu-son-3
De thi-thu-thpt-quoc-gia-mon-toan-lan-3-nam-2015-truong-thpt-trieu-son-3De thi-thu-thpt-quoc-gia-mon-toan-lan-3-nam-2015-truong-thpt-trieu-son-3
De thi-thu-thpt-quoc-gia-mon-toan-lan-3-nam-2015-truong-thpt-trieu-son-3
 
Mathvn.com 11. toan tran phu lan 12014
Mathvn.com   11. toan tran phu lan 12014Mathvn.com   11. toan tran phu lan 12014
Mathvn.com 11. toan tran phu lan 12014
 
Đề Thi HK2 Toán 9 - THCS Kiến Thiết
Đề Thi HK2 Toán 9 - THCS  Kiến ThiếtĐề Thi HK2 Toán 9 - THCS  Kiến Thiết
Đề Thi HK2 Toán 9 - THCS Kiến Thiết
 
Đề Thi HK2 Toán 9 - THCS Chu Văn An
Đề Thi HK2 Toán 9 - THCS  Chu Văn AnĐề Thi HK2 Toán 9 - THCS  Chu Văn An
Đề Thi HK2 Toán 9 - THCS Chu Văn An
 
Dap an-de-thi-mon-toan-vao-lop-10-chuyen-nam-2014-tinh-hai-duong
Dap an-de-thi-mon-toan-vao-lop-10-chuyen-nam-2014-tinh-hai-duongDap an-de-thi-mon-toan-vao-lop-10-chuyen-nam-2014-tinh-hai-duong
Dap an-de-thi-mon-toan-vao-lop-10-chuyen-nam-2014-tinh-hai-duong
 
Dap an-de-thi-mon-toan-vao-lop-10-chuyen-nam-2014-tinh-hai-duong
Dap an-de-thi-mon-toan-vao-lop-10-chuyen-nam-2014-tinh-hai-duongDap an-de-thi-mon-toan-vao-lop-10-chuyen-nam-2014-tinh-hai-duong
Dap an-de-thi-mon-toan-vao-lop-10-chuyen-nam-2014-tinh-hai-duong
 
Toan pt.de046.2010
Toan pt.de046.2010Toan pt.de046.2010
Toan pt.de046.2010
 
Đề thi tuyển sinh vào 10 - môn toán tỉnh Hải Dương - 2012-2013
Đề thi tuyển sinh vào 10 - môn toán tỉnh Hải Dương - 2012-2013Đề thi tuyển sinh vào 10 - môn toán tỉnh Hải Dương - 2012-2013
Đề thi tuyển sinh vào 10 - môn toán tỉnh Hải Dương - 2012-2013
 
Sach luyen de toan 2014 p1 (thay hung)
Sach luyen de toan 2014 p1 (thay hung)Sach luyen de toan 2014 p1 (thay hung)
Sach luyen de toan 2014 p1 (thay hung)
 
(Chuyendeonthi.wordpress.com) de thi toan on thi dh hay va dac sac phan 1
(Chuyendeonthi.wordpress.com) de thi toan on thi dh hay va dac sac phan 1(Chuyendeonthi.wordpress.com) de thi toan on thi dh hay va dac sac phan 1
(Chuyendeonthi.wordpress.com) de thi toan on thi dh hay va dac sac phan 1
 
Đề thi vào 10 môn Toán chuyên Lạng Sơn năm 2013 - 2014
Đề thi vào 10 môn Toán chuyên Lạng Sơn năm 2013 - 2014Đề thi vào 10 môn Toán chuyên Lạng Sơn năm 2013 - 2014
Đề thi vào 10 môn Toán chuyên Lạng Sơn năm 2013 - 2014
 
Đề thi thử ĐH và đáp án môn Toán học lần 1 (2014) trường THPT Trần Phú, Hà Tĩ...
Đề thi thử ĐH và đáp án môn Toán học lần 1 (2014) trường THPT Trần Phú, Hà Tĩ...Đề thi thử ĐH và đáp án môn Toán học lần 1 (2014) trường THPT Trần Phú, Hà Tĩ...
Đề thi thử ĐH và đáp án môn Toán học lần 1 (2014) trường THPT Trần Phú, Hà Tĩ...
 
Dap an chi tiet cao dang tu 2002-2004
Dap an chi tiet  cao dang tu  2002-2004Dap an chi tiet  cao dang tu  2002-2004
Dap an chi tiet cao dang tu 2002-2004
 
De l10-hai phong-2014-toan
De l10-hai phong-2014-toanDe l10-hai phong-2014-toan
De l10-hai phong-2014-toan
 
Tai lieu luyen thi dai hoc de thi dh mon toan khoi b - nam 2009
Tai lieu luyen thi dai hoc   de thi dh mon toan khoi b - nam 2009Tai lieu luyen thi dai hoc   de thi dh mon toan khoi b - nam 2009
Tai lieu luyen thi dai hoc de thi dh mon toan khoi b - nam 2009
 

More from Nguyen Thanh Tu Collection

CHUYÊN ĐỀ DẠY THÊM VẬT LÝ LỚP 12 THEO FORM THI MỚI BGD 2025 DÙNG CHUNG 3 SÁCH...
CHUYÊN ĐỀ DẠY THÊM VẬT LÝ LỚP 12 THEO FORM THI MỚI BGD 2025 DÙNG CHUNG 3 SÁCH...CHUYÊN ĐỀ DẠY THÊM VẬT LÝ LỚP 12 THEO FORM THI MỚI BGD 2025 DÙNG CHUNG 3 SÁCH...
CHUYÊN ĐỀ DẠY THÊM VẬT LÝ LỚP 12 THEO FORM THI MỚI BGD 2025 DÙNG CHUNG 3 SÁCH...
Nguyen Thanh Tu Collection
 
CHUYÊN ĐỀ DẠY THÊM TOÁN LỚP 12 THEO FORM THI MỚI BGD 2025 - KẾT NỐI TRI THỨC ...
CHUYÊN ĐỀ DẠY THÊM TOÁN LỚP 12 THEO FORM THI MỚI BGD 2025 - KẾT NỐI TRI THỨC ...CHUYÊN ĐỀ DẠY THÊM TOÁN LỚP 12 THEO FORM THI MỚI BGD 2025 - KẾT NỐI TRI THỨC ...
CHUYÊN ĐỀ DẠY THÊM TOÁN LỚP 12 THEO FORM THI MỚI BGD 2025 - KẾT NỐI TRI THỨC ...
Nguyen Thanh Tu Collection
 
CHUYÊN ĐỀ DẠY THÊM TOÁN LỚP 12 THEO FORM THI MỚI BGD 2025 - CHÂN TRỜI SÁNG TẠ...
CHUYÊN ĐỀ DẠY THÊM TOÁN LỚP 12 THEO FORM THI MỚI BGD 2025 - CHÂN TRỜI SÁNG TẠ...CHUYÊN ĐỀ DẠY THÊM TOÁN LỚP 12 THEO FORM THI MỚI BGD 2025 - CHÂN TRỜI SÁNG TẠ...
CHUYÊN ĐỀ DẠY THÊM TOÁN LỚP 12 THEO FORM THI MỚI BGD 2025 - CHÂN TRỜI SÁNG TẠ...
Nguyen Thanh Tu Collection
 
TÀI LIỆU DẠY THÊM HÓA HỌC 12 - SÁCH MỚI (BẢN HS+GV) (FORM BÀI TẬP 2025 CHUNG ...
TÀI LIỆU DẠY THÊM HÓA HỌC 12 - SÁCH MỚI (BẢN HS+GV) (FORM BÀI TẬP 2025 CHUNG ...TÀI LIỆU DẠY THÊM HÓA HỌC 12 - SÁCH MỚI (BẢN HS+GV) (FORM BÀI TẬP 2025 CHUNG ...
TÀI LIỆU DẠY THÊM HÓA HỌC 12 - SÁCH MỚI (BẢN HS+GV) (FORM BÀI TẬP 2025 CHUNG ...
Nguyen Thanh Tu Collection
 
CHUYÊN ĐỀ ÔN TẬP VÀ PHÁT TRIỂN CÂU HỎI TRONG ĐỀ MINH HỌA THI TỐT NGHIỆP THPT ...
CHUYÊN ĐỀ ÔN TẬP VÀ PHÁT TRIỂN CÂU HỎI TRONG ĐỀ MINH HỌA THI TỐT NGHIỆP THPT ...CHUYÊN ĐỀ ÔN TẬP VÀ PHÁT TRIỂN CÂU HỎI TRONG ĐỀ MINH HỌA THI TỐT NGHIỆP THPT ...
CHUYÊN ĐỀ ÔN TẬP VÀ PHÁT TRIỂN CÂU HỎI TRONG ĐỀ MINH HỌA THI TỐT NGHIỆP THPT ...
Nguyen Thanh Tu Collection
 
BÀI TẬP BỔ TRỢ TIẾNG ANH LỚP 8 - CẢ NĂM - FRIENDS PLUS - NĂM HỌC 2023-2024 (B...
BÀI TẬP BỔ TRỢ TIẾNG ANH LỚP 8 - CẢ NĂM - FRIENDS PLUS - NĂM HỌC 2023-2024 (B...BÀI TẬP BỔ TRỢ TIẾNG ANH LỚP 8 - CẢ NĂM - FRIENDS PLUS - NĂM HỌC 2023-2024 (B...
BÀI TẬP BỔ TRỢ TIẾNG ANH LỚP 8 - CẢ NĂM - FRIENDS PLUS - NĂM HỌC 2023-2024 (B...
Nguyen Thanh Tu Collection
 
TỔNG HỢP 135 CÂU HỎI DI TRUYỀN PHÂN TỬ LUYỆN THI HỌC SINH GIỎI THPT MÔN SINH ...
TỔNG HỢP 135 CÂU HỎI DI TRUYỀN PHÂN TỬ LUYỆN THI HỌC SINH GIỎI THPT MÔN SINH ...TỔNG HỢP 135 CÂU HỎI DI TRUYỀN PHÂN TỬ LUYỆN THI HỌC SINH GIỎI THPT MÔN SINH ...
TỔNG HỢP 135 CÂU HỎI DI TRUYỀN PHÂN TỬ LUYỆN THI HỌC SINH GIỎI THPT MÔN SINH ...
Nguyen Thanh Tu Collection
 
BÀI TẬP DẠY THÊM HÓA HỌC LỚP 12 - CẢ NĂM - THEO FORM THI MỚI BGD 2025 (DÙNG C...
BÀI TẬP DẠY THÊM HÓA HỌC LỚP 12 - CẢ NĂM - THEO FORM THI MỚI BGD 2025 (DÙNG C...BÀI TẬP DẠY THÊM HÓA HỌC LỚP 12 - CẢ NĂM - THEO FORM THI MỚI BGD 2025 (DÙNG C...
BÀI TẬP DẠY THÊM HÓA HỌC LỚP 12 - CẢ NĂM - THEO FORM THI MỚI BGD 2025 (DÙNG C...
Nguyen Thanh Tu Collection
 
BÀI TẬP BỔ TRỢ TIẾNG ANH LỚP 9 CẢ NĂM - GLOBAL SUCCESS - NĂM HỌC 2024-2025 - ...
BÀI TẬP BỔ TRỢ TIẾNG ANH LỚP 9 CẢ NĂM - GLOBAL SUCCESS - NĂM HỌC 2024-2025 - ...BÀI TẬP BỔ TRỢ TIẾNG ANH LỚP 9 CẢ NĂM - GLOBAL SUCCESS - NĂM HỌC 2024-2025 - ...
BÀI TẬP BỔ TRỢ TIẾNG ANH LỚP 9 CẢ NĂM - GLOBAL SUCCESS - NĂM HỌC 2024-2025 - ...
Nguyen Thanh Tu Collection
 
BÀI TẬP DẠY THÊM TIẾNG ANH LỚP 7 CẢ NĂM FRIENDS PLUS SÁCH CHÂN TRỜI SÁNG TẠO ...
BÀI TẬP DẠY THÊM TIẾNG ANH LỚP 7 CẢ NĂM FRIENDS PLUS SÁCH CHÂN TRỜI SÁNG TẠO ...BÀI TẬP DẠY THÊM TIẾNG ANH LỚP 7 CẢ NĂM FRIENDS PLUS SÁCH CHÂN TRỜI SÁNG TẠO ...
BÀI TẬP DẠY THÊM TIẾNG ANH LỚP 7 CẢ NĂM FRIENDS PLUS SÁCH CHÂN TRỜI SÁNG TẠO ...
Nguyen Thanh Tu Collection
 
CHUYÊN ĐỀ DẠY THÊM HÓA HỌC LỚP 10 - SÁCH MỚI - FORM BÀI TẬP 2025 (DÙNG CHUNG ...
CHUYÊN ĐỀ DẠY THÊM HÓA HỌC LỚP 10 - SÁCH MỚI - FORM BÀI TẬP 2025 (DÙNG CHUNG ...CHUYÊN ĐỀ DẠY THÊM HÓA HỌC LỚP 10 - SÁCH MỚI - FORM BÀI TẬP 2025 (DÙNG CHUNG ...
CHUYÊN ĐỀ DẠY THÊM HÓA HỌC LỚP 10 - SÁCH MỚI - FORM BÀI TẬP 2025 (DÙNG CHUNG ...
Nguyen Thanh Tu Collection
 
BÀI TẬP BỔ TRỢ TIẾNG ANH 8 CẢ NĂM - GLOBAL SUCCESS - NĂM HỌC 2023-2024 (CÓ FI...
BÀI TẬP BỔ TRỢ TIẾNG ANH 8 CẢ NĂM - GLOBAL SUCCESS - NĂM HỌC 2023-2024 (CÓ FI...BÀI TẬP BỔ TRỢ TIẾNG ANH 8 CẢ NĂM - GLOBAL SUCCESS - NĂM HỌC 2023-2024 (CÓ FI...
BÀI TẬP BỔ TRỢ TIẾNG ANH 8 CẢ NĂM - GLOBAL SUCCESS - NĂM HỌC 2023-2024 (CÓ FI...
Nguyen Thanh Tu Collection
 
BÀI TẬP BỔ TRỢ TIẾNG ANH I-LEARN SMART WORLD 9 CẢ NĂM CÓ TEST THEO UNIT NĂM H...
BÀI TẬP BỔ TRỢ TIẾNG ANH I-LEARN SMART WORLD 9 CẢ NĂM CÓ TEST THEO UNIT NĂM H...BÀI TẬP BỔ TRỢ TIẾNG ANH I-LEARN SMART WORLD 9 CẢ NĂM CÓ TEST THEO UNIT NĂM H...
BÀI TẬP BỔ TRỢ TIẾNG ANH I-LEARN SMART WORLD 9 CẢ NĂM CÓ TEST THEO UNIT NĂM H...
Nguyen Thanh Tu Collection
 
CHUYÊN ĐỀ BỒI DƯỠNG HỌC SINH GIỎI KHOA HỌC TỰ NHIÊN 9 CHƯƠNG TRÌNH MỚI - PHẦN...
CHUYÊN ĐỀ BỒI DƯỠNG HỌC SINH GIỎI KHOA HỌC TỰ NHIÊN 9 CHƯƠNG TRÌNH MỚI - PHẦN...CHUYÊN ĐỀ BỒI DƯỠNG HỌC SINH GIỎI KHOA HỌC TỰ NHIÊN 9 CHƯƠNG TRÌNH MỚI - PHẦN...
CHUYÊN ĐỀ BỒI DƯỠNG HỌC SINH GIỎI KHOA HỌC TỰ NHIÊN 9 CHƯƠNG TRÌNH MỚI - PHẦN...
Nguyen Thanh Tu Collection
 
BÀI TẬP BỔ TRỢ TIẾNG ANH GLOBAL SUCCESS LỚP 3 - CẢ NĂM (CÓ FILE NGHE VÀ ĐÁP Á...
BÀI TẬP BỔ TRỢ TIẾNG ANH GLOBAL SUCCESS LỚP 3 - CẢ NĂM (CÓ FILE NGHE VÀ ĐÁP Á...BÀI TẬP BỔ TRỢ TIẾNG ANH GLOBAL SUCCESS LỚP 3 - CẢ NĂM (CÓ FILE NGHE VÀ ĐÁP Á...
BÀI TẬP BỔ TRỢ TIẾNG ANH GLOBAL SUCCESS LỚP 3 - CẢ NĂM (CÓ FILE NGHE VÀ ĐÁP Á...
Nguyen Thanh Tu Collection
 
98 BÀI LUYỆN NGHE TUYỂN SINH VÀO LỚP 10 TIẾNG ANH DẠNG TRẮC NGHIỆM 4 CÂU TRẢ ...
98 BÀI LUYỆN NGHE TUYỂN SINH VÀO LỚP 10 TIẾNG ANH DẠNG TRẮC NGHIỆM 4 CÂU TRẢ ...98 BÀI LUYỆN NGHE TUYỂN SINH VÀO LỚP 10 TIẾNG ANH DẠNG TRẮC NGHIỆM 4 CÂU TRẢ ...
98 BÀI LUYỆN NGHE TUYỂN SINH VÀO LỚP 10 TIẾNG ANH DẠNG TRẮC NGHIỆM 4 CÂU TRẢ ...
Nguyen Thanh Tu Collection
 
GIÁO ÁN DẠY THÊM (KẾ HOẠCH BÀI BUỔI 2) - TIẾNG ANH 8 GLOBAL SUCCESS (2 CỘT) N...
GIÁO ÁN DẠY THÊM (KẾ HOẠCH BÀI BUỔI 2) - TIẾNG ANH 8 GLOBAL SUCCESS (2 CỘT) N...GIÁO ÁN DẠY THÊM (KẾ HOẠCH BÀI BUỔI 2) - TIẾNG ANH 8 GLOBAL SUCCESS (2 CỘT) N...
GIÁO ÁN DẠY THÊM (KẾ HOẠCH BÀI BUỔI 2) - TIẾNG ANH 8 GLOBAL SUCCESS (2 CỘT) N...
Nguyen Thanh Tu Collection
 
Nghiên cứu cơ chế và động học phản ứng giữa hợp chất Aniline (C6H5NH2) với gố...
Nghiên cứu cơ chế và động học phản ứng giữa hợp chất Aniline (C6H5NH2) với gố...Nghiên cứu cơ chế và động học phản ứng giữa hợp chất Aniline (C6H5NH2) với gố...
Nghiên cứu cơ chế và động học phản ứng giữa hợp chất Aniline (C6H5NH2) với gố...
Nguyen Thanh Tu Collection
 
50 ĐỀ LUYỆN THI IOE LỚP 9 - NĂM HỌC 2022-2023 (CÓ LINK HÌNH, FILE AUDIO VÀ ĐÁ...
50 ĐỀ LUYỆN THI IOE LỚP 9 - NĂM HỌC 2022-2023 (CÓ LINK HÌNH, FILE AUDIO VÀ ĐÁ...50 ĐỀ LUYỆN THI IOE LỚP 9 - NĂM HỌC 2022-2023 (CÓ LINK HÌNH, FILE AUDIO VÀ ĐÁ...
50 ĐỀ LUYỆN THI IOE LỚP 9 - NĂM HỌC 2022-2023 (CÓ LINK HÌNH, FILE AUDIO VÀ ĐÁ...
Nguyen Thanh Tu Collection
 
BÀI TẬP DẠY THÊM TOÁN LỚP 12 SÁCH MỚI THEO FORM THI MỚI BGD 2025 - CÁNH DIỀU ...
BÀI TẬP DẠY THÊM TOÁN LỚP 12 SÁCH MỚI THEO FORM THI MỚI BGD 2025 - CÁNH DIỀU ...BÀI TẬP DẠY THÊM TOÁN LỚP 12 SÁCH MỚI THEO FORM THI MỚI BGD 2025 - CÁNH DIỀU ...
BÀI TẬP DẠY THÊM TOÁN LỚP 12 SÁCH MỚI THEO FORM THI MỚI BGD 2025 - CÁNH DIỀU ...
Nguyen Thanh Tu Collection
 

More from Nguyen Thanh Tu Collection (20)

CHUYÊN ĐỀ DẠY THÊM VẬT LÝ LỚP 12 THEO FORM THI MỚI BGD 2025 DÙNG CHUNG 3 SÁCH...
CHUYÊN ĐỀ DẠY THÊM VẬT LÝ LỚP 12 THEO FORM THI MỚI BGD 2025 DÙNG CHUNG 3 SÁCH...CHUYÊN ĐỀ DẠY THÊM VẬT LÝ LỚP 12 THEO FORM THI MỚI BGD 2025 DÙNG CHUNG 3 SÁCH...
CHUYÊN ĐỀ DẠY THÊM VẬT LÝ LỚP 12 THEO FORM THI MỚI BGD 2025 DÙNG CHUNG 3 SÁCH...
 
CHUYÊN ĐỀ DẠY THÊM TOÁN LỚP 12 THEO FORM THI MỚI BGD 2025 - KẾT NỐI TRI THỨC ...
CHUYÊN ĐỀ DẠY THÊM TOÁN LỚP 12 THEO FORM THI MỚI BGD 2025 - KẾT NỐI TRI THỨC ...CHUYÊN ĐỀ DẠY THÊM TOÁN LỚP 12 THEO FORM THI MỚI BGD 2025 - KẾT NỐI TRI THỨC ...
CHUYÊN ĐỀ DẠY THÊM TOÁN LỚP 12 THEO FORM THI MỚI BGD 2025 - KẾT NỐI TRI THỨC ...
 
CHUYÊN ĐỀ DẠY THÊM TOÁN LỚP 12 THEO FORM THI MỚI BGD 2025 - CHÂN TRỜI SÁNG TẠ...
CHUYÊN ĐỀ DẠY THÊM TOÁN LỚP 12 THEO FORM THI MỚI BGD 2025 - CHÂN TRỜI SÁNG TẠ...CHUYÊN ĐỀ DẠY THÊM TOÁN LỚP 12 THEO FORM THI MỚI BGD 2025 - CHÂN TRỜI SÁNG TẠ...
CHUYÊN ĐỀ DẠY THÊM TOÁN LỚP 12 THEO FORM THI MỚI BGD 2025 - CHÂN TRỜI SÁNG TẠ...
 
TÀI LIỆU DẠY THÊM HÓA HỌC 12 - SÁCH MỚI (BẢN HS+GV) (FORM BÀI TẬP 2025 CHUNG ...
TÀI LIỆU DẠY THÊM HÓA HỌC 12 - SÁCH MỚI (BẢN HS+GV) (FORM BÀI TẬP 2025 CHUNG ...TÀI LIỆU DẠY THÊM HÓA HỌC 12 - SÁCH MỚI (BẢN HS+GV) (FORM BÀI TẬP 2025 CHUNG ...
TÀI LIỆU DẠY THÊM HÓA HỌC 12 - SÁCH MỚI (BẢN HS+GV) (FORM BÀI TẬP 2025 CHUNG ...
 
CHUYÊN ĐỀ ÔN TẬP VÀ PHÁT TRIỂN CÂU HỎI TRONG ĐỀ MINH HỌA THI TỐT NGHIỆP THPT ...
CHUYÊN ĐỀ ÔN TẬP VÀ PHÁT TRIỂN CÂU HỎI TRONG ĐỀ MINH HỌA THI TỐT NGHIỆP THPT ...CHUYÊN ĐỀ ÔN TẬP VÀ PHÁT TRIỂN CÂU HỎI TRONG ĐỀ MINH HỌA THI TỐT NGHIỆP THPT ...
CHUYÊN ĐỀ ÔN TẬP VÀ PHÁT TRIỂN CÂU HỎI TRONG ĐỀ MINH HỌA THI TỐT NGHIỆP THPT ...
 
BÀI TẬP BỔ TRỢ TIẾNG ANH LỚP 8 - CẢ NĂM - FRIENDS PLUS - NĂM HỌC 2023-2024 (B...
BÀI TẬP BỔ TRỢ TIẾNG ANH LỚP 8 - CẢ NĂM - FRIENDS PLUS - NĂM HỌC 2023-2024 (B...BÀI TẬP BỔ TRỢ TIẾNG ANH LỚP 8 - CẢ NĂM - FRIENDS PLUS - NĂM HỌC 2023-2024 (B...
BÀI TẬP BỔ TRỢ TIẾNG ANH LỚP 8 - CẢ NĂM - FRIENDS PLUS - NĂM HỌC 2023-2024 (B...
 
TỔNG HỢP 135 CÂU HỎI DI TRUYỀN PHÂN TỬ LUYỆN THI HỌC SINH GIỎI THPT MÔN SINH ...
TỔNG HỢP 135 CÂU HỎI DI TRUYỀN PHÂN TỬ LUYỆN THI HỌC SINH GIỎI THPT MÔN SINH ...TỔNG HỢP 135 CÂU HỎI DI TRUYỀN PHÂN TỬ LUYỆN THI HỌC SINH GIỎI THPT MÔN SINH ...
TỔNG HỢP 135 CÂU HỎI DI TRUYỀN PHÂN TỬ LUYỆN THI HỌC SINH GIỎI THPT MÔN SINH ...
 
BÀI TẬP DẠY THÊM HÓA HỌC LỚP 12 - CẢ NĂM - THEO FORM THI MỚI BGD 2025 (DÙNG C...
BÀI TẬP DẠY THÊM HÓA HỌC LỚP 12 - CẢ NĂM - THEO FORM THI MỚI BGD 2025 (DÙNG C...BÀI TẬP DẠY THÊM HÓA HỌC LỚP 12 - CẢ NĂM - THEO FORM THI MỚI BGD 2025 (DÙNG C...
BÀI TẬP DẠY THÊM HÓA HỌC LỚP 12 - CẢ NĂM - THEO FORM THI MỚI BGD 2025 (DÙNG C...
 
BÀI TẬP BỔ TRỢ TIẾNG ANH LỚP 9 CẢ NĂM - GLOBAL SUCCESS - NĂM HỌC 2024-2025 - ...
BÀI TẬP BỔ TRỢ TIẾNG ANH LỚP 9 CẢ NĂM - GLOBAL SUCCESS - NĂM HỌC 2024-2025 - ...BÀI TẬP BỔ TRỢ TIẾNG ANH LỚP 9 CẢ NĂM - GLOBAL SUCCESS - NĂM HỌC 2024-2025 - ...
BÀI TẬP BỔ TRỢ TIẾNG ANH LỚP 9 CẢ NĂM - GLOBAL SUCCESS - NĂM HỌC 2024-2025 - ...
 
BÀI TẬP DẠY THÊM TIẾNG ANH LỚP 7 CẢ NĂM FRIENDS PLUS SÁCH CHÂN TRỜI SÁNG TẠO ...
BÀI TẬP DẠY THÊM TIẾNG ANH LỚP 7 CẢ NĂM FRIENDS PLUS SÁCH CHÂN TRỜI SÁNG TẠO ...BÀI TẬP DẠY THÊM TIẾNG ANH LỚP 7 CẢ NĂM FRIENDS PLUS SÁCH CHÂN TRỜI SÁNG TẠO ...
BÀI TẬP DẠY THÊM TIẾNG ANH LỚP 7 CẢ NĂM FRIENDS PLUS SÁCH CHÂN TRỜI SÁNG TẠO ...
 
CHUYÊN ĐỀ DẠY THÊM HÓA HỌC LỚP 10 - SÁCH MỚI - FORM BÀI TẬP 2025 (DÙNG CHUNG ...
CHUYÊN ĐỀ DẠY THÊM HÓA HỌC LỚP 10 - SÁCH MỚI - FORM BÀI TẬP 2025 (DÙNG CHUNG ...CHUYÊN ĐỀ DẠY THÊM HÓA HỌC LỚP 10 - SÁCH MỚI - FORM BÀI TẬP 2025 (DÙNG CHUNG ...
CHUYÊN ĐỀ DẠY THÊM HÓA HỌC LỚP 10 - SÁCH MỚI - FORM BÀI TẬP 2025 (DÙNG CHUNG ...
 
BÀI TẬP BỔ TRỢ TIẾNG ANH 8 CẢ NĂM - GLOBAL SUCCESS - NĂM HỌC 2023-2024 (CÓ FI...
BÀI TẬP BỔ TRỢ TIẾNG ANH 8 CẢ NĂM - GLOBAL SUCCESS - NĂM HỌC 2023-2024 (CÓ FI...BÀI TẬP BỔ TRỢ TIẾNG ANH 8 CẢ NĂM - GLOBAL SUCCESS - NĂM HỌC 2023-2024 (CÓ FI...
BÀI TẬP BỔ TRỢ TIẾNG ANH 8 CẢ NĂM - GLOBAL SUCCESS - NĂM HỌC 2023-2024 (CÓ FI...
 
BÀI TẬP BỔ TRỢ TIẾNG ANH I-LEARN SMART WORLD 9 CẢ NĂM CÓ TEST THEO UNIT NĂM H...
BÀI TẬP BỔ TRỢ TIẾNG ANH I-LEARN SMART WORLD 9 CẢ NĂM CÓ TEST THEO UNIT NĂM H...BÀI TẬP BỔ TRỢ TIẾNG ANH I-LEARN SMART WORLD 9 CẢ NĂM CÓ TEST THEO UNIT NĂM H...
BÀI TẬP BỔ TRỢ TIẾNG ANH I-LEARN SMART WORLD 9 CẢ NĂM CÓ TEST THEO UNIT NĂM H...
 
CHUYÊN ĐỀ BỒI DƯỠNG HỌC SINH GIỎI KHOA HỌC TỰ NHIÊN 9 CHƯƠNG TRÌNH MỚI - PHẦN...
CHUYÊN ĐỀ BỒI DƯỠNG HỌC SINH GIỎI KHOA HỌC TỰ NHIÊN 9 CHƯƠNG TRÌNH MỚI - PHẦN...CHUYÊN ĐỀ BỒI DƯỠNG HỌC SINH GIỎI KHOA HỌC TỰ NHIÊN 9 CHƯƠNG TRÌNH MỚI - PHẦN...
CHUYÊN ĐỀ BỒI DƯỠNG HỌC SINH GIỎI KHOA HỌC TỰ NHIÊN 9 CHƯƠNG TRÌNH MỚI - PHẦN...
 
BÀI TẬP BỔ TRỢ TIẾNG ANH GLOBAL SUCCESS LỚP 3 - CẢ NĂM (CÓ FILE NGHE VÀ ĐÁP Á...
BÀI TẬP BỔ TRỢ TIẾNG ANH GLOBAL SUCCESS LỚP 3 - CẢ NĂM (CÓ FILE NGHE VÀ ĐÁP Á...BÀI TẬP BỔ TRỢ TIẾNG ANH GLOBAL SUCCESS LỚP 3 - CẢ NĂM (CÓ FILE NGHE VÀ ĐÁP Á...
BÀI TẬP BỔ TRỢ TIẾNG ANH GLOBAL SUCCESS LỚP 3 - CẢ NĂM (CÓ FILE NGHE VÀ ĐÁP Á...
 
98 BÀI LUYỆN NGHE TUYỂN SINH VÀO LỚP 10 TIẾNG ANH DẠNG TRẮC NGHIỆM 4 CÂU TRẢ ...
98 BÀI LUYỆN NGHE TUYỂN SINH VÀO LỚP 10 TIẾNG ANH DẠNG TRẮC NGHIỆM 4 CÂU TRẢ ...98 BÀI LUYỆN NGHE TUYỂN SINH VÀO LỚP 10 TIẾNG ANH DẠNG TRẮC NGHIỆM 4 CÂU TRẢ ...
98 BÀI LUYỆN NGHE TUYỂN SINH VÀO LỚP 10 TIẾNG ANH DẠNG TRẮC NGHIỆM 4 CÂU TRẢ ...
 
GIÁO ÁN DẠY THÊM (KẾ HOẠCH BÀI BUỔI 2) - TIẾNG ANH 8 GLOBAL SUCCESS (2 CỘT) N...
GIÁO ÁN DẠY THÊM (KẾ HOẠCH BÀI BUỔI 2) - TIẾNG ANH 8 GLOBAL SUCCESS (2 CỘT) N...GIÁO ÁN DẠY THÊM (KẾ HOẠCH BÀI BUỔI 2) - TIẾNG ANH 8 GLOBAL SUCCESS (2 CỘT) N...
GIÁO ÁN DẠY THÊM (KẾ HOẠCH BÀI BUỔI 2) - TIẾNG ANH 8 GLOBAL SUCCESS (2 CỘT) N...
 
Nghiên cứu cơ chế và động học phản ứng giữa hợp chất Aniline (C6H5NH2) với gố...
Nghiên cứu cơ chế và động học phản ứng giữa hợp chất Aniline (C6H5NH2) với gố...Nghiên cứu cơ chế và động học phản ứng giữa hợp chất Aniline (C6H5NH2) với gố...
Nghiên cứu cơ chế và động học phản ứng giữa hợp chất Aniline (C6H5NH2) với gố...
 
50 ĐỀ LUYỆN THI IOE LỚP 9 - NĂM HỌC 2022-2023 (CÓ LINK HÌNH, FILE AUDIO VÀ ĐÁ...
50 ĐỀ LUYỆN THI IOE LỚP 9 - NĂM HỌC 2022-2023 (CÓ LINK HÌNH, FILE AUDIO VÀ ĐÁ...50 ĐỀ LUYỆN THI IOE LỚP 9 - NĂM HỌC 2022-2023 (CÓ LINK HÌNH, FILE AUDIO VÀ ĐÁ...
50 ĐỀ LUYỆN THI IOE LỚP 9 - NĂM HỌC 2022-2023 (CÓ LINK HÌNH, FILE AUDIO VÀ ĐÁ...
 
BÀI TẬP DẠY THÊM TOÁN LỚP 12 SÁCH MỚI THEO FORM THI MỚI BGD 2025 - CÁNH DIỀU ...
BÀI TẬP DẠY THÊM TOÁN LỚP 12 SÁCH MỚI THEO FORM THI MỚI BGD 2025 - CÁNH DIỀU ...BÀI TẬP DẠY THÊM TOÁN LỚP 12 SÁCH MỚI THEO FORM THI MỚI BGD 2025 - CÁNH DIỀU ...
BÀI TẬP DẠY THÊM TOÁN LỚP 12 SÁCH MỚI THEO FORM THI MỚI BGD 2025 - CÁNH DIỀU ...
 

Recently uploaded

Từ khoá Địa Lí giup ban dat 9 diem .docx
Từ khoá Địa Lí giup ban dat 9 diem .docxTừ khoá Địa Lí giup ban dat 9 diem .docx
Từ khoá Địa Lí giup ban dat 9 diem .docx
Nguyntrnhnganh
 
Diễn giải Tâm lý - Chiêm tinh Thầy Minh Tuệ
Diễn giải Tâm lý - Chiêm tinh Thầy Minh TuệDiễn giải Tâm lý - Chiêm tinh Thầy Minh Tuệ
Diễn giải Tâm lý - Chiêm tinh Thầy Minh Tuệ
Little Daisy
 
Tai-lieu-Boi-Duong-HSG-môn-Ngữ-Văn-THPT-Tập-1.docx
Tai-lieu-Boi-Duong-HSG-môn-Ngữ-Văn-THPT-Tập-1.docxTai-lieu-Boi-Duong-HSG-môn-Ngữ-Văn-THPT-Tập-1.docx
Tai-lieu-Boi-Duong-HSG-môn-Ngữ-Văn-THPT-Tập-1.docx
NhNguynTQunh
 
CD6_DAI_CUONG_KIMLOAI_12CB218LTTTHU5.pdf
CD6_DAI_CUONG_KIMLOAI_12CB218LTTTHU5.pdfCD6_DAI_CUONG_KIMLOAI_12CB218LTTTHU5.pdf
CD6_DAI_CUONG_KIMLOAI_12CB218LTTTHU5.pdf
Nguyntrnhnganh
 
Ngon_ngu_hoc_doi_chieu Các phạm trù cơ bản.pptx
Ngon_ngu_hoc_doi_chieu Các phạm trù cơ bản.pptxNgon_ngu_hoc_doi_chieu Các phạm trù cơ bản.pptx
Ngon_ngu_hoc_doi_chieu Các phạm trù cơ bản.pptx
linhlevietdav
 
Các bình diện Ngôn ngữ học đối chiếu.pdf
Các bình diện Ngôn ngữ học đối chiếu.pdfCác bình diện Ngôn ngữ học đối chiếu.pdf
Các bình diện Ngôn ngữ học đối chiếu.pdf
linhlevietdav
 
khoaluan_Chính Sách Tiền Lương Tại Công Ty Cổ Phần Đầu Tư Hải Đường, Tỉnh Nam...
khoaluan_Chính Sách Tiền Lương Tại Công Ty Cổ Phần Đầu Tư Hải Đường, Tỉnh Nam...khoaluan_Chính Sách Tiền Lương Tại Công Ty Cổ Phần Đầu Tư Hải Đường, Tỉnh Nam...
khoaluan_Chính Sách Tiền Lương Tại Công Ty Cổ Phần Đầu Tư Hải Đường, Tỉnh Nam...
Luận Văn Uy Tín
 
GƯƠNG ĐIỂN HÌNH NGƯỜI TỐT - VIỆC TỐT DƯƠNG THU NGA - NỮ KỸ THUẬT VIÊN PHỤC HỒ...
GƯƠNG ĐIỂN HÌNH NGƯỜI TỐT - VIỆC TỐT DƯƠNG THU NGA - NỮ KỸ THUẬT VIÊN PHỤC HỒ...GƯƠNG ĐIỂN HÌNH NGƯỜI TỐT - VIỆC TỐT DƯƠNG THU NGA - NỮ KỸ THUẬT VIÊN PHỤC HỒ...
GƯƠNG ĐIỂN HÌNH NGƯỜI TỐT - VIỆC TỐT DƯƠNG THU NGA - NỮ KỸ THUẬT VIÊN PHỤC HỒ...
lamluanvan.net Viết thuê luận văn
 
kl_HOÀN THIỆN CÔNG TÁC ĐÁNH GIÁ THỰC HIỆN CÔNG VIỆC TẠI CÔNG TY CỔ PHẦN ĐẦU T...
kl_HOÀN THIỆN CÔNG TÁC ĐÁNH GIÁ THỰC HIỆN CÔNG VIỆC TẠI CÔNG TY CỔ PHẦN ĐẦU T...kl_HOÀN THIỆN CÔNG TÁC ĐÁNH GIÁ THỰC HIỆN CÔNG VIỆC TẠI CÔNG TY CỔ PHẦN ĐẦU T...
kl_HOÀN THIỆN CÔNG TÁC ĐÁNH GIÁ THỰC HIỆN CÔNG VIỆC TẠI CÔNG TY CỔ PHẦN ĐẦU T...
Luận Văn Uy Tín
 
Những khó khăn của sinh viên năm nhất Học viện Hành chính Quốc gia trong quá ...
Những khó khăn của sinh viên năm nhất Học viện Hành chính Quốc gia trong quá ...Những khó khăn của sinh viên năm nhất Học viện Hành chính Quốc gia trong quá ...
Những khó khăn của sinh viên năm nhất Học viện Hành chính Quốc gia trong quá ...
lamluanvan.net Viết thuê luận văn
 
CHỮ “TRÍ” THEO TƯ TƯỞNG NHO GIÁO VÀ Ý NGHĨA TRONG ĐỔI MỚI GIAÓ DỤC Ở VIỆT NAM...
CHỮ “TRÍ” THEO TƯ TƯỞNG NHO GIÁO VÀ Ý NGHĨA TRONG ĐỔI MỚI GIAÓ DỤC Ở VIỆT NAM...CHỮ “TRÍ” THEO TƯ TƯỞNG NHO GIÁO VÀ Ý NGHĨA TRONG ĐỔI MỚI GIAÓ DỤC Ở VIỆT NAM...
CHỮ “TRÍ” THEO TƯ TƯỞNG NHO GIÁO VÀ Ý NGHĨA TRONG ĐỔI MỚI GIAÓ DỤC Ở VIỆT NAM...
lamluanvan.net Viết thuê luận văn
 
thuvienhoclieu.com-De-thi-thu-TN-THPT-2024-Tieng-Anh-phat-trien-tu-de-minh-ho...
thuvienhoclieu.com-De-thi-thu-TN-THPT-2024-Tieng-Anh-phat-trien-tu-de-minh-ho...thuvienhoclieu.com-De-thi-thu-TN-THPT-2024-Tieng-Anh-phat-trien-tu-de-minh-ho...
thuvienhoclieu.com-De-thi-thu-TN-THPT-2024-Tieng-Anh-phat-trien-tu-de-minh-ho...
HngNguyn2390
 
[NBV]-CHUYÊN ĐỀ 3. GTLN-GTNN CỦA HÀM SỐ (CÓ ĐÁP ÁN CHI TIẾT).pdf
[NBV]-CHUYÊN ĐỀ 3. GTLN-GTNN CỦA HÀM SỐ (CÓ ĐÁP ÁN CHI TIẾT).pdf[NBV]-CHUYÊN ĐỀ 3. GTLN-GTNN CỦA HÀM SỐ (CÓ ĐÁP ÁN CHI TIẾT).pdf
[NBV]-CHUYÊN ĐỀ 3. GTLN-GTNN CỦA HÀM SỐ (CÓ ĐÁP ÁN CHI TIẾT).pdf
NamNguynHi23
 
tiếng việt dành cho sinh viên ngoại ngữ h
tiếng việt dành cho sinh viên ngoại ngữ htiếng việt dành cho sinh viên ngoại ngữ h
tiếng việt dành cho sinh viên ngoại ngữ h
huynhanhthu082007
 
Atomic Habits - Thay Đổi Tí Hon, Hiệu Quả Bất Ngờ - James Clear & L...
Atomic Habits - Thay Đổi Tí Hon, Hiệu Quả Bất Ngờ - James Clear & L...Atomic Habits - Thay Đổi Tí Hon, Hiệu Quả Bất Ngờ - James Clear & L...
Atomic Habits - Thay Đổi Tí Hon, Hiệu Quả Bất Ngờ - James Clear & L...
williamminerva131
 
vật lý 1 k23 cuối kì c10-11- Nam Lê.pptx
vật lý 1 k23 cuối kì c10-11- Nam Lê.pptxvật lý 1 k23 cuối kì c10-11- Nam Lê.pptx
vật lý 1 k23 cuối kì c10-11- Nam Lê.pptx
LinhTrn115148
 
Nguyên Lý Kế Toán - Chủ đề 4 - 24D9BAN50601602
Nguyên Lý Kế Toán - Chủ đề 4 - 24D9BAN50601602Nguyên Lý Kế Toán - Chủ đề 4 - 24D9BAN50601602
Nguyên Lý Kế Toán - Chủ đề 4 - 24D9BAN50601602
PhcVngHunhTnh
 
Hành vi tình dục không an toàn và các yếu tố liên quan trong nhóm nam quan hệ...
Hành vi tình dục không an toàn và các yếu tố liên quan trong nhóm nam quan hệ...Hành vi tình dục không an toàn và các yếu tố liên quan trong nhóm nam quan hệ...
Hành vi tình dục không an toàn và các yếu tố liên quan trong nhóm nam quan hệ...
Man_Ebook
 
Tóm tắt Tư tưởng Hồ Chí Minhhhhhhhhhhhhh
Tóm tắt Tư tưởng Hồ Chí MinhhhhhhhhhhhhhTóm tắt Tư tưởng Hồ Chí Minhhhhhhhhhhhhh
Tóm tắt Tư tưởng Hồ Chí Minhhhhhhhhhhhhh
nnguyenthao204
 
DANH SÁCH XÉT TUYỂN SỚM_NĂM 2023_học ba DPY.pdf
DANH SÁCH XÉT TUYỂN SỚM_NĂM 2023_học ba DPY.pdfDANH SÁCH XÉT TUYỂN SỚM_NĂM 2023_học ba DPY.pdf
DANH SÁCH XÉT TUYỂN SỚM_NĂM 2023_học ba DPY.pdf
thanhluan21
 

Recently uploaded (20)

Từ khoá Địa Lí giup ban dat 9 diem .docx
Từ khoá Địa Lí giup ban dat 9 diem .docxTừ khoá Địa Lí giup ban dat 9 diem .docx
Từ khoá Địa Lí giup ban dat 9 diem .docx
 
Diễn giải Tâm lý - Chiêm tinh Thầy Minh Tuệ
Diễn giải Tâm lý - Chiêm tinh Thầy Minh TuệDiễn giải Tâm lý - Chiêm tinh Thầy Minh Tuệ
Diễn giải Tâm lý - Chiêm tinh Thầy Minh Tuệ
 
Tai-lieu-Boi-Duong-HSG-môn-Ngữ-Văn-THPT-Tập-1.docx
Tai-lieu-Boi-Duong-HSG-môn-Ngữ-Văn-THPT-Tập-1.docxTai-lieu-Boi-Duong-HSG-môn-Ngữ-Văn-THPT-Tập-1.docx
Tai-lieu-Boi-Duong-HSG-môn-Ngữ-Văn-THPT-Tập-1.docx
 
CD6_DAI_CUONG_KIMLOAI_12CB218LTTTHU5.pdf
CD6_DAI_CUONG_KIMLOAI_12CB218LTTTHU5.pdfCD6_DAI_CUONG_KIMLOAI_12CB218LTTTHU5.pdf
CD6_DAI_CUONG_KIMLOAI_12CB218LTTTHU5.pdf
 
Ngon_ngu_hoc_doi_chieu Các phạm trù cơ bản.pptx
Ngon_ngu_hoc_doi_chieu Các phạm trù cơ bản.pptxNgon_ngu_hoc_doi_chieu Các phạm trù cơ bản.pptx
Ngon_ngu_hoc_doi_chieu Các phạm trù cơ bản.pptx
 
Các bình diện Ngôn ngữ học đối chiếu.pdf
Các bình diện Ngôn ngữ học đối chiếu.pdfCác bình diện Ngôn ngữ học đối chiếu.pdf
Các bình diện Ngôn ngữ học đối chiếu.pdf
 
khoaluan_Chính Sách Tiền Lương Tại Công Ty Cổ Phần Đầu Tư Hải Đường, Tỉnh Nam...
khoaluan_Chính Sách Tiền Lương Tại Công Ty Cổ Phần Đầu Tư Hải Đường, Tỉnh Nam...khoaluan_Chính Sách Tiền Lương Tại Công Ty Cổ Phần Đầu Tư Hải Đường, Tỉnh Nam...
khoaluan_Chính Sách Tiền Lương Tại Công Ty Cổ Phần Đầu Tư Hải Đường, Tỉnh Nam...
 
GƯƠNG ĐIỂN HÌNH NGƯỜI TỐT - VIỆC TỐT DƯƠNG THU NGA - NỮ KỸ THUẬT VIÊN PHỤC HỒ...
GƯƠNG ĐIỂN HÌNH NGƯỜI TỐT - VIỆC TỐT DƯƠNG THU NGA - NỮ KỸ THUẬT VIÊN PHỤC HỒ...GƯƠNG ĐIỂN HÌNH NGƯỜI TỐT - VIỆC TỐT DƯƠNG THU NGA - NỮ KỸ THUẬT VIÊN PHỤC HỒ...
GƯƠNG ĐIỂN HÌNH NGƯỜI TỐT - VIỆC TỐT DƯƠNG THU NGA - NỮ KỸ THUẬT VIÊN PHỤC HỒ...
 
kl_HOÀN THIỆN CÔNG TÁC ĐÁNH GIÁ THỰC HIỆN CÔNG VIỆC TẠI CÔNG TY CỔ PHẦN ĐẦU T...
kl_HOÀN THIỆN CÔNG TÁC ĐÁNH GIÁ THỰC HIỆN CÔNG VIỆC TẠI CÔNG TY CỔ PHẦN ĐẦU T...kl_HOÀN THIỆN CÔNG TÁC ĐÁNH GIÁ THỰC HIỆN CÔNG VIỆC TẠI CÔNG TY CỔ PHẦN ĐẦU T...
kl_HOÀN THIỆN CÔNG TÁC ĐÁNH GIÁ THỰC HIỆN CÔNG VIỆC TẠI CÔNG TY CỔ PHẦN ĐẦU T...
 
Những khó khăn của sinh viên năm nhất Học viện Hành chính Quốc gia trong quá ...
Những khó khăn của sinh viên năm nhất Học viện Hành chính Quốc gia trong quá ...Những khó khăn của sinh viên năm nhất Học viện Hành chính Quốc gia trong quá ...
Những khó khăn của sinh viên năm nhất Học viện Hành chính Quốc gia trong quá ...
 
CHỮ “TRÍ” THEO TƯ TƯỞNG NHO GIÁO VÀ Ý NGHĨA TRONG ĐỔI MỚI GIAÓ DỤC Ở VIỆT NAM...
CHỮ “TRÍ” THEO TƯ TƯỞNG NHO GIÁO VÀ Ý NGHĨA TRONG ĐỔI MỚI GIAÓ DỤC Ở VIỆT NAM...CHỮ “TRÍ” THEO TƯ TƯỞNG NHO GIÁO VÀ Ý NGHĨA TRONG ĐỔI MỚI GIAÓ DỤC Ở VIỆT NAM...
CHỮ “TRÍ” THEO TƯ TƯỞNG NHO GIÁO VÀ Ý NGHĨA TRONG ĐỔI MỚI GIAÓ DỤC Ở VIỆT NAM...
 
thuvienhoclieu.com-De-thi-thu-TN-THPT-2024-Tieng-Anh-phat-trien-tu-de-minh-ho...
thuvienhoclieu.com-De-thi-thu-TN-THPT-2024-Tieng-Anh-phat-trien-tu-de-minh-ho...thuvienhoclieu.com-De-thi-thu-TN-THPT-2024-Tieng-Anh-phat-trien-tu-de-minh-ho...
thuvienhoclieu.com-De-thi-thu-TN-THPT-2024-Tieng-Anh-phat-trien-tu-de-minh-ho...
 
[NBV]-CHUYÊN ĐỀ 3. GTLN-GTNN CỦA HÀM SỐ (CÓ ĐÁP ÁN CHI TIẾT).pdf
[NBV]-CHUYÊN ĐỀ 3. GTLN-GTNN CỦA HÀM SỐ (CÓ ĐÁP ÁN CHI TIẾT).pdf[NBV]-CHUYÊN ĐỀ 3. GTLN-GTNN CỦA HÀM SỐ (CÓ ĐÁP ÁN CHI TIẾT).pdf
[NBV]-CHUYÊN ĐỀ 3. GTLN-GTNN CỦA HÀM SỐ (CÓ ĐÁP ÁN CHI TIẾT).pdf
 
tiếng việt dành cho sinh viên ngoại ngữ h
tiếng việt dành cho sinh viên ngoại ngữ htiếng việt dành cho sinh viên ngoại ngữ h
tiếng việt dành cho sinh viên ngoại ngữ h
 
Atomic Habits - Thay Đổi Tí Hon, Hiệu Quả Bất Ngờ - James Clear & L...
Atomic Habits - Thay Đổi Tí Hon, Hiệu Quả Bất Ngờ - James Clear & L...Atomic Habits - Thay Đổi Tí Hon, Hiệu Quả Bất Ngờ - James Clear & L...
Atomic Habits - Thay Đổi Tí Hon, Hiệu Quả Bất Ngờ - James Clear & L...
 
vật lý 1 k23 cuối kì c10-11- Nam Lê.pptx
vật lý 1 k23 cuối kì c10-11- Nam Lê.pptxvật lý 1 k23 cuối kì c10-11- Nam Lê.pptx
vật lý 1 k23 cuối kì c10-11- Nam Lê.pptx
 
Nguyên Lý Kế Toán - Chủ đề 4 - 24D9BAN50601602
Nguyên Lý Kế Toán - Chủ đề 4 - 24D9BAN50601602Nguyên Lý Kế Toán - Chủ đề 4 - 24D9BAN50601602
Nguyên Lý Kế Toán - Chủ đề 4 - 24D9BAN50601602
 
Hành vi tình dục không an toàn và các yếu tố liên quan trong nhóm nam quan hệ...
Hành vi tình dục không an toàn và các yếu tố liên quan trong nhóm nam quan hệ...Hành vi tình dục không an toàn và các yếu tố liên quan trong nhóm nam quan hệ...
Hành vi tình dục không an toàn và các yếu tố liên quan trong nhóm nam quan hệ...
 
Tóm tắt Tư tưởng Hồ Chí Minhhhhhhhhhhhhh
Tóm tắt Tư tưởng Hồ Chí MinhhhhhhhhhhhhhTóm tắt Tư tưởng Hồ Chí Minhhhhhhhhhhhhh
Tóm tắt Tư tưởng Hồ Chí Minhhhhhhhhhhhhh
 
DANH SÁCH XÉT TUYỂN SỚM_NĂM 2023_học ba DPY.pdf
DANH SÁCH XÉT TUYỂN SỚM_NĂM 2023_học ba DPY.pdfDANH SÁCH XÉT TUYỂN SỚM_NĂM 2023_học ba DPY.pdf
DANH SÁCH XÉT TUYỂN SỚM_NĂM 2023_học ba DPY.pdf
 

ĐỀ THI THỬ TUYỂN SINH VÀO LỚP 10 THPT MÔN TOÁN CÁC TỈNH NĂM HỌC 2023-2024 CÓ LỜI GIẢI CHI TIẾT.pdf

  • 1. ĐỀ THI THỬ TUYỂN SINH VÀO LỚP 10 THPT MÔN TOÁN CÁC TỈNH NĂM HỌC 2023-2024 CÓ LỜI GIẢI CHI TIẾT WORD VERSION | 2023 EDITION ORDER NOW / CHUYỂN GIAO QUA EMAIL TAILIEUCHUANTHAMKHAO@GMAIL.COM Đ Ề L U Y Ệ N T H I T U Y Ể N S I N H L Ớ P 1 0 M Ô N T O Á N Ths Nguyễn Thanh Tú eBook Collection Hỗ trợ trực tuyến Fb www.facebook.com/DayKemQuyNhon Mobi/Zalo 0905779594 Tài liệu chuẩn tham khảo Phát triển kênh bởi Ths Nguyễn Thanh Tú Đơn vị tài trợ / phát hành / chia sẻ học thuật : Nguyen Thanh Tu Group vectorstock.com/28062405
  • 2. PHÒNG GIÁO DỤC VÀ ĐÀO TẠO ĐÔNG ANH ĐỀ KIỂM TRA KHẢO SÁT LỚP 9 NĂM HỌC 2023 – 2024 Môn: TOÁN Thời gian làm bài: 120 phút Bài I ( 2,0 điểm ) Cho hai biểu thức 3 5 x A x − = + và 4 2 13 9 3 3 x x x B x x x − − = + + − + − với 0; 9 x x   1) Tính giá trị của biểu thức A khi 25 x = 2) Rút gọn biểu thức B. 3) Cho . P B A = . Tìm các giá trị của x thỏa mãn ( ) 1 2 3 . 3 5 x x P x − = + + + Bài II ( 2,5 điểm ) 1) Giải bài toán sau bằng cách lập phương trình hoặc hệ phương trình: Một người đi xe đạp từ A đến B cách nhau 24 km. Khi từ B về A người đó tăng vận tốc thêm 4 km/h so với lúc đi. Vì vậy thời gian về ít hơn thời gian đi là 30 phút. Tính vận tốc của người đi xe đạp đó khi đi từ A đến B?. 2) Một hộp sữa ông thọ hình trụ có chiều cao là 8cm và bán kính đáy là 3,5 cm. Nhà sản xuất đã dán giấy xung quanh hộp sữa để ghi các thông tin về sản phẩm. Hãy tính diện tích giấy cần dùng cho 1 hộp sữa ( Coi mép giấy dán, các mép của hộp sữa và độ dày của giấy in không đáng kể. Lấy 3,14   ). Bài III (2,0 điểm ) 1) Giải hệ phương trình sau 5 3 16 2 5 4 x y x y  − + =   − − =   2) Cho phương trình 2 2 0 x mx − − = ( x là ẩn số). a) Tìm giá trị của mđể phương trình đã cho có hai nghiệm phân biệt. b) Tìm giá trị của mđể phương trình đã cho có hai nghiệm phân biệt 1 2 , x x thỏa mãn 2 2 1 2 8 x x + = Bài IV ( 3,0 điểm ): Cho đường tròn tâm ( ) O đường kính . Dây CD vuông góc với đường kính AB tại H,( H khác O, A và B). E là một điểm thuộc cung nhở BD ( E khác B và D); AE cắt CD tại F. 1) Chứng minh: Tứ giác BEFH nội tiếp đường tròn. 2) Chứng minh:H là trung điểm của CD và 2 . 4. CD AH HB = 3) Đường thẳng qua H song song với CE cắt đường thẳng AE và BE lần lượt tại I và K. Lấy G là trung điểm của đoạn thẳng IK. Hỏi ∆DGK có là tam giác cân được hay không? Bài V ( 0,5 điểm ) Cho x, y, z > 0 và x.y.z = 1 Tìm giá trị nhỏ nhất của biểu thức 2 2 2 1 1 1 x y z A y z x = + + + + + …………. Hết ………… Cán bộ coi thi không giải thích gì thêm
  • 3. HƯỚNG DẪN CHẤM KIỂM TRA KHẢO SÁT TOÁN 9 NĂM HỌC 2023 – 2024 Môn: toán A. Hướng dẫn chung - Học sinh giải theo cách khác mà đúng, đủ các bước thì giám khảo chấm điểm tối đa. - Trong mỗi bài, nếu ở một bước nào đó bị sai thì các bước sau có liên quan không được điểm. - Bài hình học bắt buộc phải vẽ đúng hình thì mới chấm điểm, nếu không có hình vẽ đúng ở phần nào thì giám khảo không ghi điểm phần lời giải liên quan đến hình của phần đó. - Điểm toàn bài là tổng điểm của các ý, các câu, tính đến 0,25 điểm và không làm tròn. B. Nội dung, biểu điểm BÀI Ý SƠ LƯỢC LỜI GIẢI ĐIỂM Bài I (2,0 điểm) 1 (0,5đ) 1) Thay x = 25 (TMĐK) vào biểu thức A ta đc : 25 3 25 5 A − = + 0,25 1 5 A = 0,25 2 (1đ) 4 2 13 9 3 3 x x x B x x x − − = + + − + − 0,25 ( )( ) 4 2 13 3 3 3 3 x x x B x x x x − − = + − + − + − 0,25 4 12 2 13 3 3 x x x x x B x − + − − − − = + 0,25 ( )( ) 25 3 3 x B x x − = + − 0,25 3 (0,5đ) ( ) ( ) ( ) ( ) ( ) 2 2 5 . 3 1 2( 3). 3 5 0; 9 1 2 10 3 5 9 2 3 5 0 2 18 4 6 5 0 5 6 5 9 4 4 0 5 3 2 0 x P A B x x x P x x x x x x x x x x x x x x x x x x − = = + − = + + +    − = − + +  + − − + =  + − − + =  + − + + + − + =  + − + − = Giải ra tìm được x = 4 ( thỏa mãn ĐKXĐ). Vậy x = 4 0,25 0,25 Bài II (2,5 điểm) 1 (2đ) Gọi vận tốc của người đi xe đạp khi đi từ A đến B là x km/h ĐK x >0 0,25 Thời gian của người đi xe đạp khi đi từ A đến B là: 24 ( ) h x 0,25 Vận tốc của người đi xe đạp khi đi từ B về A là: x + 4 (km/h) 0,25 Thời gian của người đi xe đạp khi đi từ B đến A là 24 ( ) 4 h x + 0,25
  • 4. Vì thời gian về ít hơn thòi gian đi là 30 phút bằng 1 2 giờ nên ta có phương trình : 24 24 1 4 2 x x − = + 0,25 2 4 192 0 x x  + − = 0,25 12( ) 16( ) x TM x KTM = = − Vậy vận tốc của người đi xe đạp từ A đến B là 12 km/h 0,25 1 (0,5đ) 2) 2 2 2.3,14.3,5.8 175,84 xq S Rh cm  =  = 0,25 Vậy diện tích của phần giấy cần dùng khoảng 175,84cm2 0,25 Bài III (2,0 điểm) 1 (1đ) 5 3 16 2 5 4 x y x y  − + =   − − =   2 5 6 32 2 5 4 x y x y  − + =    − − =   Đk; : 5 x  0,25 7 28 4 5 3 16 5 16 12 4 y y x y x = =        − + = − = − =     0,25 4 5 16 y x =    − =  0,25 4 21( ) y x TM =    =  Vậy hệ pt có nghiệm duy nhất là (21;4) 0,25 2 (1đ) a) Tính 2 8 m  = + 0   với mọi m 0,25 Suy ra phương trình có hai nghiệm phân biệt x1, x2 với mọi m 0,25 b) Áp dụng định lý Vi-ét ta có 1 2 1 2 . 2 x x m x x + =   = −  Để ( ) 2 2 2 1 2 1 2 1 2 8 2 . 8 x x x x x x + =  + − = Suy ra 2 4 8 m + = 0,25 2 4 2 m m  =  =  Vậy 2 m =  là giá trị cần tìm 0,25 Bài IV (3,0 điểm) Vẽ hình đúng đến ý a 0,25
  • 5. 2 (0,75đ) Xét đường tròn đường kính AB có 0 90 AEB = ( Góc nội tiếp chắn nửa đường tròn đường kính AB) 0 90 BEF  = CD AB ⊥ tại H 0 90 BHF  = 0,75 Xét tứ giác BEFH có 0 180 BEF BHF + = 0,25 Mà chúng ở vị trí đối diện nhau nên tứ giác BHFE là tứ giác nội tiếp đường tròn 0,25 2 (1đ) Xét đường tròn (O) có AB là đường kính, AB CD ⊥ tại H (gt) => CH = HD ( quan hệ đường kính, dây cung) => H là trung điểm của đoạn thẳng CD 0,25 0,25 Xét (O) có 0 90 ACB = ( góc nội tiếp chắn nửa đường tròn) Xét ABC  vuông tại C, có CH là đường cao 2 . CH AH HB  = 0,25 Mà 2 CD CH = ( H là trung điểm của CD) Nên 2 4 . CD AH HB  = 0,25 3 (1đ) / / HI CE DHI DCE  = ( hai góc so le trong) Xét (O) có DAE DCE = ( hai góc nội tiếp cùng chắn cung DE) DHI DAE DHI DAI  =  = DAHI có Xét tứ giác DAHI có DHI DAI = Mà H, A là hai đỉnh kê nhau cùng nhìn cạnh DI => Tứ giác AHID nội tiếp đường tròn 0,25 Xét đường tròn ngoại tiếp tứ giác AHID, có: AHD AID = ( hai góc nội tiếp cùng chắn cung AD) Mà 0 0 90 90 AHD AID = = = 0,25 Xét (O) có DBE DAE = ( hai góc nội tiếp cùng chắn cung DE) ( ) DAE DAI DHI cmt DHI DBE = = = = Hay DHK DBK = => Tứ giác DHBK nội tiếp đường tròn
  • 6. => 0 180 DHB DKB + = Mà 0 0 0 90 90 90 DHB DKB DKE = = = = = 0,25 Xét tứ giác DIEK có 0 90 DIE IEK DKE = = = => Tứ giác DIEK là hình chữ nhật => IK = DE và IK, DE cắt nhau tại trung điểm G của mỗi đường Mà GD = GK => ∆DGK cân tại G 0,25 Bài V (0,5 điểm) Áp dụng BĐT Cauchy cho hai số dương 2 2 2 2 2 2 1 1 2 . 1 4 1 4 1 1 2 . 1 4 1 4 1 1 2 . 1 4 1 4 x y x y x y y y z y z y z z z x z x z x x  + + +  =  + +   + +  +  =  + +   + +  +  = + +   Cộng vế với vế ba BĐT trên ta được ( ) 2 2 2 2 2 2 2 2 2 3 1 1 1 1 4 1 4 1 4 3 1 1 1 4 4 3 3 3 3 3 .3. 1 1 1 4 4 4 4 2 x y y z z x x y z y z x x y z x y z x y z y z x x y z x y z xyz y z x       + + + + + + + +  + +       + + +       + +  + +  − − + + + + + + + +  + +  −  − = + + + 3 2 A   0,25 Dấu “=” xảy ra 1( ) x y z TM  = = = Vậy giá trị nhỏ nhất của A là 3 2 khi 1 x y z = = = 0,25 Lưu ý: Các cách làm khác nếu đúng vẫn cho điểm tối đa
  • 7. PHÒNG GIÁO DỤC VÀ ĐÀO TẠO QUẬN ĐỐNG ĐA ĐỀ KHẢO SÁT CHẤT LƯỢNG LỚP 9 NĂM HỌC 2023 – 2024 Môn: TOÁN Ngày khảo sát; 17/4/2024 Thời gian làm bài: 120 phút Bài I ( 2,0 điểm ) Cho hai biểu thức 2 6 1 x A x + = − và 2 2 2 x x B x x x + = − + + − với 0, 1. x x   1) Tính giá trị của biểu thức A khi 9. x = 2) Rút gọn B. 3) Chứng minh 6 A B  Bài II ( 2,5 điểm ) 1) Giải bài toán sau bằng cách lập phương trình hoặc hệ phương trình: Một ô tô đi từ A đến B dài 90km. Khi về ô tô đi theo đường khác dài hơn 10km và mỗi giờ ô tô đi được nhiều hơn lúc đi 10km nên thời gian về ít hơn thời gian đi là 15 phút. Tính vận tốc lúc đi và lúc về?. 2) Một lon nước ngọt hình trụ có đường kính đáy là 6cm, độ dài trục là 11cm. Tính thể tích lon nước ngọt ( cho 3,14   ). Bài III (2,0 điểm ) 1) Giải hệ phương trình sau 1 4 3 2 1 5 3 2 5 2 1 5 x y x y  + =  − +    − = −  − +  2) Trong mặt phẳng toạ độOxy , cho parabol ( ) 2 : P y x = và đường thẳng ( ): 1 d y mx m = − + (mlà tham số ). a) Tìm mđể ( ) d cắt ( ) P tại 2 điểm phân biệt. b) Gọi 1 2 , x x là hoành độ giao điểm của ( ) d và ( ) P tìm tất cả các giá trị của m để 2 1 2 x x = Bài IV ( 3,0 điểm ): Cho đường tròn tâm ( ) O đường kính ABvà d là một tiếp tuyến của đường tròn ( ) O tại điểm A. Trên đường thẳng d lấy điểm M (khác A) và trên đoạn OBlấy điểm N (khác O và B). Đường thẳng MN cắt đường tròn ( ) O tại hai điểm và D (C nằm giữa M và D ). Gọi H là trung điểm của đoạn thẳngCD . 1) Chứng minh 4 điểm , , , A O H M cùng nằm trên một đường tròn. 2) Chứng minh 2 . MA MD MC = 3) Đường thẳng qua D song song với MOcắt ABvà BC lần lượt tại K và F . Chứng minh tứ giác AHKDnội tiếp và K là trung điểm của đoạn thẳng DF . Bài V ( 0,5 điểm ) Cho hai số thực a và b thoả mãn điều kiện 2 ( 1) a b ab + − = . Tìm giá trị nhỏ nhất của biểu thức 9 P a b a b = + + + …………. Hết ………… Cán bộ coi thi không giải thích gì thêm
  • 8. HƯỚNG DẪN CHẤM KHẢO SÁT TOÁN 9 NĂM HỌC 2023 – 2024 (Ngày khảo sát 17.4.2024) BÀI SƠ LƯỢC LỜI GIẢI ĐIỂM Bài I (2,0 điểm) Cho hai biểu thức 2 6 1 x A x + = − và 2 2 2 x x B x x x + = − + + − với 0, 1. x x   2,0 1) Tính giá trị biểu thức A khi x = 9. 0,5 + Thay x = 9 (TMĐK) vào biểu thức A ta đc : 2 9 6 12 3 9 1 8 2 A + = = = − Vậy giá trị của biểu thức 3 2 A = khi 9 x = 0,25 0,25 2) Rút gọn B(đk 0, 1. x x   ) 0,5 Với điều kiện 0, 1 x x   ta có: 2 2 2 ( 2)( 1) 2 2 x x x x B x x x x x x + + = − = − + + − + + − 0,25 2 ( 1) 2 ( 2)( 1) ( 2)( 1) x x x x x x x x x x + − − + − + = = + − + − 0,25 2 ( 2)( 1) x x x + = + − 0,25 1 1 x = − với 0, 1 x x   0,25 3) Chứng minh 6 A B  0,5 Ta có 2 6 4 2 1 1 A x B x x + = = + + + 0,25 4 2 2 4 6 1 x +  + = + với 0, 1 x x   0,25 Bài II (2,5 điểm) 1) Một ô tô đi từ A đến B dài 90km. Khi về ô tô đi theo đường khác dài hơn 10km và mỗi giờ ô tô đi được nhiều hơn lúc đi 10km nên thời gian về ít hơn thời gian đi là 15 phút. Tính vận tốc lúc đi và lúc về?. 2đ + Gọi vận tốc ô tô lúc đi từ A đến B là x (km/h; x >0); đổi 15 phút = 1 4 (h) 0,25 + Thời gian ô tô đi từ A đến B là: 90 ( ) h x 0,25 + Vận tốc ô tô đi từ B về A là : x + 10 (km/h) 0,25 + Thời gian ô tô đi từ B về A là 90 10 100 ( ) 10 10 h x x + = + + 0,25 => Ta có phương trình : 90 100 1 10 4 x x − = + 0,25 + Giải phương trình thu được x = 40 (tmđk) và x = - 90 (loại) 0,5 + Kết luận 0,25
  • 9. 2) Một lon nước ngọt hình trụ có đường kính đáy là 6cm, độ dài trục là 11cm. Tính thể tích lon nước ngọt ( cho 3,14   ). 0,5 Ta có R= 3cm, h=11cm nên ( ) 2 3 V = R h = 3,14.9.11= 310,86 cm  0,25 Vậy thể tích của lon nước ngọt là 310,86 cm3 0,25 Bài III (2,0 điểm) 1. Giải hệ : 1 4 3 2 1 5 3 2 5 2 1 5 x y x y  + =  − +    − = −  − =  1,0 + Đk; : 1 ; 5 2 x y   − Đặt 1 2 1 1 5 a x b y  =  −    = +   0,25 + Giải được 1; 1 a b = − = 0,25 Thay 1 1 2 1 1 0 2 1 x x x + =  − = −  = − ( TMĐK) 1 1 5 1 4 5 y y y + =  + =  = − + ( TMĐK) 0,25 + Kết luận 0,25 2) Trong mặt phẳng toạ độOxy , cho parabol ( ) 2 : P y x = và đường thẳng ( ): 1 d y mx m = − + ( mlà tham số ). 1,0 a) Tìm mđể ( ) d cắt ( ) P tại 2 điểm phân biệt. 0,5 Xét phương trình hoành độ giao điểm của đường thẳng ( ) d và parabol ( ) P 2 2 1 1 0 1 1 0 (1) x mx m x mx m a b c m m = − +  − + − = + + = − + − = Nên phương trình (1) có nghiệm là 1 x = và 1 x m = − 0,25 + Vì ( ) d cắt ( ) P tại 2 điểm phân biệt nên 1 1 2 m m  −   0,25 b) 1 2 , x x thỏa mãn 2 1 2 x x = 0,5 TH1: ( ) 2 2( ) 1 1 0( ) m TM m m TM =  = −   =  Vậy m = 0 thì…. 0,25 TH2: 2 1 1 2( ) m m TM = −  = 0,25 Bài IV (3,0 điểm) Vẽ hình đúng đến câu 1
  • 10. 0,25 1) Chứng minh 4 điểm , , , A O H M cùng nằm trên một đường tròn. 0,75 Vì MA là tiếp tuyến của 0 ( ) 90 O MAO  = H là trung điểm của CD OH CD  ⊥ tại H 0 90 OHC OHM  = = 0,25 Xét tứ giác AOHM có: 0 0 0 90 90 180 MAO OHM + = + = mà hai góc này là hai góc đối nhau nên tứ giác AOHM là tứ giác nội tiếp 0,25 Suy ra 4 điểm , , , A O H M cùng nằm trên một đường tròn. (đpcm) 0,25 2) Chứng minh 2 . MA MD MC = 1,0 Xét ∆ AMC và ∆ DMA có AMDchung 0,25 +) MDA MAC = ( góc nội tiếp và góc tạo bởi tiếp tuyến với dây cung cùng chắn cung AC) 0,25 ∆ AMC∽∆ DMA( g – g) 0,25 2 . AM MC MA MC MD DM MA  =  = (đpcm) 0,25 3) Chứng minh tứ giác AHKDnội tiếp và K là trung điểm của đoạn thẳng DF 1,0 Ta có: / / ( ) DK MO gt MDK DMO  = ( hai góc so le trong) Vì tứ giác AOHM là tứ giác nội tiếp (cm câu 1) HMO HAO  = ( góc nội tiếp cùng chắn cung HO ) ( ) BAH MDK DMO  = = 0,25 Tứ giác AHKDlà tứ giác nội tiếp (hai đỉnh kề cùng nhìn cạnh đối diện dưới các góc bằng nhau) 0,25 Tứ giác AHKD nội tiếp DAK DHK = (góc nội tiếp cùng chắn DK) DAB DCB = (góc nội tiếp cùng chắn BD) DCB DHK  = 0,25 Mà ; DCB DHK ở vị trí đồng vị nên / / / / HK CB HK CF  . Mặt khác H là trung điểm của CD K  là trung điểm của DF 0,25 Bài V (0,5 điểm) Cho hai số thực a và b thoả mãn điều kiện 2 ( 1) a b ab + − = . Tìm giá trị nhỏ nhất của biểu thức 9 P a b a b = + + + 0,5 ( ) ( ) ( ) 2 2 9 4 5 5 5 13 4 2 . 4 2 2 2 1 2 2 3 P a b a b a b a b a b a b a b a b a b ab a b a b = + = + + +  + +  + = + + + + + +   + −     +      + 0,25 Dấu “=” có 13 2 P = khi 1 a b = =
  • 11. KL: 13 min 2 P = khi 1 a b = = 0,25 Lưu ý: Các cách làm khác nếu đúng vẫn cho điểm tối đa
  • 12. UBND QUẬN HỒNG BÀNG TRƯỜNG THCS HỒNG BÀNG ĐỀ THI THỬ VÀO LỚP 10 THPT Năm học 2023 – 2024 Môn: Toán 9 Thời gian làm bài: 120 phút Bài 1. (1,5 điểm) Cho ( ) 2 2 1 1 1 3 2 3 1; : 1 2 1 x x A B x x x x     − + = − + − = − −     − + −     với 0, 1. x x   a) Rút gọn biểu thức A, B . 2) Tìm các giá trị của x để giá trị của biểu thức B bằng giá trị của biểu thức A Bài 2. ( 1,5 điểm ) 1. Cho hàm số 2 1 y mx m = + − có đồ thị là (d) . Tìm giá trị của m biết rằng (d) cắt đường thẳng 3 2 y x = − tại điểm có hoành độ bằng 2. 2. Một mảnh vườn hình chữ nhật có các kích thước là 20m và 30m. Người ta tăng các kích thước của nó lên mỗi chiều x(m) được hình chữ nhật mới có chu vi là y (m). a) Hãy lập hàm số y theo x. b) Nếu chu vi của mảnh vườn sau khi mở rộng là 200m. Hãy tính các kích thước của mảnh vườn sau khi mở rộng Bài 3. (2,5 điểm ) 1. Cho phương trình ( ) 2 2 3 0 (1) x m x m + − + − = ( m là tham số) a) Chứng minh với mọi m phương trình (1) luôn có nghiệm. b) Tìm m để phương trình (1) có hai nghiệm âm phân biệt. 2. Một phòng họp có 360 ghế và được chia thành các dãy có số ghế bằng nhau. Nếu thêm cho mỗi dãy 4 ghế và bớt đi 3 dãy thì số ghế trong phòng không thay đổi. Hỏi ban đầu có bao nhiêu dãy ghế và trong mỗi dãy có bao nhiêu ghế? Bài IV ( 0,75 điểm ): Một bồn đựng nước dạng hình hộp chữ nhật có các kích thước trên hình bên dưới. a) Tính diện tích bề mặt của bồn (không tính nắp). b) Một vòi bơm công suất 120lít/phút để bơm một lượng nước vào bồn lên độ cao cách nắp bồn là 1,5m thì phải mất bao nhiêu phút? (cho biết ban đầu bồn không chứa nước). Bài V ( 3 điểm ) Từ điểm A nằm ngoài (O) , vẽ hai tiếp tuyến AB, AC ( B, C là hai tiếp điểm), gọi H là giao điểm của OA và BC . a) Chứng minh tứ giác OBAC nội tiếp và OA BC ⊥ b) Kẻ đường kính BK của (O), cho AK cắt (O) tại điểm E ( E K  ) Chứng minh 2 . AB AE AK = và tứ giác OHEK nội tiếp. c) Tia BK và tia AC cắt nhau tại F, kẻ ( ) CI BK I BK ⊥  , AK và CI cắt nhau tại M. Gọi N là trung điểm của AB. Chứng minh ba điểm F, M, N thẳng hàng. Bài VI (0,75 đ) Cho x, y là các số dương và x + y = 2. Chứng minh 2 2 1 1 3 4 P x y x y   = + +      …………. Hết ………… Họ và tên thí sinh: ………………………….. Số báo danh: …………………………….... Giám thị 1: …………………………………... Giám thị 2: ………………..………………
  • 13. ĐÁP ÁN VÀ BIỂU ĐIỂM KỲ ĐỀ THI THỬ LẦN 1 MÔN TOÁN LỚP 9 BÀI SƠ LƯỢC LỜI GIẢI ĐIỂM Bài 1 (1,5 điểm) a) 1,0 điểm ( ) 2 3 2 3 1 3 2 3 1 A = − + − = − + − 0,25 2 3 3 1 1 = − + − = 0,25 ( )( ) ( )( ) ( ) 4 1 : 1 . 2 1 . 2 1 1 x x B x x x x x x   − −   = −   − + − +   − − 0,25 ( ) ( ) ( ) 1 3 3 . 1 2 1 . 2 x x x B x x x − − =  = − + − + 0,25 b) 0,5 điểm Cho 3 1 2 x A B x =  = + 0,25 3 2 2 2 1 1 x x x x x  = +  =  =  = ( không thỏa mãn) 0,25 Bài 2 (1,5 điểm) a) 0,75 điểm Để (d) 2 1 y mx m = + − cắt đường thẳng 3 2 y x = − thì 3 2 3 (*) 2 m m    Gọi điểm có hoành độ bằng 2 là 0 (2; ) A y Vì 0 (2; ) A y thuộc 3 2 y x = − nên 0 3.2 2 4 y = − = Do đó (2;4) A 0,25 Vì (2;4) A thuộc (d) nên 4 2 .2 1 m m = + − 5 5 1 m m  =  = (thỏa mãn điều kiện (*)) 0,25 Vậy đường thẳng (d) cắt đường thẳng 3 2 y x = − tại điểm có hoành độ bằng 2 khi m = 1 0,25 b) 0,75 điểm Chu vi mảnh vườn là ( ) 2 20 30 4 100 y x x x = + + + = + 0,25 Chu vi là 200m nên ta có 4 100 200 25 x x + =  = 0,25 Vậy kích thước của mảnh vườn sau khi mở rộng là 45m và 55m 0,25 Bài 3 (2,0 điểm) 1a ( 0,5 điểm) ( ) 2 ) 2 3 0 (1) a x m x m + − + − = ( m là tham số) ( ) ( ) ( ) 2 2 2 2 4.1. 3 8 16 4 0 m m m m m m R  = − − − = − + = −    0,25 Phương trình có nghiệm với mọi m 0,25 1b ( 1,0 điểm) ( ) ( )( ) 2 ) 2 3 0 1 3 0 b x m x m x x m + − + − =  − + − = Do đó phương trình (1) có nghiệm 1 2 1; 3 x x m = − = − 0,25 Vì 1 1 0 x = −  nên phương trình (1) có hai nghiệm phân biệt 0,5 2 2 1 0 3 0 3 3 1 4 x m m x x m m  −             −  −     0,25 Vậy với 3; 4 m m   thì phương trình đã cho có hai nghiệm âm phân biệt. 0,25
  • 14. 2. (1,0 điểm) Gọi x là số dãy ghế trong phòng họp ( x > 3 và x là số nguyên dương) 0,25 Số ghế trên một dãy là 360 x (ghế) nếu bớt đi 3 dãy thì số dãy ghế còn lại là 3 x − (dãy) Số ghế trên một dãy là 360 3 x − (ghế) 0,25 Do số ghế trong mỗi dãy tăng thêm 4 ghế nên ta có phương trình 2 18( ) 360 360 4 3 270 0 15( ) 3 x tm x x x ktm x x =  − =  − − =   = − −  0,25 Vậy số dãy ghế là 18 (dãy) và số ghế trong một dãy là 20 (ghế) 0,25 Bài 4 (0,75 điểm) Diện tích bề mặt của bồn : ( ) 2 3,1.2,3 11,5.2,3 .2 11,5.3,1 102,81( ) S m = + + = Một vòi bơm công suất 120 lít/phút để bơm một lượng nước vào bồn lên độ cao cách nắp bồn là 1,5m thì phải mất bao lâu? (bồn không chứa nước). Thể tích của nước cần bơm là: 3 11,5.3,1.(2,3 1,5) 28,52( ) V m = − = Lại có 3 1 1000 m l = vậy 3 28,52( ) 28520 m = ( lít) Lại có, 1 phút bơm được 120 lít nên 28520( lít) sẽ bơm trong 28520 237,67 120  ( phút) Bài 5 (3,0 điểm) Vẽ hình đúng đến câu a 0,25 a) 1,0 điểm Chứng minh tứ giác OBAC nội tiếp và OA BC ⊥ 0 90 OBA OCA = = ( AB, AC là tiếp tuyến của (O)) 0,25 0 180 OBA OCA  + = Vậy tứ giác OBAC nội tiếp (tứ giác có tổng hai góc đối bằng 1800 ) 0,25 Ta có OB = OC (bán kính (O) ) AB = AC (tính chất hai tiếp tuyến cắt nhau) 0,25 => OA là đường trung trực của BC => OA BC ⊥ 0,25 b) 1,0 điểm Chứng minh 2 . AB AE AK = và tứ giác OHEK nội tiếp. 0 90 BEK = ( (góc nội tiếp chắn nửa đường tròn (O) ) => BE AK ⊥ 0,25 Xét ABK  vuông tại B, có đường cao BE 2 . AE AK AB = (hệ thức lượng trong tam giác vuông). 0,25 Xét ABO  vuông tại B, có đường cao BH: 2 . AH AO AB = (hệ thức lượng trong tam giác vuông).
  • 15. 2 . . ( ) AH AO AE AK AB = = = 0,25 Xét AHE  và AKO  có OAK chung, AH AE AK AO = ( vì . . AH AO AE AK = ) ( ) AHE AKO cgc AHE AKO   =  ∽ Vậy tứ giác OHEK nội tiếp (tứ giác có góc ngoài bằng góc đối trong) 0,25 c) 0,75 điểm Chứng minh: ba điểm F, M, N thẳng hàng. 1,0 Gọi S là giao điểm của KC và BA 0 90 BCK = (góc nội tiếp chắn nửa đường tròn (O) ) BC SK  ⊥ BKS  có O là trung điểm của BK; OA//KS ( cùng vuông góc với BC) => A là trung điểm của BS => AB = AS / / ( ) IM KM IM AB BK BA KA ⊥ = = (hệ quả Talet trong KBA  ) / / ( ) CM KM CM AS BK AS KA ⊥ = = (hệ quả Talet trong KSA  ) IM CM KM BA AS KA   = = =     Mà BA = AS nên IM = CM => M là trung điểm của IC 0,25 Gọi FM cắt BA tại N’. / / ' ( ) ' ' IM FM IM N B BK BN FN ⊥ = = (hệ quả Talet trong ' FBN  ) / / '( ) ' ' CM FM CM AN BK AN FN ⊥ = = (hệ quả Talet ) ' ' ' IM CM FM BN AN FN   = = =     0,25 Mà IM = CM nên BN’ = AN’=> N’ là trung điểm của BA. Mà N là trung điểm của BA nên N trùng N’ Mà N’, F, M thẳng hàng nên N, F, M thẳng hàng 0,25 Bài 6 (0,75 điểm) Áp dụng AM-GM, ta có 2 2 2 2 1 1 1 . 3 2 . 3 P x y x y x y xy   = + +  +     0,5 Áp dụng AM-GM, ta có 2 2 1 x y xy xy = +  =  0,25 Suy ra 3 1 2 1 2 2 2 2 2 . 4 P xy xy xy xy xy xy xy xy  + = + +  +  (đpcm) Dấu “=” xảy ra khi 1 x y = = 0,25
  • 16. UBND QUẬN BA ĐÌNH PHÒNG GIÁO DỤC VÀ ĐÀO TẠO ĐỀ CHÍNH THỨC ĐỀ KHẢO SÁT CHẤT LƯỢNG LỚP 9 NĂM HỌC 2022 – 2023 Môn: TOÁN 9 Ngày kiểm tra: 10/3/2023 Thời gian làm bài: 120 phút Bài I ( 2,0 điểm ) Cho hai biểu thức 2 x A x + = và 3 4 1 2 2 x x B x x x − + = − − − với 0; 4 x x   1) Tính giá trị của biểu thức A khi 9. x = 2) Chứng minh rằng 2 x B x − = . 3) Cho : P A B = tìm số tự nhiên x để biểu thức P đạt giá trị lớn nhất Bài II ( 2,5 điểm ) 1) Giải bài toán sau bằng cách lập phương trình hoặc hệ phương trình: Hai bạn Minh và An xuất phát cùng một lúc từ địa điểm A để đi đến địa điểm B bằng phương tiện xe đạp điện. Mỗi giờ bạn Minh đi nhanh hơn bạn An 2 km nên bạn Minh đến B sớm hơn bạn An 2,5 phút. Biết quãng đường AB dài 13 km, tính vận tốc xe của mỗi người. Hỏi Minh và An đi như vậy có đúng vận tốc quy định hay không nếu căn cứ theo quy định vận tốc tối đa của xe đạp điện là 25km/h. 2) Một ly rượu bằng thủy tinh phần đựng rượu dạng hình nón có đường kính miệng ly là 9 cm, chiều cao hình nón (như hình vẽ) là 6 cm. Hỏi ly đó có thể chứa đầy được bao nhiêu mililiter (ml) rượu? (lấy 3,14   và coi độ dày thành ly là không đáng kể). Bài III (2,0 điểm ) 1) Giải hệ phương trình sau 2 3 5 1 1 2 1 1 y x y x  + =  −    − = −  −  2) Trong mặt phẳng toạ độOxy , cho đường thẳng 2 ( ) : 2( 1) 3 d y m x m m = − − + parabol ( ) 2 : P y x = (mlà tham số ). a) Với 3 m = tím tọa giao điểm của ( ) d cắt ( ) P . b) Tìm mđể ( ) d cắt ( ) P tại 2 điểm phân biệt có hoành độ là số đo chiều dài và chiều rộng của hình chữ nhật có diện tích bằng 7 4 (đvdt). Bài IV ( 3,0 điểm ): Cho tam giác ABC vuông tại A. Đường tròn( ) ; O R đường kính AB cắt đoạn thẳng BC tại điểm thứ hai là D. Kẻ đường thẳng AH vuông góc với đường thẳng OC tại điểm H; đường thẳng AH cắt đoạn thẳng BC tại điểm M. 1) Chứng minh tứ giác ACDH là tứ giác nội tiếp. 2) Chứng minh 2 .) OH OC R = và ∆OHB đồng dạng với ∆OBC 3) Từ O kẻ đường thẳng vuông góc với BD tại K. Chứng minh HM là tia phân giác của góc DHB và . . . MB MD MK MC = Bài V ( 0,5 điểm ) Cho ab, là các số thực không âm thỏa mãn a b + =1. Tìm giá trị lớn nhất và giá trị nhỏ nhất của biểu thức ( 1) ( 1) P a b b a = + + + …………. Hết …………
  • 17. HƯỚNG DẪN CHẤM BÀI KIỂM TRA KHẢO SÁT TOÁN LỚP 9 NĂM HỌC 2022 – 2023 BÀI NỘI DUNG ĐIỂM Bài I (2,0 điểm) Cho hai biểu thức 2 x A x + = và 3 4 1 2 2 x x B x x x − + = − − − với 0; 4 x x   1) Tính giá trị của biểu thức A khi 9. x = 0,5 Thay x = 9 (TMĐK) vào biểu thức A 0,25 Ta tính được : 5 3 A = 0,25 2) Chứng minh rằng 2 x B x − = . 1,0 3 4 1 2 2 3 4 1 2 ( 2) x x B x x x x x x x x B − + = − − − − + − − − = 0,25 ( ) 3 4 ( 2) 2 2 ( 2) x x x B x x x B x x − + − = − − = − 0,25 2 x B x − = (điều phải chứng minh) 0,25 3) Cho : P A B = tìm số tự nhiên x để biểu thức P đạt giá trị lớn nhất 0,5 2 2 : : 2 . 2 x x P A B x x x x P x x + − = = + = − 2 2 x P x + = − với 0; 4 x x   4 1 2 P x = + − Trường hợp 1 : 0 4 0 x P     Trường hợp 2 : 4, 5 x x N x     5 2 5 2 4 4 2 5 2 x x x    −  −   − − 4 4 1 1 2 5 2 x  +  + − −
  • 18. 5 2 5 2 P +   − 9 4 5 P   + 0,25 Kết hợp 2 trường hợp  P đạt giá trị lớn nhất là 9 4 5 + khi x = 5 (thỏa mãn điều kiện 0,25 Bài II (2,0 điểm) 1) Giải bài toán sau bằng cách lập phương trình hoặc hệ phương trình: Hai bạn Minh và An xuất phát cùng một lúc từ địa điểm A để đi đến địa điểm B bằng phương tiện xe đạp điện. Mỗi giờ bạn Minh đi nhanh hơn bạn An 2 km nên bạn Minh đến B sớm hơn bạn An 2,5 phút. Biết quãng đường AB dài 13 km, tính vận tốc xe của mỗi người. Hỏi Minh và An đi như vậy có đúng vận tốc quy định hay không nếu căn cứ theo quy định vận tốc tối đa của xe đạp điện là 25km/h. 1,5 Gọi vận tốc xe của bạn An là x (km/h, x > 0) 0,25 Khi đó vận tốc xe của bạn Minh là x + 2 (km/h 0,25 Thời gian bạn An đi hết quãng đường AB là: 13 x (h) Thời gian bạn Minh đi hết quãng đường AB là: 13 2 x + (h) 0,25 Vì bạn Minh đến nơi sớm hơn An 2,5 phút nên ta có phương trình 2 13 13 1 2 624 0 2 24 x x x x − =  + − = + (1) 0,25 Giải phương trình (1) ta được x1 = 24 (TMĐK); x2 = −26 (Loại) 0,25 Vận tốc xe của An là 24 km/h, vận tốc xe của Minh là 26 km/h. Vậy bạn An đi đúng vận tốc quy định, còn bạn Minh đi không đúng vận tốc quy định. 0,25 2) Một ly rượu bằng thủy tinh phần đựng rượu dạng hình nón có đường kính miệng ly là 9 cm, chiều cao hình nón (như hình vẽ) là 6 cm. Hỏi ly đó có thể chứa đầy được bao nhiêu mililiter (ml) rượu? (lấy 3,14   và coi độ dày thành ly là không đáng kể). 0,5 Thể tích hình nón là 2 2 3 1 1 (9: 2) .6 40,5 127,17( ) 3 3 V r h cm    = = =  127,17 V ml   0,25 Vậy ly có thể chứa đầy khoảng 127,17 cm3 0,25 Bài III (2,0 điểm) 1) Giải hệ phương trình sau 2 3 5 1 1 2 1 1 y x y x  + =  −    − = −  −  1,0 Đk; : 1 x  2 2 3 5 3 5 2 3 5 1 1 1 1 2 2 1 4 2 7 7 1 1 y y y x x x y y y x x   + = + =    + = − −    −         − = − − = − =    − −   0,25 0,25
  • 19. 0( ) 1 1 1 2( ) 1 1 x tm y x x tm y y  =    =  − =        = =     =   0,25 Vậy hệ phương trình đã cho có 2 nghiệm ( ) ( ) ( )   ; 0;1 ; 2;1 x y  0,25 2) Trong mặt phẳng toạ độOxy , cho đường thẳng 2 ( ) : 2( 1) 3 d y m x m m = − − + parabol ( ) 2 : P y x = (mlà tham số ). a) Với 3 m = tím tọa giao điểm của ( ) d cắt ( ) P . 1,0 Hoành độ giao điểm của ( ) d và ( ) P là nghiệm của phương trình: 2 2 2 2 2( 1) 3 2( 1) 3 0 (*) x m x m m x m x m m = − − +  − − + − = Thay 3 m = vào (*) ta có: ( ) 2 4 0 – 4 0 0 x x x x x − =  =  = hoặc 4 x = 0,5 Với 1 1 0 0 x y =  = Với 2 2 2 0 4 16 x y =  = = 0,25 Vậy với m = 3 thì ( ) d cắt ( ) P tại hai điểm phân biệt ( ) 0;0 A và ( ) 4;16 B 0,25 b) Tìm mđể ( ) d cắt ( ) P tại 2 điểm phân biệt có hoành độ là số đo chiều dài và chiều rộng của hình chữ nhật có diện tích bằng 7 4 (đvdt). 0,5 Xét phương trình (*) 2 2 2( 1) 3 0 x m x m m − − + − = ( ) 2 2 2 2 2 1; 2( 1); 3 ' 1 1( 3 ) 2 1 3 1 a b m c m m m m m m m m m = = − = − = − − − = − + − + = + ( ) d cắt ( ) P tại 2 điểm phân biệt khi ∆’>0 1 0 1 m m  +    − 0,25 Theo Vi- et có 2 1 2 1 2 : 2 – 2 ; . – 3 x x m x x m m + = = Để 1 2 , x x là chiều dài và chiểu rộng của hình chữ nhật có diện tích bằng 7 4 thì cần có: 1 2 0; 0 x x   sao cho 1 2 2 1 7 . 4 4 12 7 0 (**) m x x m m   =   − − =  Giải PT(**) suy ra 1 1 2 m = − ( loại); 2 7 2 m = (Thỏa mãn) Vậy 7 2 m = là giá trị cần tìm. 0,25 Bài IV Cho tam giác ABC vuông tại A. Đường tròn( ) ; O R đường kính AB cắt đoạn thẳng BC tại điểm thứ hai là D. Kẻ đường thẳng AH vuông góc với đường thẳng OC tại điểm H; đường thẳng AH cắt đoạn thẳng BC tại điểm M. 1) Chứng minh tứ giác ACDH là tứ giác nội tiếp. 1,0 Vẽ hình đúng đến câu a 0,25
  • 20. (3,0 điểm) Chứng minh được 0 90 AHC ADC = = 0,25 Mà 2 đỉnh H và D là 2 đỉnh kề nhau 0,25 Suy ra tứ giác ACDH là tứ giác nội tiếp (đpcm) 0,25 2) Chứng minh 2 .) OH OC R = và ∆OHB đồng dạng với ∆OBC 1,0 Chứng minh 2 . OH OC OA = mà OA OB R = = nên 2 . OH OC R = 0,25 Suy ra 2 . OH OC OB = suy ra OH OB OB OC = 0,25 Xét ∆OHBvà ∆OBC có HOB chung ; OH OB OB OC = 0,25 Suy ra ∆OHB∽ ∆OBC (c.g.c) 0,25 3) Từ O kẻ đường thẳng vuông góc với BD tại K. Chứng minh HM là tia phân giác của góc DHB và . . . MB MD MK MC = 1,0 ∆OHB∽ ∆OBC (cmt) => OHB OBC = Chứng minh ; CHD CAD CAD OBC = = Suy ra OHB CHD = Từ đó suy ra DHM BHM = hay HM là tia phân giác của BHD 0,25 ∆ DHB có HM là đường phân giác trong MD HD MB HB  = Chứng minh HC là phân giác ngoài của ∆ DHB CD HD CB HB = = Vậy . . MD CD MD CB MB CD MB CB =  = 0,25
  • 21. Vì OK vuông góc với BD tại K nên K là trung điểm của BD Từ đó suy ra .( ) .( ) 2 . ( ) MD MB MC MB MC MD MD MB MC MB MD + = −  = − 0,25 2 . 2 . . . MD MB MK MC MB MD MK MC  =  = 0,25 Bài V (0,5 điểm) Cho ab, là các số thực không âm thỏa mãn a b + =1. Tìm giá trị lớn nhất và giá trị nhỏ nhất của biểu thức ( 1) ( 1) P a b b a = + + + 0,5 +) Tìm giá trị nhỏ nhất của biểu thức P Vì , 0 0 a b ab    . Ta có ( 1) ( 1) a b ab a a b a + = +  +  Tương tự có ( 1) b a b +  Suy ra 1 P a b a b P  +  +   Dấu “=” xảy ra khi , 0 1 1 0 0 a b a a b b ab   =   + =    =   =  hoặc 0 1 a b =   =  Vậy 1 MinP =  1 0 a b =    =  hoặc 0 1 a b =   =  0,25 + ) Tìm giá trị lớn nhất của biểu thức P Ta có 2 3 2 3 ( 1) 2 3 ( 1) P a b b a = + + + Áp dụng bất đẳng thức Cauchy với hai số không âm 3 , 1 a b+ ta có: 2 3 ( 1) 3 1 a b a b +  + + Tương tự 2 3 ( 1) 3 1 b a b a +  + + Suy ra 2 3 3 1 3 1 2 3 4 4 2 P a b b a P a b  + + + + +   + + 2 3 6 3 P P     Dấu “=” xảy ra khi , 0 1 1 3 1 2 3 1 a b a b a b a b b a    + =    = =  = +   = +  KL: 1 3 2 MaxP a b =  = = 0,25 Lưu ý: Các cách làm khác nếu đúng vẫn cho điểm tối đa
  • 22. TRƯỜNG THPT SƠN TÂY ĐỀ THI THỬ KỲ THI TUYỂN SINH LỚP 10 THPT ( LẦN 2) NĂM HỌC 2023 – 2024 Môn thi: TOÁN Thời gian làm bài: 120 phút Bài I ( 2,0 điểm Cho hai biểu thức 1 1 1 x A x x = + − − và 1 x B x x = + + với 0, 1. x x   a) Tính giá trị của biểu thức B khi 4. x = b) Rút gọn biểu thức M = AB. c) Khi x > 1 hãy so sánh giá trị biểu thức M với 1. Bài II ( 2,0 điểm ) 1) Giải bài toán sau bằng cách lập phương trình hoặc hệ phương trình: Tuần trước, Khuê mua 1 kg táo và 1 kg cam ở siêu thị hết 130 nghìn đồng. Hôm nay, Khuê quay lại siêu thị mua cùng lượng hoa quả như vậy nhưng phải trả 154 nghìn đồng. Người bán hàng giải thích giá cam đã tăng 15% và giá táo đã tăng 20% so với tuần trước. Hỏi giá tiền mỗi kg táo và cam ngày hôm nay là bao nhiều nghìn đồng. 2) Quả bóng rổ size 7 có đường kính 24,5 cm. Tính diện tích bề mặt quả bóng rổ đó ( lấy 3,14   ). Bài III (2,5 điểm ) 1) Giải hệ phương trình sau 1 3 1 2 3 x y x y  + =     − =   2) Trong mặt phẳng toạ độOxy , cho parabol ( ) 2 : P y x = và đường thẳng : 2 1 d y x m = + + (mlà tham số ). a) Chứng minh đường thẳng d luôn cắt ( ) P tại 2 điểm phân biệt. b) Tìm x để đường thẳng d cắt ( ) P tại 2 điểm phân biệt có hoành độ 1 2 , x x ( với 1 2 x x  )) thỏa mãn 1 2 2 1 8. x x x x + − = Bài IV ( 3,0 điểm ) Cho đường tròn tâm ( ) O và điểm M nằm ngoài đường tròn ( ) O . Qua điểm M kẻ các tiếp tuyến , MA MB tới đường tròn ( ) O với , A Blà hai tiếp điểm. Từ điểm A kẻ đường thẳng song song với MB cắt đường tròn ( ) O tại điểm C ( C khác A), đường thẳng MC cắt đường tròn ( ) O tại điểm D ( D khác C ). Gọi H là giao điểm của AB và . MO a) Chứng minh tứ giác MAOB nội tiếp. b) Chứng minh 2 . MA MD MC = và . BDM ADB = c) Gọi F là điểm đối xứng với D qua MO. Chứng minh , , C H F thẳng hàng. Bài V ( 0,5 điểm ) Cho hai số dương a,b thoả mãn điều kiện 2 2 a b +  . Tìm giá trị nhỏ nhất của biểu thức 1 5 2 2 P ab ab a b = + + + …………. Hết ………… Họ và tên thí sinh:…………………………………Số báo danh……………………………. Họ và tên của cán bộ coi thi:…………………….Chữ ký của cán bộ coi thi…………….
  • 23. TRƯỜNG THPT SƠN TÂY HƯỚNG DẪN CHẤM MÔN TOÁN ĐỀ THI THỬ KỲ THI TUYỂN SINH LỚP 10 THPT ( LẦN 2) NĂM HỌC 2023 – 2024 ( Hướng dẫn chấm gồ 04 trang ) I. HƯỚNG DẪN CHUNG • Hướng dẫn chấm chỉ trình bày sơ lược các bước giải, lời giải của học sinh cần lập luận chặt chẽ, hợp logic. Nếu học sinh trình bày cách làm khác mà đúng thì vẫn được điểm theo thang điểm tương ứng. • Điểm toàn bài không làm tròn. II. ĐÁP ÁN VÀ THANG ĐIỂM BÀI SƠ LƯỢC LỜI GIẢI ĐIỂM Bài I (2,0 điểm) a) Tính giá trị của biểu thức Bkhi 4 x = 0,5 Với 4 x = thoả mãn điều kiện, ta thay 4 x = vào biểu thức B. 0,25 Tính được 4 4 7 4 4 1 B = = + + 0,25 b) Rút gọn biểu thức M AB = 0,5 Với điều kiện 0, 1 x x   ta có: ( )( ) 1 1 . . . 1 1 1 1 1 1 1 x x x x M A B x x x x x x x x x       = = + = +     − − + + − + + + −     0,25 ( )( ) ( )( ) 1 . 1 1 1 1 1 x x x x x x x x x   +   = +   + + + − + −   0,25 1 . 1 1 x x x x x x + + = − + + 0,25 Vậy 1 x M x = − với 0, 1 x x   0,25 c) Khi 1 x  hãy so sánh giá trị của biểu thức M với 1 0,5 Ta có ( ) 1 1 1 1 1 1 1 x x M x x x − + = = = + − − − 0,25 Khi 1 x  ta có 1 1 1 0 0 1 1 1 1 x x x −     +  − − Vậy khi 1 x  ta có 1. M  0,25 Bài II (2,0 điểm) 1) Giải bài toán lập phương trình hoặc hệ phương trình 1,5 Gọi x và y theo thứ tự là giá của 1 kg táo, 1 kg cam tuần trước ( tính theo nghìn đồng), điều kiện 0 , 130. x y   0,25 Tuần trước, Khuê mua 1 kg táo và 1 kg cam hết 130 nghìn đồng nên ta có x+y=130. (1) 0,25 Hôm nay, giá 1 kg cam đã tăng 15% nên số tiền cam phải trả là 115 15% 100 y y y + = giá 1 kg táo đã tăng 20% nên số tiền táo phải trả là 120 20% 100 x x x + = 0,25
  • 24. Khuê phải trả 154 nghìn đồng nên ta có phương trình ( ) 120 115 154. 2 100 100 x y + = Từ (1) và (2) ta có hệ phương trình 120 115 154 1 1 130 00 00 x y x y + =     =  + 0,25 120 115 15400 120 115 15400 120 115 15400 120 115 130 120 120 15600 5 200 40 90 40 0 154 0 x x y x y x y x y x y y y y x y + = + = =             = =       =   + = + = + = + = 0,25 Vậy tuần này, giá 1 kg táo là 120 108 100 x = ( nghìn đồng), giá 1 kg cam là 115 46 100 y = (nghìn đồng) 0,25 2) Tính diện tích bề mặt bóng rổ 0,5 Bán kính quả bóng rổ 2 24,5 12,25( ) 2 R cm = = . Diện tích bề mặt bóng rổ 2 4 S R  = 0,25 ( ) 2 2 4.3,14. 12,25 1884,785( ) S S cm = =  . Vậy giá trị gần đúng của diện tích bề mặt quả bóng rổ là 2 1884,785( ) cm 0,25 Bài III (2,5 điểm) 1. Giải hệ PT 1 3 1 2 3 x y x y  + =     − =   1,0 Điều kiện xác định: 0 y  , Đặt 1 u y = ta có hệ phương trình 3 2 3 x u x u + =   − =  0,25 3 6 2 3 3 3 1 x x x x u u x u = = =          + = = − =    0,25 1 1 1 y y =  = ( TMĐK) 0,25 Vậy hệ phương trình có nghiệm là ( ) ( ) ; 2;1 x y = 0,25 2. Giải bài toán về parabol ( ) 2 : P y x = và đường thẳng : 2 1 d y x m = + + 1,5 a) Chứng minh d luôn cắt ( ) P 1,0 Phương trình hoành độ giao điểm của đường thẳng d và ( ) P là: 2 1 x x m = + + 0,25 2 2 1 0 x x m  − − − = (1) 0,25 Biệt thức ∆’ 2 m = + 0,25 Vì 0 2 0 m m m R m R    = +    nên PT (1) luôn có hai nghiệm phân biệt Vậy đường thẳng đường thẳng d luôn cắt ( ) P tại 2 điểm phân biệt. 0,25 b) Tìm x để 1 2 2 1 8. x x x x + − = 0,5 Theo giả thiết ta suy ra 1 2 , x x là hai nghiệm phân biệt của PT (1) Theo định lý Viet ta có 1 2 1 2 2 . 1 x x x x m + =    = − −   Do 1 2 1 0 . ,0 m x x − −  = mà 1 2 1 2 0; 0 x x x x  =   0,25
  • 25. 1 2 2 1 1 2 2 1 8. 8 x x x x x x x x + − =  − + + = 5 5 5 m m m =   =   = −  Vậy tập hợp các giá trị cần tìm của m là {-5; 5} 0,25 Bài IV (3,0 điểm) a) Chứng minh tứ giác MAOB nội tiếp. 1,0 Vẽ hình đúng đến ý a) 0,25 Do MAlà tiếp tuyến với đường tròn ( ) O tại A suy ra 0 90 OAM = Do MBlà tiếp tuyến với đường tròn ( ) O tại B suy ra 0 90 OBM = 0,25 Suy ra 0 180 OAM OBM + = Mà OAM và OBM là hai góc đối nên tứ giác MAOBnội tiếp 0,25 0,25 b) Chứng minh 2 . MA MD MC = và . BDM ADB = 1,5 Chứng minh 2 . MA MD MC = 0,75 Xét ∆ MAC và ∆ MDA có AMC chung 0,25 ACM DAM = ( góc nội tiếp và góc tạo bởi tiếp tuyến với dây cung cùng chắn cung AD ) 0,25 ∆ MAC∽∆ MDA( g-g) 2 . MA MC MA MD MC MD MA  =  = (đpcm) 0,25 Chứng minh . BDM ADB = 0,75 Xét ∆ BDM và ∆ ADB ta có: +) DBM DAB = ( góc nội tiếp và góc tạo bởi tiếp tuyến với dây cung cùng chắn cung BD ) +) ACD DBA = ( góc nội tiếp và góc tạo bởi tiếp tuyến với dây cung cùng chắn cung AD ) Mà ACM DMB = ( hai góc so le trong do AC//MB) nên DBA DMB = 0,25 0,25 ∆ BDM ∽∆ ADB( g – g) BDM ADB  = 0,25 c) Gọi F là điểm đối xứng với D qua MO. Chứng minh , , C H F thẳng hàng 0,5 Chứng minh tứ giác DHOC nội tiếp Ta có ∆ AMOvuông tại A và AH MO ⊥ 2 . MA MH MO = = Theo kết quả phần (b) 2 . MA MD MC = Suy ra . . MD MC MH MO = Xét ∆ MCO và ∆ MHD có CMO chung và MC MO MH MD = nên ∆ MCO và ∆ MHD đồng dạng (cg-c) MCO MHD DHOC  =  là tứ giác nội tiếp. 0,25 D H O M A B C D H O M A B C F
  • 26. Ta có CHO CDO = ( góc nội tiếp cùng chắn cung OC ) Lại có CDO MCO = ( tam giác OCDcân tại O ) Mà MCO MHD = ( do tứ giác DCOH nội tiếp) MHD MHF = ( do , D F đối xứng nhau qua MO) Vậy CHO MHF = Ta lại có , , M H O thẳng hàng nên 0 0 180 180 CHO MHC MHF MHC + =  + = Vậy ba điểm , , F H C thẳng hàng. 0,25 Bài V (0,5 điểm) Ta có 2 2 2 2 1 1 1 4 1 ( ) ( ) 2 2 2 10 P ab ab ab ab ab a b ab a b = + + = + + + + + + Ta có BĐT ( ) ( ) ( ) 2 2 2 0 4 4 4 x y x y xy xy x y xy −   − +   +  Với hai số dương , x yta có ( ) 2 4 1 1 4 4 x y x y xy xy x y x y x y + +     +  + + Áp dụng BĐT 1 1 4 x y x y +  + , BĐT Côsi 2 x y xy +  cho hai số dương , x y và giả thiết ta có: ( ) 2 2 2 2 2 1 1 4 1 4 1 2 2 1 4 ; 2 . 4; 2 2 2 4 ( ) (2 2) ab ab ab ab ab ab a b a b a b +   +  =   = + + + Suy ra 19 4 P  0,25 Dấu “=” xảy ra khi 4 2 2 2 a b ab a b ab a b  =   =  = =    + =  Vậy giá tri nhỏ nhất của biểu thức 19 4 P = khi 2 a b  = = 0,25 …Hết…
  • 27. PHÒNG GIÁO DUC VÀ ĐÀO TẠO QUẬN HÀ ĐÔNG ĐỀ CHÍNH THỨC ĐỀ KHẢO SÁT CHẤT LƯỢNG LỚP 9 (LẦN 1) Năm học 2023 - 2024 Môn: TOÁN Thời gian làm bài: 120 phút (Không kể thời gian giao đề) (Đề gồm có 01 trang) Câu I. (2,0 điểm) Cho hai biểu thức 3 1 5 2 ; 4 2 2 x x x P Q x x x + − − = = − − − + với 0, 4 x x   . 1. Tính giá trị của biểu thức P khi 25 x = 2. Chứng minh 2 x Q x = − 3. Tìm giá trị của x để biểu thức P Q đạt giá trị nhỏ nhất. Câu II. (2,0 điểm) 1. Giải bài toàn bằng cách lập phương trình hoặc hệ phương trình Hai bạn Linh và Chi ở hai địa điểm cách nhau 18 km đạp xe đi ngược chiều nhau để gặp nhau. Nếu hai bạn khởi hành cùng một lúc thì sẽ gặp nhau sau 40 phút. Nhưng nếu Linh khởi hành trước 18 phút thì các bạn sẽ gặp nhau sau 30 phút tính từ lúc Chi bắt đầu đi. Tính vận tốc của mỗi bạn? 2. Một chiếc cốc có dạng hình trụ với chiều cao 8 cm, bán kính đáy là 3 cm. Hỏi chiếc cốc này có đựng được 200ml sữa không? (Lấy 3,14   và bỏ qua bề dày của chiếc cốc). Câu III. (2,5 điểm) 1. Giải hệ phương trình: 2 2 1 8 2 3 1 3 x y x y  − + − =   − − − = −   2. Cho Parabol ( 2 P): 2 x y = và đường thằng (d): 1 y mx m = − + (với m là tham số). a) Tìm m để ( d) cắt ( ) P tại 2 điểm phân biệt A và B nằm về hai phía của trục tung. b) Gọi K là giao điểm của (d) và trục Oy. Tìm m để Δ 2 KOA KOB S S = Câu IV. (3,0 điểm) Từ điểm A ở ngoài đường tròn ( ) ; O R vẽ hai tiếp tuyến , ( , AB AC B C là các tiếp điểm) và cát tuyến ADE thuộc nửa mặt phẳng bờ là đường thẳng OA không chứa điểm B của đường tròn ( ) O . Gọi H là giao điểm của OA và BC . 1. Chứng minh bốn điểm A,B,O,C cùng thuộc một đường tròn. 2. Chứng minh AO BC ⊥ tại H và AH.AO AD.AE = 3. Đường thẳng đi qua điểm D và song song với đường thẳng BE cắt , AB BC lần lượ tại I, K. Chứng minh tứ giác OHDE nội tiếp và D là trung điểm của IK . Câu V. (0,5 điểm) Với các giá trị của m để phương trình ( ) 2 2 2 1 9 0 x m x m − + + + = có nghiệm, hãy tìm giá trị nhỏ nhất của biểu thức ( )( ) 2 2 2 4 m m m B m + − + = . -----------------------Hết -----------------------------
  • 28. PHÒNG GIÁO DỤC VÀ ĐÀO TẠO QUẬN HÀ ĐÔNG HƯỚNG DẪN CHẤM MÔN TOÁN KHẢO SÁT CHẤT LƯỢNG LỚP 9 (LẦN 1) Năm học 2023 - 2024 Câu Nội dung Điểm Câu I (2,0 điềm) 1) Tính giá trị của P khi 25 x = với 0, 4 x x   . 0,5 Ta có ( ) x 25 TMCK = thay vào biểu thức P được 25 3 25 2 P + = − 0,25 Tính được 28 28 5 2 3 P = = − 0,25 2) Chứng minh 2 x Q x = − với 0, 4 x x   . 1,0 1 5 2 1 5 2 4 4 2 2 x x x x x x Q x x − − − − − = + + = − − + ( )( ) ( )( ) 1 2 5 2 2 2 x x x x x − − + − = + − 0,25 ( )( ) 3 2 5 2 2 2 x x x x x − + + − = + − 0,25 ( )( ) 2 2 2 x x x x + = + − 0,25 ( ) ( )( ) 2 2 2 2 x x x x x x + = = − + − 0,25 3) Tìm giá trị của x để biểu thức P Q đạt giá trị nhỏ nhất 0,5
  • 29. * Với 0, 4 x x   biến đổi 3 3 2 3 3 : 2 2 2 P x x x x x x Q x x x x x x + + − + = =  = = + − − − *Vì 3 0, 4 0, 0 x x x x      Áp dụng BĐT Cosi cho 2 số 3 ; x x ta có: 3 3 2 2 3 P x x Q x x = +   = Dấu "=" xảy ra khi ( ) 3 3 7 x x M x =  = Vậy Min 2 3 3 P x Q =  = 0,25 Câu II (2,0 điểm ) 1) Gọi vận tốc của Linh là ( ) km / h x , vận tốc của Chi là ( ) km / h y . Điều kiện: 0; 0 x y   Đổi: 40 phút 2 3 = giò; 18 phút 3 10 = giờ; 30 phút 1 2 = giờ. 0,25 Hai bạn khởi hành cùng một lúc thì sẽ gặp nhau sau 40 phút 2 3 = giờ + Quãng đường Linh đi được là: ( ) 2 km 3 x  + Quãng đường Chi đi được là: ( ) 2 km 3 y  Linh và Chi ở hai địa điểm cách nhau 18 km đạp xe đi ngược chiều nhau nên, ta có phương trình: 2 2 18 27 3 3 x y x y  +  =  + = 0,25 + Thời gian của Chi tính từ lúc đi đến lúc gặp nhau là 1 2 giờ, nên quãng đường Chi đi được là: ( ) 1 km 2 y + Thời gian của Linh tính từ lúc đi đến lúc gặp nhau là 3 10 giờ 1 2 + giờ 4 5 = giờ, nên quãng đường Linh đi được là: ( ) 4 km 5 x 0,25
  • 30. Ta có phương trình: 4 1 18 8 5 180 5 2 x y x y  +  =  + = Từ (1) và (2) ta có hệ phương trình: ( ) ( ) 15 tm 27 5 5 135 12 tm 8 5 180 8 5 180 x x y x y y x y x y    = + = + =         = + = + =       0,5 Vậy vận tốc của Linh là ( ) 15 km / h , vận tốc của Chi là ( ) 12 km / h . 0,25 2) Thể tích của chiếc cốc là: ( ) 2 3 3 .8 226,08 cm    0,25 Vì 3 226,08 cm 226,08ml 200ml =  . Nên chiếc cốc này có thể đựng được 200ml sữa. 0,25 Câu III (2,5 diêm ) 1) ĐKXĐ: 2, 1 x y   Đặt ( ) 2 ; 1 0, 0 x a y b a b − = − =   Ta có hệ phương trình 2 8 2 8 2 8 2 8 3 3 3 2 6 6 7 14 2 2 a b a b a b a b a a b a b b b b     + = + = + = + = =              − = − − = − = = =         Theo cách đặt có 2 3 2 9 11 1 4 5 1 2 x x x y y y  − =  − = =        − = = − =     (tmđk) Kết luận 0,25 0,5 0,25 2) a) Tìm m để ( ) d cắt ( ) P tại 2 diểm phân biệt A và B nằm về hai phía so với trục tung. Xét phương trình hoành độ giao điểm ( ) d và ( ) P : ( ) 2 2 1 2 2 2 0 * 2 x mx m x mx m = − +  − + − = (d) cắt ( ) P tại hai điểm phân biệt A và B nằm ở hai phía của trục tung.  Phương trình ( ) * có hai nghiệm 1 2 ; x x trái dấu 0 ac   2 2 0 1 m m  −    0,25 0,25 b) Gọi K là giao điểm của (d) và trục Oy. Tìm m để Δ 2 KOA KOB S S =
  • 31. Với ( ) * m 1,pt  có hai nghiệm 1 2 x ,x . Giả sử 1 2 x 0 x   Theo định lý Viet, ta có 1 2 1 2 2 (1) 2 2 (2) x x m x x m + =   = −  Gọi H và I lần lượt là hình chiếu của A và B trên Oy. 1 2 AH ;BI x x  = = Hai tam giác trên chung đáy OK nên tỉ số diện tích bằng tỉ số hai đường cao Theo đề bài, ta có: 1 1 2 2 2 2 2 KOA KOB x S x x S x =  =  = Mà 1 2 1 2 0 2 x x x x    − = Từ (1) và (3) 1 2 4 ; 2 x m x m  = = − Thay vào (2), ta có: ( ) 2 4 2 2 2 4 1 0 m m m m m − = −  + − = Giải ra ( ) ( ) 1 2 1 17 8 1 17 8 m tmdk m tmdk  − + =    − −  =   Vậy 1 17 1 17 ; 8 8 m   − + − −         0,5 0,25 0,25 0,25
  • 32. 1) Xét ( ) O có AB,AC là hai tiếp tuyến cắt nhau ( B,C là các tiếp điểm) 90 ABO ACO  = = B,C  thuộc đường tròn đường kính OA  Bốn điểm A,B,O,C cùng thuộc đường tròn đường kính OA. 0,75 2) Xét ( ) O có AB,AC là hai tiếp tuyến cắt nhau tại A,B và C là các tiếp điểm AB AC A  =  thuộc trung trực của đoạn BC Có ( ) OB OC R O = =  thuộc trung trụ ̂ c của đoạn BC Do đó OA là trung trực của đoạn BC OA BC  ⊥ tại H . +) Xét ABO có 90 , ABO BH OA = ⊥ 2 AH AO AB   = (hệ thức lượng) (1) +) Xét ( ) O có ABD AEB = (góc nội tiếp và góc tạo bời tia tiếp tuyến và dây cùng chắn BD ) C/ m  dược: ABL  ( ) 2 g g AD.AE AB AEB  −  = ∽ (2) Từ (1) và ( ) 2 AH AO AD AE   =  0,25 0,25 0,25 0,25 3) Có . AH AE AH AO AD AE AD AO =   = C/ m  dược: ADH  ( ) c.g.c AOE ADH AOE   = ∽ C/ m  dược tứ giác DHOE nội tiếp +) Có tứ giác DHOE nội tiếp (cmt) AHD OED  = Xét ( ) O có ( ) OD OE R ODE = =  cân tại O OED ODE  = Lại có tứ giác DHOE nội tiếp ( ) cmt ODE OHE  = AHD OHE  = Gọi M là giao điểm của BC và AE C/ m được HM là phân giác của DHE Lại có HA HM ⊥ (vì OA BC ⊥ ) HA  là phân giác của góc ngoài tại H của HDE  MD AD DH ME AE EH    = =     Có ( ) IK//BE gt DK//BE DK DM BE ME   = Có IK ( ) //BE gt DI//BE DI AD BE AE   = DK DI DK DI BE BE  =  = hay D là trung điểm của IK. 0,25 0,25 0,25 0,25
  • 33. Câu V. (0,5 điểm) PT có nghiệm ( ) ( ) 2 2 Δ 0 [ 2 1 ] 4 9 0 4 m m m    − + − +    Biến đổi ( ) 3 2 2 2 8 8 8 15 8 16 16 2 2 8 15 ( 4) 16 2 2 m m B m m m m m m m m m m m +   = = + = − + + + + −       = − + + + −     0,25 Với 4 m  chứng minh được 2 ( 4) 0 m −  , 8 15 4 và 16 14 2 2 m m m   +  −      Dấu "=" xãy ra khi 4 m = Vậy GTNN của 18 B = khi 4 m = 0,25 (HS giải theo cách khác nếu đủng vẫn cho điểm tối đa)
  • 34. 1 SỞ GIÁO DỤC VÀ ĐÀO TẠO HẢI DƯƠNG ĐỀ THI GIAO LƯU AN THANH- QUANG KHẢI ĐỀ THI THỬ VÀO LỚP 10 THPT NĂM HỌC 2024 - 2025 Môn thi : TOÁN Thời gian làm bài: 120 phút, không kể thời gian giao đề (Đề thi gồm 02 trang- 05 câu) Câu 1 (2,0 điểm) 1. Giải phương trình sau: 2 x - 5 - 3= 0 2. Giải hệ phương trình sau: 2 7 . 2 1 x y x y = +   + =  Câu II. (2,0 điểm) 1. Rút gọn biểu thức sau: 2 a+b a+b b a b a M= : - + - a-b a+ b b- ab ab+a −       (Với ). 2) Xác định hàm số y = ax + b. Biết rằng đồ thị hàm số song song với đường thẳng y = 1 2 x – 3 và cắt trục hoành tại điểm có hoành độ là -2 Câu 3 (2 điểm) 1)Ông Ba đầu tư 600 triệu đồng để kinh doanh: đầu tư trồng rau sạch với lãi suất 10% mỗi tháng và đầu tư vào một nhà hàng với lãi suất 12% mỗi tháng . Cuối tháng ông Ba nhận được 64 triệu đồng tiền lãi. Hỏi khoản tiền mà ông Ba đã đầu tư vào mỗi khoản là bao nhiêu triệu đồng? 2) Cho phương trình: 2 2 2 1 4 0 x m x m m Tìm m để phương trình có hai nghiệm phân biệt 1 2 ; x x thỏa mãn: 1 2 3 2 16 x x m Câu 4 (3,0 điểm) 0   a b
  • 35. 2 1)Trên mặt biển, khi khoảng cách AB từ ca nô đến chân tháp hải đăng là 250 m, một người đứng trên tháp hải đăng đó nhìn về phía ca nô theo phương CA tạo với phương nằm ngang Cx một góc là 0 32 ACx ( Hình vẽ). Tính chiều cao của tháp hải đăng( làm tròn kết quả đên chữ số thập phân thứ nhất, biết AB//Cx và độ cao từ tầm mắt của người đó đến đỉnh tháp hải đăng là 3,2 m. 2)Từ một điểm M nằm ngoài đường tròn (O ; R), kẻ tiếp tuyến MA, MB với đường tròn (O) (A và B là tiếp điểm). a) Chứng minh tứ giác MAOB nội tiếp. Xác định tâm đường tròn ngoại tiếp tứ giác MAOB b.1) Gọi E là trung điểm MA, AB cắt OM tại H và K là giao điểm của BE với đường tròn (O). Chứng minh: 2 . 2 BK BE AH b.2) Kẻ đường kính AC của đường tròn (O) , MC cắt đường tròn (O) tại D, kẻ DI vuông góc với AC tại I. Gọi F là giao điểm của AB và DI. Chứng minh điểm C, E, F thẳng hàng. Câu 5(1 điểm). Cho các số thực dương x, y, z thỏa mãn: 2 2 2 3 x y z xyz + + = Tìm giá trị lớn nhất của biểu thức 2 2 2 4 4 4 x y z P x yz y xz z xy = + + + + + ----------- Hết ---------- Họ và tên thí sinh:......................................................... Số báo danh:........................ Chữ ký giám thị 1: ..................................... Chữ ký giám thị 2: .................................
  • 36. 3 SỞ GIÁO DỤC VÀ ĐÀO TẠO HẢI DƯƠNG ĐỀ GIAO LƯU AN THANH- QUANG KHẢI HƯỚNG DẪN CHẤM THI THỬ VÀO LỚP 10 THPT NĂM HỌC 2024 - 2025 Môn thi : TOÁN Thời gian làm bài: 120 phút, không kể thời gian giao đề (Đáp án gồm 08 trang- 05 câu) Câu Hướng dẫn chấm Biểu điểm 1 1)2 x - 5 - 3= 0 . Điều kiện 5 x  0,25 2 x - 5 = 3  2 2 2 5 3 x 4 5 9 x 0,25 29 4 x (t/m) 0,25 Vậy phương trình có nghiệm 29 4 x 0,25 2 2) 2 7 2 1 x y x y = +    + =  2x y 7 4x 2y 14 x 2y 1 x 2y 1 − = − =      + = + =   0,25 5x 15 x 2y 1 =    + =  0,25 x 3 y 1 =    = −  0,25 Vậy hệ phương trình trên có nghiệm duy nhất (x,y)=(3; -1) 0,25 2 1) 2 a+b a+b b a b a M= : - + - a-b a+ b b- ab ab+a −       Với   + + − = + +     − + − +   a b a b b a a b : _ a b a b a b a b 2 0,25 + + + + − + − = + − + a b a b b ab ab a a b : a b a b 2 0,25 0   a b
  • 37. 4 + + − = + − + − − = + = − a b (a b) a b : b a a b a b a b a b 2 2 2 2 0,25 Vậy b a M − = với 0   a b . 0,25 2)Xét hàm số y = ax + b (*) Đồ thị (*) cắt trục hoành khi và chỉ khi: 0 a 0.25 Do đồ thị (*) song song với đường thẳng y = 1 2 x – 3 Khi và chỉ khi ( ) 1 a TM a 0 2 b 3  =      −  Hàm số (*) trở thành: 1 3 2 y x b b (*) 0.25 Vì đồ thị hàm số (*) cắt trục hoành tại điểm có hoành độ là (-2) nên đồ thị hàm số (*) đi qua điểm A(-2;0). Do đó ta có: 1 . 2 0 1 2 b b ( Thỏa mãn ĐK 3 b ) 0.25 Vậy hàm số cần xác định là: 1 1 2 y x 0.25 3 1)Gọi số tiền ông Ba đầu tư trồng rau sạch là x (Triệu đồng) Số tiền ông Ba đầu tư vào nhà hàng là y ( Triệu đồng) ĐK: 0<x,y<600 (*) + Vì tổng số tiền ông Ba đầu tư là 600 triệu đồng nên ta có phương trình: 600 1 x y 0.25 Sau một tháng : Số tiền lãi ông Ba được hưởng khi đầu tư vào trồng rau sạch là : 10%.x ( Triệu đồng) Số tiền lãi ông Ba được hưởng khi đầu tư vào nhà hàng là : 12%.y ( Triệu đồng) 0.25
  • 38. 5 Vì cuối tháng ông Ba nhận được 64 triệu đồng tiền lãi nên ta có phương trình: 10%. 12%. 64 x y 5 6 3200 x y (2) Từ (1) và (2) ta có hệ phương trình : 150 400 .... 5 6 3200 200 x y x x y y ( Thỏa mãn ĐK(*)) 0.25 Vậy số tiền ông Ba đầu tư vào vào trồng rau sạch là : 400 (Triệu đồng) Số tiền lãi ông Ba được hưởng khi đầu tư vào nhà hàng là : 200 (Triệu đồng) 0.25 2) Xét phương trình: 2 2 2 1 4 0 x m x m m (*) Ta có: 2 ' 2 1 1. 4 3 3 m m m m Phương trình (*) có hai nghiệm phân biệt khi và chỉ khi: ' 0 3 3 0 1 m m 0.25 Với m>1, áp dụng hệ thức Vi-ét ta có : 1 2 2 1 2 2 2 1 4 2 x x m x x m m 0.25 Ta có : 2 2 1 15 4 0 2 4 m m m Với mọi giá trị của m 1 2 0 x x Với mọi giá trị của m Phương trình (*) có hai nghiệm cùng dấu Với 1 2 2 0 m m 1 2 0 x x Phương trình (*) có hai nghiệm dương phân biệt khi m>1 Vì 1 2 0; 0 x x nên 1 1 2 2 ; x x x x Theo bài ra : 1 2 3 2 16 x x m 1 2 3 2 16 x x m (3) 0.25 Từ (1) và (3) ta có hệ phương trình :
  • 39. 6 1 2 1 1 2 2 2 2 2 ... 3 2 16 4 x x m x m x x m x m Thay 1 2 2 4 x m x m vào (2) ta được : 2 2 4 4 m m m m …………. 4 m ( Thỏa mãn ĐK : m>1) Vậy m=4 là giá trị cần tìm 0.25 4 1)Hình vẽ : O.25 3,2 m x 250 m 320 B A C
  • 40. 7 (HS cần vẽ hình minh họa dưới) Vì Cx//AB nên : BAC ACx mà 0 32 ACx nên : 0 32 BAC Vì tam giác ABC vuông tại B nên áp dụng hệ thức giữa cạnh và góc trong tam giác vuông ta có : .tan BC AB BAC 0 250.tan32 BC =156,217338 156,2 0.5 Chiều cao của tháp hải đăng là : 156,2+3,2=159,4 (m) Vậy chiều cao tháp hải đăng xấp xỉ khoảng 159,4 m 0.25 2) Hình vẽ câu a 0.25 a) Xét tứ giác MAOB có : 0 90 MAO ( Do MA là tiếp tuyến của đường tròn (O)) 0 90 MBO ( Do MB là tiếp tuyến của đường tròn (O)) 0 0 0 90 90 180 MAO MBO Tứ giác MAOB nội tiếp đường tròn đường kính OM ( Tổng hai góc đối diện bằng 1800 ) Vậy tứ giác MAOB nội tiếp đường tròn đường kính OM. Tâm đường tròn ngoại tiếp tứ giác MAOB là trung điểm của đoạn thẳng OM 0.25 0.25 0.25 B A O M
  • 41. 8 b.1) Hình vẽ câu b.1 + Chứng minh được OM là đường trung trực của AB H là trung điểm của AB Chứng minh được EH là đường trung bình của tam giác ABM HE//BM 2 1 E B ( Cặp góc so le trong) Mà : 2 1 A B ( Góc nội tiếp và góc tạo bởi tia tiếp tuyến và dây cung cùng chắn BK của đường tròn (O)) 2 2 E A Tứ giác AEKH nội tiếp (hai đỉnh liên tiếp E và A cùng nhìn cạnh KH dưới một góc bằng nhau) 1 K HAE Hay 1 K BAE 0.25 + Chứng minh được : BAE đồng dạng với BKH BA BE BK BH . . BA BH BK BE Mà H là trung điểm của AB (cm trên) 1 2 2 1 H K E B A O M
  • 42. 9 1 2 BH AH AB Do đó : 2 . 2. . 2 BK BE AH AH AH 0.25 b.2) Hình vẽ Kẻ tiếp tuyến tại C của đường tròn (O) cắt DE tại N, Gọi giao điểm của NO với CD là Q. Đường thẳng CN cắt đường thẳng AD tại P + Chứng minh được DE là tiếp tuyến của đường tròn (O) Chỉ ra được ON là đường trung trực của CD Chứng minh được ba điểm H, B, N thẳng hàng. Chứng minh được N là trung điểm của CP ( Dựa vào định lí đường trung bình trong tam giác ACP) 0.25 + Chứng minh được F là trung điểm của ID Gọi giao điểm của CF với MA là E’ . Chứng minh được E’ là trung điểm của MA Vậy điểm E trùng với điểm E’ , nghĩa là ba điểm C, F, E thẳng hàng. 0.25 5 2 2 2 3 3 x y z x y z xyz yz xz xy + + =  + + = P Q N F I D C H E B A O M
  • 43. 10 Áp dụng bất đẳng thức Cô-si cho hai số dương ; x y yz xz ta có: 2 2 . x y x y yz xz yz x z +  = 0.25 Tương tự ta cũng có: 2 2 ; y z z x xz xy x xy yz y +  +  2 2 2 x y y z z x yz xz xz xy xy yz z x y        + + + + +  + +             1 1 1 1 1 1 3 x y z yz zx xy x y z x y z  + +  + +  + +  0.25 Lại có: 2 4 4 2 4 1 1 1 1 1 1 1 2 2 .2. . ( ) 4 4 2 x x yz x yz x yz x yz y z yz y z +  =   =  + + Tương tự 2 2 4 4 1 1 1 1 1 1 ( ); ( ) 4 4 y z y xz x z z xy x y  +  + + + Suy ra 0.25 2 2 2 4 4 4 1 2 2 2 1 1 1 1 3 ( ) ( ) 4 2 2 3 2 x y z P x yz y xz z xy x y z x y z P = + +  + + = + +  + + + =  Vậy giá trị lớn nhất của P = 3 2 khi x = y = z = 1 0.25 Lưu ý: Học sinh làm theo cách khác đúng vẫn cho điểm tối đa.
  • 44. SỞ GIÁO DỤC VÀ ĐÀO TẠO TỈNH PHÚ YÊN ĐỀ CHÍNH THỨC KỲ THI CHỌN HỌC SINH GIỎI THCS CẤP TỈNH, NĂM HỌC 2023 - 2024 Môn thi: TOÁN Ngày thi: 06/3/2024 Thời gian: 150 phút (không kể thời gian giao đề) Câu 1.(3,50 điểm) Cho các biểu thức: 1 1 1 1 ; 1 2 2 3 3 4 8 9 A = + + ++ + + + + 1 1 1 1 ; 1 2 3 8 B = + + ++ 1 2 3 8. C = + + ++ a) Tính giá trị của biểu thức A. b) So sánh giá trị các biểu thức A, B và C . Câu 2. (3,50 điểm) Giài hệ phương trình: ( ) ( ) 2 2 2 2 4 5 4 3 x x y y y y x x  + =   − =   Câu 3. ( 4,00 điểm) Cho hình vuông , ABCD I là trung điểm của cạnh AB. Dựng đường tròn tâm I , đường kính AB. Tiếp tuyến DE với đường tròn (I cắt cạnh BC tại F ( E là tiếp điểm). a) Biết 6,25 cm EF = , tính cạnh của hình vuông. b) Trên nửa đường tròn đường kỉnh AB (phần không cùng phía với hinh vuông ABCD ) lấy các điềm , M N sao cho 15 cm BM MN = = ( M nằm giữa B và N ). Tính chu vi tứ giác BMNA. Câu 4. (3,00 điểm) a) Đặt ; ; a b b c c a x y z a b b c c a − − − = = = + + + , với , , a b c là các số tự nhiên dương. Chứng minh rằng: ( )( )( ) ( )( )( ) 1 1 1 1 1 1 x y z x y z − − − = + + + . b) Biết ( )( )( ) ( )( )( ) 12 a b b c c a a b b c c a + + + = − − − ; tinh giá trị biểu thức a b c S a b b c c a = + + + + + . Câu 5. (3,00 điểm) Cho , x y là hai số thực dương thỏa: 2 2 1 5 4 4 y x = − . Tìm giá trị nhò nhất của biểu thức 4 2024 P x y = + + . Câu 6. (3,00 điểm) Cho tam giác ABC vuông tại A. D là điểm di động trên cạnh AC . Đường thẳng qua A và vuông góc với BD cắt đường thằng qua C và vuông góc với AC tại E . Chứng minh rắng đường tròn đường kinh DE đi qua điểm cố định thứ hai (khác điểm C ). ------Hết------ Thí sinh không sử dụng tài liệu. Giám thị không giải thích gì thêm. Họ và tên thi sinh:……………………....;Số báo danh:……………………. Chữ ki giám thị 1:…………………………………. Chữ kí giám thị 2:……………………..
  • 45. UBND TP. TAM KỲ PHÒNG GIÁO DỤC VÀ ĐÀO TẠO Đề chính thức KỲ KHẢO SÁT HỌC SINH GIỎI LỚP 9 LẦN 3 NĂM HỌC 2023 - 2024 MÔN TOÁN Ngày 06 tháng 03 năm 2024 Thời gian: 150 phút (không kể thời gian phát dề) Câu 1. (4.0 điểm) a) Cho ( ) 2 2 1 2 1 f n n n = + + − với n là số nguyên dương. Tính giá trị của biểu thức ( ) ( ) ( ) ( ) 1 2 3 60 . P f f f f = + + ++ b) Tìm tất cả các giá trị của tham số m để phương trình ( ) 2 2 2 3 1 4 0 x m x m m − − + − − = có hai nghiệm phân biệt 1 2 , x x thỏa mãn 1 2 1 2 1 2 1 2 2024 x x x x x x x x + + = − + − . Câu 2. (4.0 điểm) a) Giải phương trình: ( ) 4 3 2 4 1 32 80 50 3 x x x x x − − = − + − + . b) Giải hệ phương trình: 3 2 2 1 3 4 1 9 8 52 4 x y x x y x y xy  − + =   − + − − = − −   Câu 3. (4.0 điểm). Cho tam giác ABC , kẻ các đường phân giác trong , , AD BE CF của tam giác ABC . Chứng minh rằng a) AD AB AC BD DC =  −  b) 1 1 1 1 1 1 AB AC BC AD BE CF + +  + + Câu 4. (4.0 điểm). Cho đường tròn ( ) ; O R và điểm M cố định ở bên ngoài đường tròn. Từ M kẻ các tiếp tuyến , MA MB và cát tuyến MCD dến đường tròn ( ) ; O R , với , A B là các tiếp điểm. , C D thuộc đường tròn ( ) O sao cho , 2 MC MD CD R   . Gọi E là trung điểm của CD. a) Chứng minh bốn điểm , , , A E O B cùng nằm trên một đường tròn. b) Gọi F là giao điểm của AB và OE . Chứng minh FC là tiếp tuyến của đường tròn ( ) ; O R . c) Gọi T là điểm thay đổi trên cung nhỏ AB của đường tròn ( ) O . Tiếp tuyến tại T của đường tròn ( ) O cắt , MA MB lần lượt tại các điểm , I K . Chứng minh rằng chu vi tam giác MIK không đổi. Xác định vị trí của điểm T trên cung nhỏ AB sao cho tam giác MIK có diện tích lớn nhất. Câu 5. (5.0 điểm) a) Tìm tất cả các bộ ba số nguyên tố ( ) , , p q r sao cho 200 p q r pqr + + = − . b) Cho các số thực , , , 1 a b c d  − . Tìm tất cả các giá trị thực của k sao cho ( ) 3 3 3 3 1 Hêt a b c d k a b c d + + + +  + + + === === Lưu ý: Thí sinh không được phép sử dụng tài liệu. Giám thị không được gợi ý gì thêm. Họ và tên thí sinh............................................... Số báo danh……………………………….
  • 46. Hướng dẫn chấm môn Toán - Thi khảo sát HSG lớp 9 lần 3 năm học 2023-2024 UBND TP. TAM KỲ PHÒNG GIÁO DỤC VÀ ĐÀO TẠO KHẢO SÁT HỌC SINH GIỎI LỚP 9-LẦN 3 Năm hoc: 2023 - 2024 Ngày 06 tháng 03 năm 2024 Môn: TOÁN HƯỚNG DÃN CHÁM Câu Nội dung Điểm 1 (4.0 điểm ) a) (2.0 điểm) Ta có ( ) ( ) ( )( ) 2 2 1 2 1 2 1 2 1 2 1 2 1 n n f n n n n n + − − = + + − + − − (do 2 1 2 1 0) n n + − −  0.5 ( ) ( ) ( ) 2 2 1 2 1 2 1 2 1 2 1 2 1 n n n n n n + − − = = + − − + − − 0.5 Khi đó ( ) ( ) ( ) ( ) 3 1 5 3 7 5 121 119 P = − + − + − ++ − 0.5 121 1 11 1 10 = − = − = 0.5 b) (2.0 điểm) Phương trình ( ) ( ) 2 2 2 3 1 4 0 1 x m x m m − − + − − = có hai nghiệm phân biệt 2 Δ 0 8 5 5 0 m m   − +    0.25 2 5 135 8 0; 16 32 m m    − +        0.25 Theo định lí Vi-ét, ta có: ( ) 1 2 2 1 2 2 3 1 4 x x m x x m m + = −   = − −  1 2 x x xác định ( ) 2 1 2 0 4 0 * x x m m    − −  0.25 Đặt 1 2 1 2 1 2 1 2 ; u x x x x v x x x x = + + = + − Ta có ( ) 2 2 2 2 2 1 2 1 2 1 1 2 3 . 0; , 2 4 x x u v x x x x x x x   = + − = + +       Suy ra u và v luôn cùng dấu. Do đó u v u v + = + 0.25
  • 47. Hướng dẫn chấm môn Toán - Thi khảo sát HSG lớp 9 lần 3 năm học 2023-2024 Ta có 1 2 1 2 1 2 1 2 2024 x x x x x x x x + + + + − = 1 2 1 2 1 2 1 2 2024 x x x x x x x x  + + + + − = 1 2 1012 x x  + = 0.5 2 3 1 1012 m  − = 169 3 1 506 3 1 506 505 3 1 506 3 m m m m m =   − =    − =     − = − = −    (thỏa điều kiện ( )) * . 0.5 2 (4.0 điểm) a) (2.0 điểm ) Điều kiện: 1 x  0.25 Ta có ( ) 2 1 4 1 4 1 3 3 2 x x x   − − = − − +      0.5 Dấu đẳng thức xảy ra khi và chỉ khi 1 5 1 2 4 x x − =  = . 0.25 Lại có 2 4 3 2 2 5 32 80 50 3 3 32 3 4 x x x x x   − + − + = − −      0.5 Dấu đẳng thức xảy ra khi và chỉ khi 0 5 0 5 4 4 x x x x =     − =     =    0.25 Qua hai đánh giá trên ta thấy, phương trình đã cho có nghiệm duy nhất 5 4 x = . 0.25 b) (2.0 điểm ) Điều kiện: 1 y  − 0.25 Ta có: ( ) 2 2 3 3 2 1 3 2 1 3 4 1 ( 3) 4 6 5 x x x y y x y x y x x     − + =  + = −     + = − = − +   0.75 Thay 2 4 6 5 y x x = − + vào phương trình thứ hai của hệ phương trình ta được: ( ) ( ) ( ) 3 2 2 2 2 3 9 2 6 5 52 6 5 x x x x x x x x x − − − − − + = − − − + 0.5
  • 48. Hướng dẫn chấm môn Toán - Thi khảo sát HSG lớp 9 lần 3 năm học 2023-2024 2 3 4 21 0 7 x x x x = −   − − =   =  . So với điều kiện 3 x  , chọn nghiệm 7 x = Thay vào phương trình 2 4 6 5 y x x = − + ta được 3 y = . Vậy hệ phương trình đã cho có nghiệm duy nhất là: ( ) 7;3 . 0.5 3 (3.0 điểm) a) (1.5 điểm) Lấy K thuộc tia đối của tia DA sao cho AKB ACB = . Vì ( ) g g AD AC ACD AKB AB AC AD AK AB AK    =   =  (1) 0.5 Vì ( ) g.g . DC AC AD DAC DBK BD DC DK AD DK BK BD   = =   = (2) 0.5 Trừ (1) và (2) vế theo vế ta được ( ) 2 AB AC DC BD AD AK KD AD AD AD  −  =  − =  = hay AD AB AC BD DC =  −  . 0.5
  • 49. Hướng dẫn chấm môn Toán - Thi khảo sát HSG lớp 9 lần 3 năm học 2023-2024 b) (1.5 điểm) Qua B kẻ đường thẳng song song với AD, cắt đường thẳng AC tại M ABM  cân tại A AM AB  = . 0.25 Theo bất đẳng thức tam giác: 2 MB AM AB MB AB  +   0.25 Do / / AD BM nên AD CA AC AC BM CM AC AM AC AB = = = + + (do CM=AC+AM; AM=AB) AD AC BM AC AB  = + 2 2 AC AC AB AB AC AD BM AC AB AC AB AC AB    =   = + + + 1 1 1 1 1 1 2 2 2 2 AC AB AD AB AC AC AB AC AB +     = + =  +      (3) 0.5 Tương tự, 1 1 1 1 2 BE AB BC     +     0.25 1 1 1 1 2 CF AC BC     +     Cộng vế theo vế các bất đẳng thức (3), (4) và (5) ta được: 1 1 1 1 1 1 AD BE CF AB BC AC + +  + + 0.25 4 (4.0 điểm)
  • 50. Hướng dẫn chấm môn Toán - Thi khảo sát HSG lớp 9 lần 3 năm học 2023-2024 a) (1.0 điểm) Xét tứ giác AOBM có: 90 MAO = (vì MA là tiếp tuyến của đường tròn ( ) O ) 90 MBO = (vì MB là tiếp tuyến của đường tròn ( )) O 90 90 180 MAO MBO  + = + = Do đó tứ giác AOBM nôi tiếp đường tròn đường kính MO (1) 0.5 Vì E là trung điểm của đoạn thẳng CD nên OE CD ⊥ Xét MOE vuông tại E nên nội tiếp đường tròn đường kính ( ) 2 MO Từ (1) và (2) suy ra 5 điểm , , , , A E O B M cùng nằm trên đường tròn đường kính MO. Vậy bốn điểm , , , A E O B cùng nằm trên một đường tròn. 0.5 b) (1.0 điểm) ( ) g.g OE OM OEM OHF OH OF   = 2 2 OE OF OM OH OA OC   =  = = OE OC OC OF  = 0.5 Xét OEC và OCF có OE OC OC OF = và EOC COF = . Do đó OEC OCF  (c.g.c). Suy ra 90 OEC OCF = = . Mà ( ) D O  nên FC là tiếp tuyến của đường tròn ( ) ; O R . 0.5 c) (2.0 điểm) Do IK là tiếp tuyến của đường tròn ( ) O tại T nên IT IA = và KT KB = IK IT KT IA KB  = + = + Kí hiệu MIK C là chu vi ΔMIK Ta có: 2 MIK C MI MK IK MI MK IA KB MA MB MA = + + = + + + = + = mà điểm A và M cố định nên MA không đổi. Do đó MIK C không đổi 0.5 Qua O kẻ đường thẳng song song AB cắt MA tại 1 A , cắt MB tại 1 B  Ta có: 1 1 1 1 1 1 1 2 2 2 KOI KOT TOI BOT TOA AOB AOM MAB MB A = + = + = = = = Từ đó suy ra 1 1 Δ Δ IOA IOK OB K   (g.g) 1 1 1 1 IA OB OA B K  = hay 2 1 1 1 1 1 1 4 AB IA B K OA OB  =  = (không đổi) 0.5
  • 51. Hướng dẫn chấm môn Toán - Thi khảo sát HSG lớp 9 lần 3 năm học 2023-2024 Ta có: ( ) 1 1 1 1 1 1 1 1 1 2 MIK MA B KOI IOA KOB S S S S S MO AB R IK IA KB = − − − =  − + +     Mà 2 IK KM MA + = (chứng minh trên) nên ( ) ( ) 1 1 1 1 2 2 IK MA MI MK MA MA IA MB KB = − − = − − − − 1 1 1 2 2 MA MA IA KB = − + + ( ) 1 1 1 1 1 1 2 2 2 2 2 IMK S MO AB R MA MA IA KB  =  − − + +     ( ) 1 1 1 1 1 1 2 MO AB R AA R IA KB =  +  − + 0.5 Áp dụng bất đẳng thức AM GM − , ta có: 2 1 1 1 1 1 1 1 1 2 2 4 AB IA KB IA KB AB +   = = 1 1 1 1 1 1 2 IMK S MO AB R AA RAB    +  − . Dấu "=" xảy ra 1 1 n IA KB T AB  =   sao cho / / IK AB. 0.5 5 (5.0 diêm ) a) (2.5 điểm) Không mất tính tổng quát, giả sử p q r   . Phương trình đã cho tương đương với ( )( ) ( )( ) 1 1 1 1 202 qr p r q − − + − − = . Nếu p lẻ thì , q r cũng lẻ. Khi đó vế trái của phương trình trên chia hết cho 4 nhưng vế phải thì không (vô lí). Do đó 2 p = . 0.5 Thay 2 p = vào phương trình trên ta được ( )( ) 4 2 202 4 2 2 1 405 2 1 2 1 5.3 qr q r qr q r q r − − =  − − + =  − − = Do 3 2 1 2 1 q r  −  − nên ( )( ) 2 9 (2 1) 2 1 2 1 405 q q r  −  − − = Suy ra 3 2 1 20 q  −  . 0.5 Mà 2 1 q − lẻ nên   2 1 3;5;9;15 q −  . 0.25 Nếu 2 1 3 2 q q − =  = thì 68 r = (không phải là số nguyên tố nên loại). 0.25 Nếu 2 1 5 3 q q − =  = thì 41 r = (là số nguyên tố nên thỏa). 0.25 Nếu 2 1 9 5 q r − =  = thì 23 r = (là số nguyên tố nên thỏa). 0.25 Nếu 2 1 15 8 q q − =  = thì loại vì không phải là số nguyên tố. 0.25 Vậy tất cả các bộ ba số nguyên tố cần tìm là ( ) ( ) 2,5,23 , 2,3,41 cùng các hoán vị của nó. Lưu ý: Nếu thiếu cụm từ “các hoán vị của nó” thì trừ 0.25 điểm. 0.25
  • 52. Hướng dẫn chấm môn Toán - Thi khảo sát HSG lớp 9 lần 3 năm học 2023-2024 b) (2.5 điểm) Nếu 1 a b c d = = = = − thì ( ) 3 . 4 k −  − . Do đó 3 4 k  . 0.5 Nếu 1 2 a b c d = = = = thì 1 1 4 1 4 8 2 k    +        . Suy 3 4 k  . Vì vậy 3 4 k = . 0.5 Với 3 4 k = , ta sẽ chứng minh bất đẳng thức ( ) 3 3 3 3 3 1 , , , , 1. 4 a b c d a b c d a b c d + + + +  + + +   − Đầu tiên, ta chứng minh 3 4 1 3 , 1 x x x +    − . Thật vậy ( ) 3 2 4 1 3 1 (2 1) 0 x x x x +   + −  luôn đúng với mọi 1 x  − . 0.5 Áp dụng bất đẳng thức trên ta có 3 4 1 3 a a +  , 3 4 1 3 b b +  , 3 4 1 3 c c +  , 3 4 1 3 d d +  . 0.5 Cộng vế theo vế bốn bất đẳng thức trên ta được ( ) 3 3 3 3 3 1 , , , , 1. 4 a b c d a b c d a b c d + + + +  + + +   − Vậy 3 4 k = là giá trị cần tìm. 0.25 ===Hết=== Lưu ý : • Nhũng câu, nhũng ý nào thí sinh làm khác với đáp án nhung lập luận họpp lý và đáp số đúng vẫn đạt điểm tối đa ở câu đó, ý đó. • Nhũng kiến thức mới thí sinh đưa vào bài làm nếu đúng vẫn được chấp nhận bởi vì đây là đối tượng học sinh giỏi có năng khiếu Toán. Khuyến khích nhũng cách giải sáng tạo, độc đáo và có cộng điểm thưởng cho nhüng cách giải này.
  • 53. 45o 4m 20 m A B H C UBND THÀNH PHỐ HẢI DƯƠNG PHÒNG GIÁO DỤC VÀ ĐÀO TẠO ĐỀ KHẢO SÁT HỌC SINH LỚP 9 - LẦN 2 NĂM HỌC 2023 - 2024 MÔN TOÁN Thời gian làm bài: 120 phút (Đề gồm 06 câu, 01 trang) Câu 1. (2,0 điểm) a) Giải phương trình 2 1 5. 3 x − = b) Giải hệ phương trình ( ) 2 1 3 3 1 0 x y x y  − + =  − + =  . Câu 2. (2,0 điểm) a) Rút gọn biểu thức ( ) 2 5 1 : 4 3 6 2 x A x x x x x   + = − + −     + + − −   với 0, 4, 16 x x x    . b) Cho hai đường thẳng ( ) ( ) 2 : 1 2 3 d y m x m = + + − và ( ) ' : 10 6 d y x m = + − , với m là tham số. Tìm m để hai đường thẳng đã cho song song với nhau. Câu 3. (2,0 điểm) a) Công ty HD xây dựng kế hoạch cho 2 phân xưởng sản xuất với tổng sản phẩm làm được là là 520 sản phẩm. Tuy nhiên, các phân xưởng đều rất trách nhiệm và áp dụng kĩ thuật tốt nên đã nâng cao hiệu quả so với kế hoạch là 20% và tổng số sản phẩm sản xuất được của 2 phân xưởng là 596 sản phẩm. Hỏi theo kế hoạch ban đầu, mỗi phân xưởng làm bao nhiêu sản phẩm? b) Cho phương trình ( ) 2 2 2 1 2 0 x m x m − + + + = , với m là tham số. Tìm m để phương trình đã cho có hai nghiệm phân biệt 1 2 , x x thỏa mãn: ( ) 2 2 2 1 2 1 2 3 2 1 x x m x x − = + − . Câu 4 (1,0 điểm) Một người đứng ở vị trí A trên nóc một ngôi nhà cao 4 m đang quan sát một cây cao, cách ngôi nhà 20 m và đo được 45 BAC = (tham khảo hình vẽ). Tính chiều cao của cây đó (theo đơn vị mét, làm tròn kết quả đến hàng phần mười), biết rằng nếu góc  nhọn và thỏa mãn 1 tan 5  = thì ta chọn 11  = . Câu 5 (2,0 điểm) Cho tam giác nhọn ABC có AB AC  , các đường cao BD và CE cắt nhau tại H . a) Chứng minh rằng tứ giác ADHE nội tiếp đường tròn. b) Gọi M là trung điểm của BC , đường thẳng DE cắt BC tại N , AH cắt BC tại . K Chứng minh rằng DEK DMC = và NH AM ⊥ . Câu 6 (1,0 điểm) Cho , x y là các số thực không âm, thỏa mãn 2 x  và ( ) 2 2 xy y = + . Tìm giá trị lớn nhất của biểu thức 2 2 2 2 1 1 1 y y x x B x y x y + − = + + − + + . ĐỀ CHÍNH THỨC
  • 54. UBND THÀNH PHỐ HẢI DƯƠNG PHÒNG GIÁO DỤC VÀ ĐÀO TẠO ĐỀ KHẢO SÁT HỌC SINH LỚP 9 - LẦN 2 NĂM HỌC 2023 - 2024 MÔN TOÁN Thời gian làm bài: 120 phút (Đề gồm 06 câu, 01 trang) Câu Đáp án Điểm 1a a) Giải phương trình 2 1 5. 3 x − = + Ta có 2 1 5 2 1 15 3 x x − =  − = 0,5 2 16 8 x x  =  = + Vậy phương trình có nghiệm là 8 x = 0,5 1b b) Giải hệ phương trình ( ) 2 1 3 3 1 0 x y x y  − + =  − + =  . + Ta có: ( ) 2 3 2 1 3 3 1 3 1 0 x y x y x y x y + =  − + =     − = − − + =   0,25 6 3 15 7 14 2 3 1 3 1 3 1 x y x x x y x y x y + = = =          − = − − = − − = −    0,5 2 2 3 3 1 x x y y = =       = =   + Vậy hệ phương trình có nghiệm ( ) ( ) , 2; 1 x y = . 0,25 2a a) Rút gọn biểu thức ( ) 2 5 1 : 4 3 6 2 x A x x x x x   + = − + −     + + − −   với 0, 4, 16 x x x    . + Ta có: ( ) 2 5 1 : 4 3 6 2 x A x x x x x   + = − + −     + + − −   ( )( ) ( )( ) ( )( ) ( )( ) ( )( ) 2 2 5 3 1 . 4 3 2 3 2 3 2 12 1 . 4 3 2 x x x x x x x x x x x x x x x   + − +   = − −   − + − + − + −   − − = − + − 0,5 ( )( ) ( )( ) 3 4 1 1 . 4 2 3 2 x x x x x x + − = = − − + − + Vậy 1 2 A x = − 0,5 HƯỚNG DẪN CHẤM
  • 55. 2b b) Cho hai đường thẳng ( ) ( ) 2 : 1 2 3 d y m x m = + + − và ( ) ' : 10 6 d y x m = + − , với m là tham số. Tìm m để hai đường thẳng đã cho song song với nhau. + Điều kiện để hai đường thẳng song song là 2 1 10 2 3 6 m m m  + =  −  −  0,5 2 9 3 3 9 m m m  =   = −    Vậy 3 m = − 0,5 Câu 3a a) Công ty HD xây dựng kế hoạch cho 2 phân xưởng sản xuất với tổng sản phẩm làm được là là 520 sản phẩm. Tuy nhiên, các phân xưởng đều rất trách nhiệm và áp dụng kĩ thuật tốt nên đã nâng cao hiệu quả so với kế hoạch là 20% và tổng số sản phẩm sản xuất được của 2 phân xưởng là 596 sản phẩm. Hỏi theo kế hoạch ban đầu, mỗi phân xưởng làm bao nhiêu sản phẩm? + Gọi số sản phẩm theo kế hoạch mà phân xưởng thứ nhất và phân xưởng thứ hai làm được lần lượt là , x y (sản phẩm) ( ) , x y  + Tổng sản phẩm làm được là 520 sản phẩm nên 520 x y + = 0,25 + Thực tế, số sản phẩm phân xưởng thứ nhất làm được là 10% 1,1 x x x + = , số sản phẩm phân xưởng thứ hai làm được là 20% 1,2 y y y + = 0,25 + Số sản phẩm mà 2 phân xưởng làm được là 596 nên 1,1 1,2 596 x y + = Ta có hệ phương trình: 520 1,1 1,2 596 x y x y + =   + =  0,25 280 ( ) 240 x tmdk y =    =  Vậy theo kế hoạch, số sản phẩm 2 phân xưởng làm là 280 và 240 sản phẩm. 0,25 3b b) Cho phương trình ( ) 2 2 2 1 2 0 x m x m − + + + = , với m là tham số. Tìm m để phương trình đã cho có hai nghiệm phân biệt 1 2 , x x thỏa mãn: ( ) 2 2 2 1 2 1 2 3 2 1 x x m x x − = + − . + Điều kiện để phương trình có hai nghiệm phân biệt là ( ) ( ) 2 2 ' 0 1 2 0 m m    + − +  0,25 1 2 1 0 2 m m  −    + Vậy 1 2 m  th ì phương trình có hai nghiệm phân biệt 1 2 , x x 0,25 + Theo Vi – et: ( ) ( ) ( ) 1 2 2 1 2 2 1 1 1 2 x x m x x m + = +    = +   T ừ gi ả thi ết ( ) ( ) 2 2 2 2 2 2 1 2 1 2 1 1 2 2 3 2 1 2 3 2 1 x x m x x x x x x m − = + −  − − = − ( )( ) ( )( ) ( )( ) ( )( ) 1 2 1 2 1 2 3 2 1 1 2 1 3 2 1 1 x x x x m m m x x m m  + − = + −  + − = + − Vì 1 2 m  nên ( ) 2 1 0 m +  , do đó ( ) 1 2 3 1 3 x x m − = − 0,25
  • 56. 45o 4m 20 m K A B H C + Từ (1), (3) ta được 1 2 7 5 4 3 4 m x m x +  =    +  =   Thay vào (2): ( )( ) 2 2 1 7 5 3 2 9 26 17 0 17 16 9 m m m m m m m =  + +  = +  − + =   =  Kết hợp điều kiện 1 2 m  , ta có 1 m = thỏa mãn yêu cầu bài toán. Vậy 1 m = . 0,25 4 Một người đứng ở vị trí A trên nóc một ngôi nhà cao 4 m đang quan sát một cây cao, cách ngôi nhà 20 m và đo được 45 BAC =  (tham khảo hình vẽ). Tính chiều cao của cây đó (theo đơn vị mét, làm tròn kết quả đến hàng phần mười), biết rằng nếu góc  nhọn và thỏa mãn 1 tan 5  = thì ta chọn 11  = . + Ta có 1 tan 11 5 AH ABH ABH BH = =  =  0,25 + Kẻ AK vuông góc với BC tại K . Vì AK BH ∥ nên 11 KAB ABH = = . Do đó 45 11 34 CAK = −  =  0,25 + Xét tam giác ACK có ( ) tan .tan34 13,5 CK CAK CK AK m AK =  =   0,25 + Chiều cao của cây là: ( ) 4 13,5 17,5 BC BK KC m = +  + = 0,25 5a Cho tam giác nhọn ABC có AB AC  , các đường cao BD và CE cắt nhau tại H . a) Chứng minh rằng tứ giác ADHE nội tiếp đường tròn. H E D B C A
  • 57. + Do , BD CE là các đường cao cắt nhau tại H nên 90 AEH ADH = =  0,5 + Tứ giác ADHE có 180 AEH ADH + =  nên tứ giác ADHE nội tiếp đường tròn. 0,5 5b b) Gọi M là trung điểm của BC , đường thẳng DE cắt BC tại N , AH cắt BC tại . K Chứng minh rằng DEK DMC = và NH AM ⊥ . + Tứ giác ADHE nội tiếp nên DAH DEH = Tứ giác BKHE nội tiếp nên HBK HEK = Mà 90 DAH C HBK C DAH HBK + = + =  = Do đó 2 2 DEK DEH HEK HBK DBM = + = = 0,25 + Tam giác DBC vuông tại D v à c ó DM l à trung tuy ến DM BM MC  = = Có ( ) 2 2 DMC DBM BDM DBM = + = + Từ (1) và (2) ta có: DEK DMC = 0,25 + Gọi I là giao điểm của đường thẳng NH và đường tròn ngoại tiếp tứ giác ADHE. Ta sẽ chứng minh , , A I M thẳng hàng. Ta có tứ giác ADHE nội tiếp đường tròn đường kính AH , do đó 90 AIH =  (3) Có ( . ) . . NE NI NEI NHD g g NE ND NH NI NH ND    =  = ∽ 0,25 + Do DEK DMC = nên tứ giác DEKM nội tiếp Có ( . ) . . NE NK NEK NMD g g NE ND NM NK NM ND    =  = ∽ Vậy . . NH NK NH NI NK NM NM NI =  = . Tam giác NHK và NMI có chung góc N . Từ đó suy ra ( . . ) NHK NMI c g c   ∽ Do đó NHK NMI = nên tứ giác HIMK nội tiếp 90 HIM =  (4) + Từ (3), (4) ta có 180 AIH HIM + = , tức , , A I M thẳng hàng. Do vậy NH AM ⊥ . 0,25 I K N M H E D B C A
  • 58. 6 Cho , x y là các số thực không âm, thỏa mãn 2 x  và ( ) 2 2 xy y = + . Tìm giá trị lớn nhất của biểu thức 2 2 2 2 1 1 1 y y x x B x y x y + − = + + − + + . + Từ giả thiết ( ) ( ) ( ) ( )( ) 2 2 1 1 3 1 1 xy y x y x y = +  − + + + = − + ( )( ) 1 1 3 1 1 1 1 1 x y x y  + + = − + − + Ta cũng có ( ) ( ) 2 2 1 1 1 1 1 1 1 B x y x y = − + − + + − + 0,25 + Đặt ( ) 1 1 ; , 0 1 1 a b a b x y = =  − + Khi đó , a b thỏa mãn 3 1 a b ab + + = và 2 2 1 1 ab B a b a b = − + − + + + Ta có ( ) 2 3 1 3 4 a b ab a b a b = + +  + + + . Do đó ( ) ( ) ( )( ) 2 2 3 4 4 0 2 3 3 2 0 3 a b a b a b a b a b + + + −   + + + −   +  (vì 2 0 a b + +  , với mọi , 0 a b  ) 0,25 + Lại có ( ) ( ) 2 2 2 2 2 4 2 1 1 2 2 2 2 2 3 a b a b a b   + − + −  − −  −        ( ) ( ) ( ) 1 1 1 1 3 1 1 . 3 3 3 3 2 3 6 a b ab a b a b a b − + = = −  − = + + + Dấu “=” xảy ra khi 1 3 a b = = . 0,25 + Vậy 2 2 4 2 1 8 2 1 1 1 3 6 6 ab B a b a b + = − + − +  + = + Giá trị lớn nhất của B là 8 2 1 6 + . Dấu “=” xảy ra khi 1 3 a b = = , khi đó 4 2 x y =   =  0,25
  • 59. PHÒNG GIÁO DỤC VÀ ĐẠO TẠO HUYỆN BÁ THƯỚC ĐỀ KHẢO SÁT CHẤT LƯỢNG LỚP 9 NĂM HỌC 2023 – 2024 Môn: Toán Thời gian: 120 phút (không kể thời gian giao đề) Ngày thi: …../03/2024 Đề thi có: 01 trang gồm 05 câu. Câu 1. (2,0 điểm): 1. Giải phương trình: 2 3 4 1 0 x x − + = 2. Giải hệ phương trình:  − =  − =  2 7 3 6 x y x y Câu 2. (2,0 điểm): Cho biểu thức 4 8 2 : 3 , 4 2 2 x x x P x x x     + = − +     − + −     với 0, 1 x x   và 4 x  1. Rút gọn biểu thức 2. Tìm các giá trị của x để 4 P = − Câu 3. (2,0 điểm): 1. Cho đường thẳng ( ): d y ax b = + . Tìm , a b để đường thẳng ( ) d song song với đường thẳng ( ) ' : 2 3 d y x = + và đi qua điểm ( ) 1; 1 A − . 2. Cho phương trình 2 1 0 − + − = x nx n (n là tham số, x là biến số). Tìm các giá trị của n để phương trình có hai nghiệm phân biệt. Câu 4. (3,0 điểm): Cho nửa đường tròn đường kính AB = 2R. Từ A và B kẻ hai tiếp tuyến Ax, By. Qua điểm M thuộc nửa đường tròn kẻ tiếp tuyến thứ ba cắt các tiếp tuyến Ax , By lần lượt ở C và D. Các đường thẳng AD và BC cắt nhau tại N. 1. Chứng minh AC + BD = CD. 2. Chứng minh OC // BM 3. Xác định vị trí của M để chu vi tứ giác ACDB đạt giá trị nhỏ nhất. Câu 5. (1,0 điểm): Cho các số thực dương , , x y z thỏa mãn 3. x y z + + = Chứng minh rằng: 1 1 1 3 xy x y yz y z zx z x + +  + + + + + + ……….. Hết ……… Cán bộ coi thi không giải thích gì thêm. Họ và tên thí sinh …………………………………….Số báo danh……… ĐỀ CHÍNH THỨC
  • 60. 2 PGD&ĐT BÁ THƯỚC HƯỚNG DẪN VÀ BIỂU CHẤM TOÁN 9 NĂM HỌC 2023 – 2024 Câu NỘI DUNG Điểm Câu 1 (2,0 điểm) 1. Giải phương trình: 2 3 4 1 0 x x − + = 1,0 Ta có: ( ) + + = + − + = 3 4 1 0 a b c Nên phương trình đã cho có hai nghiệm phân biệt 1 1 x = và = 2 1 3 x 0,75 Vậy phương trình có hai nghiệm phân biệt 1 1 x = và = 2 1 3 x 0,25 2. Giải hệ phương trình: 2 7 3 6 x y x y  − =   − =  1,0  − =  − =  2 7 3 6 x y x y 2 7 2 6 12 x y x y  − =   − =  2 7 5 5 x y y  − =   = −  0,5 2 1 7 1 x y  + =   = −  3 1 x y  =   = −  0,25 Vậy hệ phương trình có nghiệm duy nhất là  =  = −  3 1 x y 0,25 Câu 2 (2,0 điểm) Câu2. (2,0 điểm): Cho biểu thức 4 8 2 : 3 , 4 2 2 x x x P x x x     + = − +     − + −     với 0, 1 x x   và 4 x  1. Rút gọn biểu thức 2. Tìm các giá trị của x để 4 P = − 2 1. Rút gọn biểu thức P. 1
  • 61. 3 Với 0, 1, 4 x x x    ( ) ( ) ( ) ( ) ( ) ( ) ( ) ( ) 4 8 2 : 3 4 2 2 4 8 2 3 6 : 2 2 2 2 . 2 4 8 8 4 4 : 2 2 . 2 4 2 2 . 2 . 2 4 1 1 x x x P x x x x x x x x x x x x x x x x x x x x x x x x x x x     + = − +         − + −         + −   = − +       + − − + −       − − −   =   − + −   − + − = + − − − = − Vậy với 0, 1, 4 x x x    thì 1 x A x − = − 0,25 0,25 0,25 0,25 2.Tìm các giá trị của x để 4 P = − 1 Với 0, 1, 4 x x x    4 4 4 4 4 3 4 3 1 x P x x x x x − = −  = −  = −  − = −  = − 16 9 x  = (thỏa mãn điều kiện). 0,5 0,25 Vậy 16 9 x = thì 4 P = − . 0,25 Câu 3 2,0 điểm 1. Cho đường thẳng ( ): d y ax b = + . Tìm , a b để đường thẳng ( ) d song song với đường thẳng ( ) ' : 2 3 d y x = + và đi qua điểm ( ) 1; 1 A − . 1 Do ( ) ( ) ' // d d nên 2 3 a b =     . 0,25 Do ( ) d đi qua điểm ( ) 1; 1 A − nên: 1 2.1 3 b b − = +  = − (thỏa mãn điều kiện 3 b  ) . Vậy 2 a = , 3 b = − . 0,5 Vậy 2 a = , 3 b = − . 0,25
  • 62. 4 2. Cho phương trình 2 1 0 − + − = x nx n (n là tham số, x là biến số). Tìm các giá trị của n để phương trình có hai nghiệm phân biệt. 1,0 Ta có : ( ) ( ) ( ) 2 2 4 1 2 0, 2 n n n n  = − − − = −    Suy ra với mọi 2  n phương trình có hai nghiệm phân biệt 1 2 , x x 1 Câu 4 3,0 điểm Câu 4 : Cho nửa đường tròn đường kính AB = 2R. Từ A và B kẻ hai tiếp tuyến Ax, By. Qua điểm M thuộc nửa đường tròn kẻ tiếp tuyến thứ ba cắt các tiếp tuyến Ax , By lần lượt ở C và D. Các đường thẳng AD và BC cắt nhau tại N. 1. Chứng minh AC + BD = CD. 1,0 1/ Theo tính chất hai tiếp tuyến cắt nhau ta có: CA = CM; DB = DM => AC + BD = CM + DM. Mà CM + DM = CD => AC + BD = CD 0,5 0,5 2. Chứng minh OC // BM 1,0 2/ Theo tính chất hai tiếp tuyến cắt nhau ta có: OC là tia phân giác của góc AOM; OD là tia phân giác của góc BOM, mà AOM và BOM là hai góc kề bù => COD = 900 . Theo trên COD = 900 nên OC ⊥ OD . (1) Theo tính chất hai tiếp tuyến cắt nhau ta có: DB = DM; lại có OM = OB =R => OD là trung trực của BM => BM ⊥ OD . (2). Từ (1) Và (2) => OC // BM ( Vì cùng vuông góc với OD). 0,25 0,25 0,25 0,25 3. Xác định vị trí của M để chu vi tứ giác ACDB đạt giá trị nhỏ nhất. 1,0 3/ Ta có chu vi tứ giác ACDB = AB + AC + CD + BD mà AC + BD = CD => Chu vi tứ giác ACDB = AB + 2CD mà AB không đổi 0,25 0,25 / / y x N C D I M B O A
  • 63. 5 Chú ý: - Các cách làm khác nếu đúng vẫn cho điểm tối đa - Bài hình nếu hình vẽ sai cơ bản hoặc không có hình thì không chấm bài hình.. - Các trường hợp khác tổ chấm thống nhất phương án chấm. => Chu vi tứ giác ACDB nhỏ nhất khi CD nhỏ nhất , mà CD nhỏ nhất khi CD là khoảng cách giữa Ax và By tức là CD vuông góc với Ax và By. Khi đó CD // AB => M phải là trung điểm của cung AB. 0,25 0,25 Câu 5 1,0 điểm Câu 5. Cho các số thực dương , , x y zthỏa mãn 3. x y z + + = Chứng minh rằng : 1 1 1 3 xy x y yz y z zx z x + +  + + + + + + 1 Ta chứng minh ( ) ( ) 2 1 3 x y xy x y + +  + + với mọi , x y Thật vậy, bất đẳng thức trên tương đương với : ( ) ( ) 2 2 2 1 2 2 2 6 x y xy x y xy x y + + + + +  + + ( ) ( ) ( ) 2 2 2 1 1 x y x y  − + − + − . Dấu " " = xảy ra 1 x y  = = Do đó 1 3 1 x y xy x y  + + + + , với , 0 x y  . Dấu " " = xảy ra 1 x y  = = Tương tự ta suy ra 1 1 1 3 3 3 1 1 1 x y y z z x xy x y yz y z zx z x + +  + + + + + + + + + + + + + + Dấu " " = xảy ra ( ) 1 1 x y z  = = = Ta chứng minh 1 1 1 9 , m n p m n p + +  + + với mọi , , 0 m n p  Thật vậy, bất đẳng thức trên tương đương với : 1 1 1 9 n p m p m n m m n n p p + + + + + + + +  6 n m p m p n m n m p n p        + + + + +              Theo bđt Cô si ta thấy bất đẳng thức trên luôn đúng. Dấu " " = xảy ra m n p  = = Do đó : ( ) ( ) 3 3 3 9 3 3 2 1 1 1 2 3 x y y z z x x y z + +  = + + + + + + + + + Từ (1) và (2) suy ra điều phải chứng minh . Dấu " " = xảy ra 1 x y z  = = = 0,25 0,25 0,25 0,25
  • 64. PHÒNG GIÁO DỤC VÀ ĐÀO TẠO QUẬN HÀ ĐÔNG ĐỀ CHÍNH THỨC ĐỀ KHẢO SÁT CHẤT LƯỢNG LỚP 9 (LẦN 1) Năm học 2023-2024 Môn: TOÁN Thời gian làm bài:120 phút (không kể thời gian giao đề) (Đề gồm có 01 trang) Câu I. (2,0 điểm) Cho hai biểu thức 3 1 5 2 ; 4 2 2 x x x P Q x x x + − − = = − − − + với 0, 4 x x   . 1) Tính giá trị của biều thức P khi 25 x = 2) Chứng minh 2 x Q x = − 3) Tìm giá trị của x để biểu thức P Q đạt giá trị nhỏ nhất. Câu II. (2,0 điểm) 1) Giải bài toán bằng cách lập phương trình hoặc hệ phương trình Hai bạn Linh và Chi ở hai địa điểm cách nhau 18 km đạp xe đi ngược chiều nhau để gặp nhau. Nếu hai bạn khởi hành cùng một lúc thì sẽ gặp nhau sau 40 phút. Nhưng nếu Linh khởi hành trước 18 phút thi các bạn sẽ gặp nhau sau 30 phút tính từ lúc Chi bắt đầu đi. Tính vận tốc của mỗi bạn? 2) Một chiếc cốc có dạng hình trụ với chiều cao 8 cm, bán kính đáy là 3 cm. Hỏi chiếc cốc này có đựng được 200ml sữa không? (Lấy 3,14   và bỏ qua bề dày của chiếc cốc). Câu II. (2,0 điểm) 1) Giài hệ phương trình: 2 2 1 8 2 3 1 3 x y x y  − + − =   − − − = −   2) Cho Parabol ( ) 2 P : 2 x y = và đường thẳng (d): 1 y mx m = − + (với m là tham số). a) Tìm m để (d) cắt ( ) P tại 2 điểm phân biệt A và B nằm về hai phía của trục tung. b) Gọi K là giao điểm của (d) và trục Oy . Tìm m để 2 KOA KOB S S = Câu IV. (3,0 điểm) Từ điểm A ở ngoài đường tròn ( ) O;R vẽ hai tiếp tuyến AB,AC(B,C là các tiếp điểm) và cát tuyến ADE thuộc nửa mặt phẳng bờ là đường thẳng OA không chứa điểm B của đường tròn ( ) O . Gọi H là giao điểm của OA và BC. 1) Chứng minh bốn điểm A,B,O,C cùng thuộc một đường tròn. 2) Chứng minh AO BC ⊥ tại H và AH.AO AD.AE = 3) Đường thẳng đi qua điểm D và song song với đường thẳng BE cắt AB,BC lần lượt tại I,K . Chứng minh tứ giác OHDE nội tiếp và D là trung điểm của IK . Câu V. (0,5 điểm) Với các giá trị của m để phương trình ( ) 2 2 2 1 9 0 x m x m − + + + = có nghiệm, hãy tìm giá trị nhỏ nhất của biểu thức ( )( ) 2 2 2 4 m m m B m + − + = ---------------Hết---------------
  • 65. PHÒNG GIÁO DỤC VÀ ĐÀO TẠO QUẬN HÀ ĐÔNG HƯỚNG DẪN CHẤM MÔN TOÁN KHẢO SÁT CHẤT LƯỢNG LỚP 9 (LẦN 1) Năm học 2023-2024 Câu Nội dung Điểm Câu I (2.0 điềm) 1) Tính giá trị của P khi 25 x = với 0, 4 x x   . 0,5 Ta có ( ) x 25 TMC = thay vào biểu thức P được 25 3 25 2 P + = − 0,25 Tính được 28 28 5 2 3 P = = − 0,25 2) Chứng minh 2 x Q x = − với 0, 4 x x   1,0 ( )( ) ( )( ) 1 2 5 2 1 5 2 1 5 2 4 4 2 2 2 2 x x x x x x x Q x x x x x x − − + − − − − − = − = + = − − + + + − 0,25 ( )( ) 3 2 5 2 2 2 x x x x x − + + − = + − 0,25 ( )( ) 2 2 2 x x x x + = + − 0,25 ( ) ( )( ) 2 2 2 2 x x x x x x + = = − + − 0,25 3) Tìm giá trị của x để biểu thức P Q đạt giá trị nhỏ nhất 0,5 * Với 0, 4 x x   biến dối 3 3 2 3 3 : 2 2 2 P x x x x x x Q x x x x x x + + − + = =  = = + − − − * Vì 3 0, 4 0, 0 x x x x      Áp dụng BĐT Cosi cho 2 số 3 ; x x ta có: 3 3 2 2 3 P x x Q x x = +   = Dấu "=" xảy ra khi ( ) 3 3 x x TM x =  = Vậy Min 2 3 3 P x Q =  = 0,25